Daily Questions Flashcards

1
Q

After taking a split thickness skin graft from the
postauricular skin you ask for Monsel’s solution to
obtain hemostasis. Unfortunately, they do not have
it available in the operating room and the nurse
asks what the solution is made of so he can look
for it in the storage center. What is Monsel’s made
of?

A) Resorcinol, salicylic acid, lactic acid, and ethanol
B) Glycolic Acid 10% C) Glycolic Acid 70% D)
Trichloroacetic 50% E) Ferric Subsulfate
c

A

E: Monsel’s solution is a hemostatic agent made of ferric subsulfate that works well on split thickness skin graft donor sites although it can cause discoloration of the skin if left on to long. Resorcinol, salicylic acid, lactic acid and ethanol make up Jessner’s solution which is used for light and medium chemical peels. Glycolic acid can be used for light and medium chemical peels at 10% and 70% respectively. Trichloroacetic (TCA) is similarly used for chemical peels. - See Wikipedia “Ferric subsulfate solution”

How well did you know this?
1
Not at all
2
3
4
5
Perfectly
2
Q

A 14 year old male is brought to the clinic for
evaluation of hearing loss. He has bilateral
symmetrical sensorineural hearing loss which has
been stable on audiogram for several years. He
denies vestibular symptoms. He endorses
worsening vision which has progressed over the
last 2 years as well. His exam and a CT temporal
bone are unremarkable. A mutation in which of the
following genes is responsible for this patients
condition?

A) EYA1 B) TCOF1 C) USH2A D) SCL26A E)
GJB2

A

C; This patient’s presentation is consistent with Usher’s syndrome. These patients typically have moderate to profound hearing loss, may or may not have vestibular symptoms and develop vision loss in the 1st or 2nd decade of life. While Usher’s syndrome can be caused by a multitude of genetic mutations, the most common one is USH2A. Mutations in EYA1 causes branchio-oto-renal syndrome. TCOF1 is associated with treacher collins syndrome. SCL26A mutations lead to pendred/enlarged vestibular aqueduct. GJB2 mutations causes deficiencies in connexin 22 proteins and
sensorineural hearing loss. - See Wikipedia “Usher Syndrome”

How well did you know this?
1
Not at all
2
3
4
5
Perfectly
3
Q

Which of the following is true regarding the
submandibular gland?

A) Produces 70% of unstimulated saliva B)
Produces thinner saliva relative to the parotid gland
C) Receives parasympathetic innervation via the
glossopharyngeal nerve D) Produces 50% of all
salivary gland calculi E) Are the first salivary glands
to be formed during embryogenesis

A

A; The submandibular gland produces the majority of saliva in the unstimulated state. Upon eating, saliva production from the parotid gland increases significantly bringing the submandibular glands total share of production down to ~ 50%. The submandibular gland
produces thicker, more viscous saliva compared with the parotid gland, receives parasympathetic innervation from the facial (via the chorda tympani), produces 80% of all salivary calculi and is formed later than the parotid gland during embryogenesis. - See Wikipedia
“Submandibular Gland”

How well did you know this?
1
Not at all
2
3
4
5
Perfectly
4
Q

How many openings are there from the
semicircular canals into the vestibule?

A) 2 B) 3 C) 4 D) 5 E) 6

A

D; There are three semicircular canals and each has both an ampulated and nonampulated end. However, the posterior and superior canals share a common crus which opens into the vestibule. Therefore the 5 openings into the vestibule are the ampulated ends
of all 3 canals, the nonampulated end of the lateral canal and the common crus of the posterior and superior canal, for a total of 5 openings into the vestibule. - See Wikipedia “Semicircular Canals”

How well did you know this?
1
Not at all
2
3
4
5
Perfectly
5
Q

Which of the following is true regarding pediatric
subglottic stenosis?

A) The subglottis in the pediatric population is
prone to injury as it is the narrowest portion of the
airway, the subglottic mucosa is dense preventing
significant edema to develop and the stratified
squamous epithelium is particularly delicate B)
Coexistence of gastric reflux disease and need for
repeated intubations are the most important factors
in the development of acquired subglottic stenosis
C) Premature infants develop subglottic stenosis
much faster (within days) compared with older
children or adults D) The endotracheal tube should
be small enough to allow a cuff leak at 20 cm water
pressure E) 65% of acquired pediatric subglottic
stenosis is secondary to intubation

A

D; In order to prevent subglottic stenosis in the pediatric population management of the pressure exerted on the tracheal wall is critical. Excessive cuff pressure or a endotracheal tube that is too large will exert significant pressure on the airway walls damaging the delicate pseudostratified columnar mucosa and leading to significant airway edema. This can lead to scarring and ultimately stenosis. To help prevent this, the pressure should be light enough to allow for a cuff leak at 20 cm water. The subglottis is the narrowest portion of the airway in children but the mucosa is loose and allows significant edema and the mucosa is pseudostratified columnar, not stratified squamous cells. While reflux and repeated intubations are risk factors, duration of intubation and size of the endotracheal tube are the most important factors that affect the development of subglottic stenosis. Premature infants can tolerate several weeks of intubation without developing stenosis. 90% of acquired pediatric subglottic stenosis is due to intubation, not 65%.

  • See Cummings 6th ed pg 3160
How well did you know this?
1
Not at all
2
3
4
5
Perfectly
6
Q

What percentage drop in systolic blood pressure
(BP) is expected upon induction of anesthesia
using propofol?

A) No drop in BP B) 5-20% drop in BP C) 20-30%
drop in BP D) 40-50% drop in BP E) More than
50% drop in BP in most patients.

A

C; Propofol causes a drop in arterial blood pressure of ~ 20-30%. Additional side effects include local pain on injection, apnea, airway obstruction and oxygen desaturation. Advantage includes rapid hypnosis which can be achieved in ~ 40 seconds. -See KJ Lee 10th
ed pg 839.

How well did you know this?
1
Not at all
2
3
4
5
Perfectly
7
Q

What is the function of the internal branch of the
superior laryngeal nerve?

A) Sensory innervation to the mucosa inferior to the
true vocal cords B) Sensory innervation to mucosa
superior to the true vocal cords C) Motor
innervation to the cricothyroid muscle D) Motor
innervation to the posterior cricoarytenoid muscle
E) Motor innervation to the thyroarytenoid muscle

A

B; The internal branch of the superior laryngeal nerve provides sensation to the distal pharynx superior to the true vocal cords. Damage to this nerve can increase risk of aspiration. The external branch of the superior laryngeal nerve provides motor sensation to the cricothyroid. The recurrent laryngeal nerve provides sensation below the true vocal cords and innervates the cricoarytenoid and thyroarytenoid muscles. -See KJ Lee 10th ed pg 577.

How well did you know this?
1
Not at all
2
3
4
5
Perfectly
8
Q

What is the mode of inheritance for Fanconi
Anemia Syndrome?

A) Autosomal Dominant B) Autosomal Recessive
C) X-linked Dominant D) X-linked Recessive E)
Mitochondrial Inheritance

A

B; Fanconi Anemia Syndrome is characterized by absent/deformed thumbs accompanied by other malformations involving the heart and kidneys, increased skin pigmentation, mental retardation,
pancytopenia and conductive hearing loss. Death due to leukemia usually occurs within the first 2 years of life. -See KJ Lee 10th ed pg
135.

How well did you know this?
1
Not at all
2
3
4
5
Perfectly
9
Q

What is the full range of human hearing?

A) 20-12,000 Hz B) 20-20,000 Hz C) 100-12,000
Hz D) 100 - 20,000 Hz E) 5 - 18,000 Hz

A

B; Although most audiograms only test hearing between 250-8,000 Hz, the full range of human hearing is 20-20,000 Hz. Speech sounds are mostly contained within 250 - 6,000 Hz. -See KJ Lee 10th ed pg
24.

How well did you know this?
1
Not at all
2
3
4
5
Perfectly
10
Q

80% of accidental tracheostomy decannulation
occur in patients with what risk factor?

A) Decreased nurse:patient ratio B) BMI 30+ C)
Altered Mental Status D) Increased secretions

A

B; Obesity is a major risk factor for complications related to tracheostomy. It is often considered a contraindication to percutaneous tracheostomy. 15% of obese patients will experience complications related to tracheostomy compared to 8% of nonobese
patients. -See Cummings 6th ed pg 99.

How well did you know this?
1
Not at all
2
3
4
5
Perfectly
11
Q

Korner’s septum is a remnant of what structure?

A) Tympanomastoid suture B) Tympanosquamous
suture C) Tympanoparietal suture D) Petromastoid
suture E) Petrosquamous suture

A

E; Korner’s septum (which must be opened to expose the mastoid antrum) is a remnant of the petrosquamous suture. -See Cummings 6th ed pg 2191

How well did you know this?
1
Not at all
2
3
4
5
Perfectly
12
Q

Which facial muscle is responsible for “bunny
lines”?

A) Frontalis B) Depressor Supercilli C) Corrugator
D) Procerus E) Nasalis

A

E; Frontalis can create horizontal forehead wrinkles whereas the corrugators (and to some degree depressor supercilii) create vertical wrinkles in the glabella. Both procerus and nasalis contribute to
“bunny lines” however nasalis is the dominant contributor and should be targeted with botulinum toxin to address this cosmetic issue. - Becker-Wegerich, P., Rauch, L. and Ruzicka, T. (2001), Botulinum toxin A in the therapy of mimic facial lines. Clinical and Experimental
Dermatology, 26: 619–630.

How well did you know this?
1
Not at all
2
3
4
5
Perfectly
13
Q

This structure forms a fibrous sling stretching below
the eyeball and blends with the cheek ligaments as
well as the medial/lateral horns of the levator
palpebrae superioris aponeurosis. Name that
structure.

A) Lockwood Ligament B) Whitnall Ligament C)
Tarsal Plate D) Medial Canthal Tendon E) Lateral
Canthal Tendon

A

A; The ligament of Lockwood is a suspensory ligament that is located underneath the eyeball and acts as a sling. It attaches to the cheek ligaments as well as the medial and lateral horns of the aponeurosis of the levator muscle. Whitnall ligament suspends the levator palpebrae superioris muscle and extends from its attachment to the trochlea over to the lateral orbital tubercle (Whitnall tubercle). The tarsal plates are plates of dense connective tissue that provide structure to the eyelids. The medial and lateral canthal tendons attach the tarsus to the bony structures of the orbit. -See KJ Lee 10th ed pg 896.

How well did you know this?
1
Not at all
2
3
4
5
Perfectly
14
Q

An 8 y/o male presents with left orbital pain,
swelling and double vision. The patient’s mother
states he had a cold for the past two weeks and
“gross stuff” coming out of his nose. A CT scan is
ordered and a small fluid collection underneath the
periosteum is found along the medial floor of the
left orbit. Despite the patient’s diplopia his vision is
intact and he has no other significant findings on
neurologic exam. What is the most appropriate
Chandler’s classification for this patient?

A) Group I B) Group II C) Group III D) Group IV E)
Group V

A

C; The patient’s CT scan indicates a subperiosteal abscess which is categorized as Group III. Group I = Periorbital edema (can treat w/ oral antibiotics and close observation) Group II = Periorbital cellulitis Group III = Subperiosteal abscess Group IV = Orbital abscess Group
V = Cavernous sinus thrombosis. -See KJ Lee 10th ed pg 987.

How well did you know this?
1
Not at all
2
3
4
5
Perfectly
15
Q

A 58 y/o female patient presents to your clinic for
evaluation of the aging face. You offer her an
endoscopic brow lift to treat her brow ptosis and
dynamic rhytids of the forehead. What is the best
landmark used intraoperatively to alert the surgeon
that they are close to the temporal branch of the
facial nerve?

A) Deep layer of the deep temporal fascia B)
Superficial layer of the deep temporal fascia C)
Zygomaticotemporal vein D) Superficial temporal
artery E) Supraorbital neurovascular bundle

A

C; The zygomaticotemporal vein (aka sentinel vein) has been shown to be a consistent landmark used to locate the temporal branch of the facial nerve in the endoscopic brow lift procedure. This structure
should be considered a marker that the surgeon is entering nerve territory and caution should be taken. When performing a Gillies procedure the plane used is deep to the superficial layer of the deep temporal fascia in order to protect the facial nerve (which is above
this) but an endoscopic brow lift dissection is performed directly on the skull so these structure do not come into play. The supraorbital neurovascular bundle is not a reliable landmark to use in order to locate the facial nerve.-Trinei, Filippo A., Janek Januszkiewicz, and
Foad Nahai. “The Sentinel Vein: An Important Reference Point for Surgery in the Temporal Region.” Plastic and Reconstructive Surgery
101.1 (1998): 27-32.

How well did you know this?
1
Not at all
2
3
4
5
Perfectly
16
Q

A 47 y/o F presents to your clinic with nasal
congestion, sinus pressure and thick nasal
discharge. She has not responded to medical
management so a CT scan is ordered which shows
complete opacification of a single sinus. On your
review of the scan there is a single, well
circumscribed radiodense mass similar to the
surrounding bone which appears to be obstructing
the affected sinus. Given the nature of these
masses, what is the most likely sinus affected in
this patient?

A) Ethmoid B) Frontal C) Maxillary D) Sphenoid E)
Unable to determine from the information provided

A

B; The description of the mass is consistent with an osteoma. These are benign lesions, however they can cause obstruction of sinus outflow tracts. They are most commonly found in the frontal sinus. The ethmoids are the second most common location followed by the
maxillary sinuses. -See KJ Lee 10th ed pg 672

How well did you know this?
1
Not at all
2
3
4
5
Perfectly
17
Q

Elective neck dissection is indicated for patients
with clinically and radiographically N0 necks if their
risk of micrometastases is - - - ?

A) 5% B) 10% C) 15% D) 20% E) All patients
should undergo elective neck dissection

A

D; It is generally accepted that if a patient’s risk of occult metastasis is 20% or greater, elective neck dissection should be performed. The rate of occult mets is estimated based on the patient’s primary site
and tumor staging. -See KJ Lee 10th ed pg 699

How well did you know this?
1
Not at all
2
3
4
5
Perfectly
18
Q

Which of the following is not a cause of
angioedema?

A) Food Allergy B) Lisinopril C) C1 Esterase
Deficiency D) Idiopathic E) All of the above are
causes of angioedema

A

E; Angioedema can be caused by hypersensitivity reactions, ACE inhibitors (of which lisinopril is one), and genetic causes such as C1 esterase deficiency. Most cases of angioedema however are idiopathic. Angioedema should be managed w/ epinephrine,
benadryl, steroids and above all protection of the airway. Intubation or tracheostomy may be required. -See KJ Lee 10th ed pg 518

How well did you know this?
1
Not at all
2
3
4
5
Perfectly
19
Q

Your junior resident cuts his hand while practicing
emergency tracheotomies on a plastic model. One
week later you see him in the halls and he tells you
that his cut has fully healed. Trying to turn this into
a teaching moment you ask him what the tensile
strength of his wound is compared to his preinjury
state. He answers correctly by saying - - - ?

A) 1% B) 10% C)30% D) 50%

A

B; The wound tensile strength at 1 week after injury is ~ 10%. This occurs during the proliferative phase (24hrs - 6 wks). By 10 weeks the tensile strength is up to 80%. -See KJ Lee 10th ed pg 765-766

How well did you know this?
1
Not at all
2
3
4
5
Perfectly
20
Q

—- cancer is more likely to present at an early
stage, whereas — cancer tends to present at more
advanced stages. —- cancer is quite rare.
A) Supraglottic - Subglottic - Glottic B) Supraglottic
- Glottic - Subglottic C) Glottic - Subglottic -
Supraglottic D) Glottic - Supraglottic - Subglottic

A

D; Because even small masses can cause changes in the vibratory pattern of the vocal cords, even small glottic cancers tend to cause symptoms and therefore present at an early stage. Additionally, the glottis is relatively devoid of lymphatics and does not metastasize as easily making presentation without regional mets more common. In contrast, supraglottic tumors do not cause symptoms until they are significantly larger and have a rich network of lymphatics making regional metastasis more common at presentation. Subglottic
malignancy is rare. -See Lalwani 3rd ed pg 458-460

How well did you know this?
1
Not at all
2
3
4
5
Perfectly
21
Q

The stapedius tendon attaches to what structure?

A) Ponticulus B) Subiculum C) Pyramidal process
D) Cochleariform process E) Neck of the malleus

A

C; The stapedius is the smallest muscle in the human body. It spans from the pyramidal process to the neck of the stapes and is innervated by CN VII. The tensor tympani makes a sharp turn at the cochleariform process and attaches to the neck of the malleus. It is
innervated by CN V. The ponticulus and subiculum are not attachment points for any middle ear muscles. -See KJ Lee 10th ed pg 13

How well did you know this?
1
Not at all
2
3
4
5
Perfectly
22
Q

A 68 y/o male presents to your clinic with
complaints of persistent daytime fatigue. He has a
BMI of 31, hypertension and headaches that are
worse in the morning. His wife tells him that he
snores loudly and sometimes stops breathing at
night. You order a sleep study. During a sleep
study, what is the definition of apnea?
A) Cessation of airflow B) Cessation of airflow for
>5 seconds C) Cessation of airflow for >5 seconds
with oxygen desaturation to < 95% D) Cessation of
airflow for >10 seconds E) Cessation of airflow for
>10 seconds with oxygen desaturation to < 95%

A

D; Apnea is defined as cessation of airflow for greater than 10 seconds. While blood oxygen saturation is also measured, apnea can occur with or without a decrease in blood oxygen saturation. Hypopnea is more difficult to define but is either a reduction in airflow of 50% for 10 seconds or longer, or is a reduction in airflow greater
than 30% lasting at least 10 seconds and associated with at least a 4% decrease in blood oxygen levels. -See KJ Lee 10th ed pg 247

How well did you know this?
1
Not at all
2
3
4
5
Perfectly
23
Q

A friend of yours is worried about developing sea
sickness on an upcoming fishing trip. You
recommend he take some Meclizine with him to
treat any potential nausea. Meclizine is what kind
of medication?

A) Cholinergic Agonist B) First Generation
Antihistamine C) Second Generation Antihistamine
D) Third Generation Antihistamine E) Dopamine
Agonist

A

B; Meclizine (aka Antivert/Bonine) is a first generation antihistamine (H1 receptor antagonist), dopamine antagonist and has anticholinergic properties. -https://en.wikipedia.org/wiki/Meclizine

How well did you know this?
1
Not at all
2
3
4
5
Perfectly
24
Q

All of the following are true with regards to saliva
except … ?

A) High in sodium and low in potassium B) Parotid
secretions are less viscous than submandibular
gland secretions C) Contains IgA D) ~ 1 liter is
secreted per day E) Most abundant protein is
alpha-amylase

A

A; Saliva is low in sodium and high in potassium. The other answer choices are all true. Saliva is made in the acinus and modified in the salivary duct. It also contains lysozymes, leukotaxins and opsonins.
-See KJ Lee 10th ed pg 491-492

How well did you know this?
1
Not at all
2
3
4
5
Perfectly
25
Q

What is the first line antibiotic of choice for acute otitis media?

A) Cefpodoxime 10 mg/kg/d B) Augmentin 90
mg/kg/d C) Cefuroxime 30 mg/kg/d D) Amoxicillin
90 mg/kg/d E) Ceftriaxone 50 mg/kg/d

A

D; Amoxicillin is the recommended 1st line antibiotic for AOM, the rest are all second line. Amoxicillin should be given for 10 days in children under 5 and 5-7 days in children older than 6. Use a second line antibiotic if there is no clinical improvement within 3 days. Note
that all of the choices are oral antibiotics except for Ceftriaxone which is given as a single IM injection.-See KJ Lee 10th ed pg 311-312

How well did you know this?
1
Not at all
2
3
4
5
Perfectly
26
Q

What is the incidence of pneumothorax after tracheostomy?

A) less than 0.1% B) 1.1% C) 4.3% D) 8.2%

A

C; Pneumothorax is an uncommon intraoperative complication of tracheostomy, but can be caused by direct injury to the pleura or rupture of an alveolar bleb. The incidence has been reported to be as high as 4.3% but rarely is intervention required. Clinically the rate
is much lower so there is no need to obtain routine screening chest x rays postop.
-See Cummings 6th ed pg 100-101

How well did you know this?
1
Not at all
2
3
4
5
Perfectly
27
Q

The anterior ethmoid artery enters the nose — mm
posterior to the orbital rim and the posterior
ethmoid artery enters the nose —mm posterior to
the anterior ethmoid artery.

A) 24mm; 12mm B) 12mm; 24mm C) 12mm; 6mm
D) 6mm; 12mm E) 12mm; 12mm

A

A; Use the 24-12-6 rule. The anterior ethmoid artery is 24mm posterior to the orbital rim/lacrimal crest, the posterior ethmoid is 12 mm posterior to the anterior ethmoid artery and the optic nerve is 6 mm posterior to the posterior ethmoid artery. These relationships are crucial when approaching the orbital floor or ligating these vessels to
control epistaxis. -http://emedicine.medscape.com/article/835021-
overview

How well did you know this?
1
Not at all
2
3
4
5
Perfectly
28
Q

A 3 year old male is brought to your clinic with
findings of new onset bilateral hearing loss. An
audiogram demonstrates bilateral moderate to
severe sensorineural hearing loss. On history the
mother states she had some type of infection while
she was pregnant but can’t remember what it was
called. A full workup including imaging and genetic
testing are unrevealing. You conclude that the
patient suffered the most common infection
associated with congenital hearing loss. Which of
the following is true about this disease?

A) Over 90% of patients are asymptomatic at birth
B) Is potentially treatable with penicillin C) Patients
are also likely to suffer from cataracts D) Exposure
to cats is a risk factor for this disease

A

A; Cytomegalovirus is the most common infection to cause
congenital hearing loss and the vast majority of patients are asymptomatic at birth. Hearing loss is often delayed and may not be diagnosed for several years. Syphilis is the only infectious cause of hearing loss that is potentially treatable with antibiotics. Patients with congenital rubella can suffer from cataracts in addition to hearing loss. Exposure to cats is a risk factor for toxoplasmosis. -See KJ
Lee 10th ed pg 815-816

How well did you know this?
1
Not at all
2
3
4
5
Perfectly
29
Q

A 63 y/o M undergoes a partial glossectomy with
radial forearm free flap reconstruction of the defect.
Due to concerns for airway obstruction a
tracheostomy is placed at the time of the
procedure. Post-op the patient becomes lost to
follow up for several months. Six months after the
procedure the patient presents to your clinic asking
for his tracheostomy to be removed. How likely is it
that he will have a persistent tracheocutaneous
fistula after decannulation?

A) 5% B) 20% C) 50% D) 70% E) 90%

A

D; Patients who have a tracheostomy tube for over 4 months have a 70% chance of developing a tracheocutaneous fistula. This is due to epithelialization of the tracheostomy tract. Bjork flaps and prior radiation exposure are also risk factor for tracheocutaneous fistula. - See Cummings 6th ed pg 101.

How well did you know this?
1
Not at all
2
3
4
5
Perfectly
30
Q

What is the superior border of the conus elasticus?

A) Superior border of cricoid cartilage B) Inferior
border of cricoid cartilage C) Vocal ligament D)
Vallecula E) Tip of epiglottis

A

C; The conus elasticus (aka cricovocal membrane or triangular membrane) extends from the superior border of the cricoid inferiorly up to the deep surface of the apex of the thyroid cartilage and the vocal process of the arytenoid. Its superior free edge is the vocal
ligament. -See KJ Lee 10th ed pg 530

How well did you know this?
1
Not at all
2
3
4
5
Perfectly
31
Q

The left recurrent laryngeal nerve loops around — , the right
recurrent laryngeal nerve loops around — , after which both run
in — .

A) Ligamentum arteriosum, subclavian artery, the
tracheoesophageal groove B) Aorta, subclavian
artery, the tracheoesophageal groove C)
Subclavian artery, Ligamentum arteriosum, the
tracheoesophageal groove D) Ligamentum
arteriosum, subclavian artery, posterior to the
esophagus E) Subclavian artery, ligamentum
arteriosum, posterior to the esophagus

A

A; The left RLN loops around the ligamentum arteriosum and has a relatively vertical course. The right RLN wraps around the subclavian artery and enters the neck more laterally creating a more obtusely angulated course. Both nerves ultimately run within the tracheoesophageal groove on their way to innervating the larynx. -
See KJ Lee 10th ed pg 577-579

How well did you know this?
1
Not at all
2
3
4
5
Perfectly
32
Q

Describe the changes in tissue composition that
occur during the maturation stage of wound
healing.

A) Type III collagen is replaced by type I B) Type III
collagen is replaced by type II C) Type II collagen is
replaced by type I D) Type II collagen is replaced
by type III E) Type I collagen is replaced by type III

A

A; The maturation phase of wound healing occurs between 2 weeks and 18 months and involves replacement of type III collagen with type I collagen which makes the scar softer and smaller. The wound
reaches its maximal strength (80% of its preinjury strength) during this phase. -See KJ Lee 10th ed pg 766

How well did you know this?
1
Not at all
2
3
4
5
Perfectly
33
Q

Which immunoglobulin is a dimer and is the most
common immunoglobulin in saliva?

A) IgG B) IgM C) IgA D) IgD E) IgE

A
C; IgA is present in salivary secretions and exists as a dimer.    IgG is involved in secondary immune responses and is the only Ig class that crosses the placenta.  IgM is primarily associated with early immune responses and exists as a pentamer.  IgD is found on circulating B 
cells.  Of all immunoglobulins, the body has the least amount of IgE which is involved in type I hypersensitivity reactions (anaphylaxis) as well as atopy.  -See KJ Lee 10th ed pg 452-453
How well did you know this?
1
Not at all
2
3
4
5
Perfectly
34
Q

What muscle is the sole abductor of the vocal
cords?

A) Interarytenoid B) Thyroarytenoid C) Cricothyroid
D) Lateral Cricoarytenoid E) Posterior
Cricoarytenoid

A

E; The posterior cricoarytenoid is the only abductor of the larynx. The cricothyroid is the only laryngeal muscle innervated by the external branch of the superior laryngeal nerve. The interarytenoid is the only unpaired muscle in the larynx. -See KJ Lee 10th ed pg 531-
532

How well did you know this?
1
Not at all
2
3
4
5
Perfectly
35
Q

Cleft lips are caused by the failure to fuse of what
two embryologic structures?

A) Maxillary prominence and the lateral palatine
process B) Medial nasal prominence and the
maxillary prominence C) Maxillary prominence and
the lateral palatine process D) Medial nasal
prominence and lateral nasal prominence

A

B; The lateral lip is created by fusion of the medial nasal prominence to the maxillary prominence. This failure to fuse can also create a cleft in the primary palate. Failure of the maxillary prominence to fuse with the lateral palatine process results in a cleft of the secondary palate. Risk factors for clefts include pregestational
maternal diabetes, fetal alcohol and tobacco exposure, folic acid deficiencies, retinoic acid derivatives and anticonvulsant medications.
-See KJ Lee 10th ed pg 285-287

How well did you know this?
1
Not at all
2
3
4
5
Perfectly
36
Q

A 4 y/o male is brought to the emergency room with
significant right ear pain. Otoscopy reveals a
bulging, erythematous tympanic membrane and the
diagnosis of acute otitis media is made. The
parents ask your recommendation on whether or
not they should give their son antibiotics. You
inform them that he has a —% change of
improvement within 3 days without any intervention
and a —% chance of improvement if they start
antibiotics immediately.

A) 50% ; 50% B) 50% ; 80% C) 80% ; 80% D) 80%
; 92% E) 92% ; 80%

A

D; 80% of children with AOM will show clinical improvement within 3 days with no intervention. That rate goes up to 92% if antibiotics are started immediately. However, this should be weighed against the 30% of children who will develop a rash and 80% of children who will develop diarrhea with antibiotics.-See KJ Lee 10th ed pg 311

How well did you know this?
1
Not at all
2
3
4
5
Perfectly
37
Q

A 3 y/o F presents to the ED with drooling and
stridor which developed over the past few hours.
Vital signs demonstrate an elevated respiratory rate
and fever to 103. A lateral neck x-ray
demonstrates a “thumbprint sign” and the patient is
taken emergently to the operating room where the
patient is safely intubated. Once the patient is
stable you question the parents about the child’s
vaccination history and they state that she is up to
date on all her vaccinations. Given the most
common cause of epiglottitis, what is the most
likely immunoglobulin which is deficient in this
child?

A) IgG1 B) IgG2 C) IgA1 D) IgA2 E) IgM

A

B; The most common cause of epiglottitis in children is Haemophilus influenzae, however the incidence of epiglottitis has decreased significantly since vaccination against this bacteria was introduced. While IgA is the immunoglobulin most represented in the mucosa,
failure to mount an immune response after vaccination represents a failure of the IgG antibody as this is the immunoglobulin stimulated by vaccines. The most common IgG deficiency in children is IgG2.

  • Oxelius, Aurivillius, Carlsson and Musil (1999), Serum Gm Allotype Development During Childhood. Scandinavian Journal of Immunology, 50: 440–446.
How well did you know this?
1
Not at all
2
3
4
5
Perfectly
38
Q

A 52 y/o M with PMH of poorly controlled diabetes
presents to the ED with pain and swelling of his left
cheek and neck. On history he states that he has
had some tooth pain for several months and that
within the last day he developed this pain and
swelling. On exam he has diffuse erythema of the
left face and neck w/ an “orange-peel” appearance
and subcutaneous crepitus. He has a fever of
103.2 and a LRINEC score of 7 is calculated.
Which of the following is the most likely cause of
this patient’s infection?

A) Clostridium perfringens B) Pseudomonas C)
Klebsiella D) Strep Pyogenes E) Bacteroides

A

D; This patient has classic signs and symptoms of necrotizing fasciitis. In the head and neck, dental infections are the most common cause and patients who are immunocompromised have an elevated risk. The infection should be treated aggressively with broad spectrum antibiotics, ICU level of care and surgical
exploration/debridement. Although all of the bacteria listed can cause necrotizing fasciitis, Strep pyogenes and Staph Aureus are the most common causes. The LRINEC score can be used to help make the diagnosis and consists of six blood test: CRP, WBC, Hg, Na, Cr
and glucose.

-See KJ Lee 10th ed pg 573 and -Wong, Chin-Ho, Lay- Wai Khin, Kien-Seng Heng, Kok-Chai Tan, and Cheng-Ooi Low. “The LRINEC (Laboratory Risk Indicator for Necrotizing Fasciitis) Score: A Tool for Distinguishing Necrotizing Fasciitis from Other Soft Tissue
Infections*.” Critical Care Medicine 32.7 (2004): 1535-541.

How well did you know this?
1
Not at all
2
3
4
5
Perfectly
39
Q

Which immunoglobulin can cross the placenta and
is involved in secondary immune responses?

A) IgG B) IgM C) IgA D) IgD E) IgE

A
A; IgG is involved in secondary immune responses and is the only Ig class that crosses the placenta.  IgA is present in salivary secretions and exists as a dimer.   IgM is primarily associated with early immune responses and exists as a pentamer.  IgD is found on circulating B 
cells.  Of all immunoglobulins, the body has the least amount of IgE which is involved in type I hypersensitivity reactions (anaphylaxis) as well as atopy.  

-See KJ Lee 10th ed pg 452-453

How well did you know this?
1
Not at all
2
3
4
5
Perfectly
40
Q

A 59 y/o female presents to your clinic requesting a
chemical peel. On exam she demonstrates signs
of severe photoaging. Using Baker’s phenol you
perform a deep chemical peel. To what layer does
this chemical peel penetrate?

A) Stratum Granulosum B) Stratum Basale C)
Papillary Dermis D) Reticular Dermis E)
Subcutaneous Tissue

A

D; Deep chemical peels penetrate down to the reticular dermis. This is a thick layer made of compact collagen. Care must be taken with deep chemical peels as damage to this layer results in permanent scarring. Medium depth peels reach the papillary dermis or the
superficial reticular dermis. Superficial peels remove the epidermis (of which both the stratum granulosum and stratum basale are sublayers of) and some of the superficial papillary dermis.

-See KJ Lee 10th ed pg 756

How well did you know this?
1
Not at all
2
3
4
5
Perfectly
41
Q

A 44 y/o female with diabetes presents to the
emergency room two days after undergoing
endoscopic sinus surgery with a complaint of
continuous clear nasal discharge. She states that
she taste a salty taste in the back of her throat but
otherwise feels well. Because she was placed on a
postop course of prednisone she has had difficulty
managing her blood sugar level and her serum
glucose is found to be 315. To confirm the
suspected diagnosis you collect some of the clear
rhinorrhea and test its glucose level. If your
diagnosis is correct what level of glucose would
you expect?

A) 10 mg/dL B) 80 mg/dL C) 200 mg/dl D) 300
mg/dL E) 400 mg/dL

A

C; This patient presents with symptoms and history concerning for postoperative cerebrospinal fluid(CSF) leak. While B2 transferrin is the best test to confirm a CSF leak it can take several days for these test results to return. Glucose testing is a fast way to differentiate CSF rhinorrhea from other nasal discharge. CSF fluid is expected to
have 60-70% of the serum glucose level present whereas normal nasal discharge has very low levels of glucose. Normal CSF glucose levels are 40-80 mg/dL but it would be higher in this patient given her elevated serum glucose.

-https://en.wikipedia.org/wiki/CSF_glucose

How well did you know this?
1
Not at all
2
3
4
5
Perfectly
42
Q

An 8 year old male presents to the ED with fever,
odynophagia, muffled voice and PO intolerance.
On exam his uvula is deviated to the right and he
has 3cm trismus. After appropriate treatment he is
able to tolerate PO intake and he is discharged
home on oral antibiotics. What is the chance that
he will have a recurrence of his disease process?

A) <1% B) 7% C) 22% D) 48% E) 74%

A

B; The patient presents with classic symptoms of peritonsillar abscess (PTA). Appropriate treatment involves needle aspiration and/or incision and drainage. Once the patient can tolerate PO they are usually prescribed systemic antibiotics (clindamycin) for 10 days.
16% of adults and 7% of children will have a recurrence of their PTA. If a patient has a recurrence of their PTA they should be offered a delayed tonsillectomy as they are at significantly increased risk of a
third episode.

-KJ Lee 10th ed pg 571

How well did you know this?
1
Not at all
2
3
4
5
Perfectly
43
Q

You take a 13 y/o female to the OR to treat her
recurrent respiratory papillomatosis. This is her
19th trip to the operating room. The procedure
goes well however postoperatively she has some
shortness of breath. A CXR is obtained which
demonstrates abdominal distention and a
pneumothorax. What is the the most likely cause
of this patient’s complication?

A) Use of the Co2 laser B) Use of the Argon laser
C) Use of jet ventilation D) Sudden opening of
previously chronic obstruction E) None of the above

A

C; Jet ventilation is commonly used during treatment of recurrent respiratory papillomatosis (RRP) because it allows better visualization and access to the airway. Complications include abdominal distention, subcutaneous emphysema, pneumomediastinum, pneumothorax and hypoventilation. Although
in theory a pneumothorax could be caused by iatrogenic trauma to the tracheal/laryngeal wall, this is not common and would not cause abdominal distention. Sudden opening of a previously chronic obstruction can lead to pulmonary edema but not pneumothorax.

-KJ Lee 10th ed pg 1003

How well did you know this?
1
Not at all
2
3
4
5
Perfectly
44
Q

A 42 y/o male with a BMI of 31 and a neck circumference
of 19 inches presents to your clinic with symptoms of
snoring, hypersomnolence, early morning headaches
and hypertension. You send him for a polysomnography
and correctly make the diagnosis of sleep apnea. During what stage of sleep is he least likely to suffer from apnea events?

A) Stage 1 B) Stage 2 C) Stage 3 D) Rapid Eye
Movement sleep E) Apnea is equally prevalent in all
stages

A

C; While respiratory events including apneas and hypopneas occur in all stages of sleep, they are least likely to occur during stage 3 sleep. Apneas that occur during REM tend to be associated with lower
oxygen desaturations.

-See KJ Lee 10th ed pg 417

How well did you know this?
1
Not at all
2
3
4
5
Perfectly
45
Q

What is the lateral border of the anatomic space
that spans from the glottis to the inferior border of
the cricoid cartilage?

A) Conus elasticus B) Broyles’ tendon C)
Quadrangular membrane D) Vocal ligament E)
Piriform sinus

A

A; The space that spans from the glottis to the inferior border of the cricoid is the subglottis and its lateral border is the conus elasticus (in addition to the cricoid). The conus elasticus attaches to the superior border of the cricoid inferiorly. Superior it attaches to both the deep surface of the apex of the thyroid cartilage and the vocal process of the arytenoid and forms the median cricothyroid ligament. Its superior free edge forms the vocal ligament.

-See KJ Lee 10th ed pg 530-531

How well did you know this?
1
Not at all
2
3
4
5
Perfectly
46
Q

Which structures are best visualized on a Water’s
view xray?

A) Frontal sinus B) Maxillary sinus C) Sphenoid
sinus D) Posterior ethmoid sinuses

A

B; A Waters’ view xray is taken at a 45° angle to the orbitomeatal line and best shows the maxillary sinuses, although it also can help evaluate the anterior ethmoids and orbital floor. Other plain films views of the sinuses include the Lateral view (frontal, maxillary and
sphenoid sinus), Caldwell view (Frontal sinuses and posterior ethmoid cells) and Submentovertex view (sphenoid and anterior/posterior walls of frontal sinuses).

  • Williams, John W., Leroy Roberts, Bruce Distell, and David L. Simel. “Diagnosing Sinusitis by X-ray.” J Gen Intern Med Journal of General Internal Medicine 7.5
    (1992) : 481-85.
How well did you know this?
1
Not at all
2
3
4
5
Perfectly
47
Q

Which of the following is false regarding
nasopharyngeal carcinoma?

A) HPV+ tumors have a improved 5 year survival
rate B) WHO Type III is the most common subtype
in North America C) A diet high in salted fish is a
risk factor D) 87% of patients have palpable nodal
disease at presentation E) 20% of patients are
under the age of 30

A

A; Unlike in the oropharynx where p16/HPV+ tumors are correlated with better outcomes, HPV+ nasopharyngeal carcinomas (NPC) are associated with poorer outcomes. Epstein Barr Virus (EBV) is a double stranded DNA virus found in the vast majority of patients with NPC.

-Stenmark, Matthew H., “Nonendemic HPV-Positive
Nasopharyngeal Carcinoma: Association With Poor Prognosis.” and KJ Lee 10th ed pg 716-719

How well did you know this?
1
Not at all
2
3
4
5
Perfectly
48
Q

A 39 y/o F presents to your office with complaints
of vocal fatigue which has progressively become
worse over the last few months. She states that by
the end of the day she is barely able to speak,
however her voice is strong again after a full night’s
sleep. She has also noticed some double vision
and decreased exercise tolerance. On exam she
has mild bilateral upper lid ptosis. No abnormalities
are found on flexible laryngoscopy. You order the
proper test to confirm the suspected diagnosis.
What additional testing should be performed in this
patient given her diagnosis?

A) CT chest B) CT neck w/ contrast C) CT neck w/o
contrast D) MRI brain E) Chlamydia screening

A

A; This patient presents with symptoms concerning for myasthenia gravis which is caused by antibodies blocking the postsynaptic endplates of the neuromuscular junction. Symptoms include voice fatigue, double vision, ptosis, oropharyngeal muscle weakness, head drop, limb weakness. These symptoms tend to be worse later in the day or after activity. Diagnosis can be made with a tensilon (edrophonium chloride) test. CT scan of the chest is indicated in all of myasthenia patients to rule out tumors of the thymus which are present in 10-15% of patients.

-See KJ Lee 10th ed pg 883

How well did you know this?
1
Not at all
2
3
4
5
Perfectly
49
Q

You are called to consult on a neonatal patient to
evaluate the patient for a weak cry and concerns
for aspiration. The child has never been intubated
but does have a nasogastric feeding tube in place.
On exam the child has a weak breathy cry but no
other significant findings. Vitals are stable and the
child does not appear to be in any distress. When
performing flexible laryngoscopy on this child what
is the most likely finding?

A) Vocal cord nodule B) Bilateral vocal cord
paralysis C) Right vocal cord paralysis D) Left vocal
cord paralysis

A

D; Because the course of the left recurrent laryngeal nerve is longer (passing under the ligamentum arteriosum) there is a higher chance that this patient’s vocal cord paralysis is on the left side compared to
the right given the otherwise unexplained etiology. Bilateral vocal cord paralysis is unlikely given that the patient has no evidence of respiratory distress. Vocal cord nodules are unlikely to cause a breathy voice and would not contribute significantly to aspiration.

-Lalwani 3rd ed pg 476

How well did you know this?
1
Not at all
2
3
4
5
Perfectly
50
Q

A 48 y/o female presents to your clinic requesting a
chemical peel. On exam she demonstrates signs
of mild pigment changes and actinic damage.
Using Jessner’s solution you perform a superficial
chemical peel. To what layer does this chemical
peel penetrate?

A) Stratum Corneum B) Superficial Papillary Dermis
C) Deep Papillary Dermis D) Superficial Reticular
Dermis E) Deep Reticular Dermis

A

B; Superficial chemical peels penetrate past the epidermis (including the stratum corneum) and penetrate to the superficial papillary dermis. Deep chemical peels penetrate down to the reticular dermis. Medium depth peels reach the papillary dermis or the superficial
reticular dermis.

-See KJ Lee 10th ed pg 756

How well did you know this?
1
Not at all
2
3
4
5
Perfectly
51
Q

A 5 y/o male is brought to your clinic by his mother
for evaluation of left sided ear pain. You make the
diagnosis of acute otitis media (AOM), discuss the
use of antibiotics with the parents (they decide not
to use them) and the child is sent home. On follow
up 1 week later the patient’s pain has improved and
he has no middle ear effusion. Unfortunately, 2
months later the patient returns with the same
presentation. The mother asks you if there is
anything you can do to prevent this from happening
again. How many episodes of AOM must the
patient have before you would offer an
adenoidectomy?

A) 3 or more episodes in a 6 month period B) 4 or
more episodes in a 12 month period C) 3 episodes
in a 6 month period despite tympanostomy tubes D)
4 or more episodes in a 12 month period despite
tympanostomy tubes E) Adenoidectomy is not
indicated

A

E; A 2009 Cochrane review demonstrated no benefit to
adenoidectomy in decreasing the number of episodes of AOM. Adenoidectomy should only be offered to patients with otitis media with effusion in an effort to help resolution of a persistent effusion, however this is not routine and should be performed on an individual
basis.

-Aardweg. “Adenoidectomy for Recurrent or Chronic Nasal Symptoms and Middle Ear Disease in Children up to 18 Years of Age.” Protocols Cochrane Database of Systematic Reviews (2009).

How well did you know this?
1
Not at all
2
3
4
5
Perfectly
52
Q

Which of the following is not a subsite of the
hypopharynx?

A) Postcricoid region B) Posterior pharyngeal wall
C) Pyriform sinus D) Valleculae E) All of the above
are subsites of the hypopharynx

A

D; The subsites of the hypopharynx are the “3 Ps”: postcricoid region, posterior pharyngeal wall and pyriform sinus. The valleculae is a subunit of the oropharynx. Other subunits of the oropharynx include the soft palate/uvula, base of tongue, pharyngoepiglottic/glossoepiglottic folds, palatine arch, and oropharyngeal walls.

-See KJ Lee 10th ed pg 506

How well did you know this?
1
Not at all
2
3
4
5
Perfectly
53
Q

A 49 y/o female presents to your clinic 6 months
s/p total thyroidectomy with complaints of voice
changes. On further clarification she states that
she is able to speak normally but cannot hit the
high notes while singing in her church choir the
way that she used to. Which of the following would
support the most likely diagnosis?

A) Furstenberg sign B) Gutman sign C) Brown sign
D) Hitzelberger sign E) Griesinger sign

A

B; This patient most likely has superior laryngeal nerve paralysis given that her voice changes are limited to singing (otherwise recurrent laryngeal nerve damage would be suspected). Gutman sign is when lateral pressure over the thyroid cartilage causes decreased voice pitch whereas anterior pressure causes increased
voice pitch which is indicative of superior laryngeal nerve paralysis. In a normal individuals the opposite is true.

-See KJ Lee 10th ed pg 255

How well did you know this?
1
Not at all
2
3
4
5
Perfectly
54
Q

Which of the following is false regarding glomus
tumors?

A) Glomus tumors are the most common neoplasm
affecting the middle ear B) Metastatic change
occurs in 3-4% of tumors C) Fisch type D tumors
extend to the infralabyrinthine region D)
Glasscock/Jackson Type B glomus tympanicums
completely fill the middle ear space E) There is a 5:
1 F>M ratio

A

C; Fisch Type C tumors extend to the infralabyrinthine region, Type D tumors have less than 2 cm diameter and intracranial extension.

  • See KJ Lee 10th ed pg 186
How well did you know this?
1
Not at all
2
3
4
5
Perfectly
55
Q

Which of the following are found within the foramen
lacerum?

A) Nodes of Krause B) Greater petrosal nerve C)
External carotid artery D) Middle meningeal artery
E) Labyrinthine artery

A

B; Contents of the foramen lacerum include the internal carotid artery (not external), deep petrosal nerve, greater petrosal nerve (aka superficial petrosal nerve), terminal branch of the ascending pharyngeal artery and emissary veins. Nodes of Krause are found in the posterior jugular foramen. The middle meningeal artery is found in foramen spinosum. The labyrinthine artery is found in the internal auditory canal.

  • See KJ Lee 10th ed pg 954
How well did you know this?
1
Not at all
2
3
4
5
Perfectly
56
Q

A 39 y/o male presents to your clinic with an
asymmetric audiogram. An MRI reveals a left side
2.1 cm cerebellopontine angle lesion which is
isointense on T1, slightly hyperintense on T2 and
enhances with contrast. There is a similar 1.1 cm
lesion on the right side. What is the protein most
commonly associated with this patient’s disease
process?

A) GJB2 B) P53 C) Merlin D) RET E) None of the
above

A

C; The patient has bilateral acoustic neuromas (characteristically isointense on T1, hyperintense on T2 and enhances w/ contrast). This is pathognomonic for neurofibromatosis type 2 which is caused by deletions in the NF2 gene which codes for the tumor suppressor
protein Merlin. As a side fact, Merlin is an acronym for “Moesin- Ezrin-Radixin-Like Protein”.

-See Lalwani 3rd ed pg 715

How well did you know this?
1
Not at all
2
3
4
5
Perfectly
57
Q

A 62 y/o male with a 50 pack year history of
smoking presents with voice changes and is found
to have a vocal cord mass. A biopsy is obtained in
the clinic and the report indicates squamous cell
carcinoma (SCC). However, upon review by the
tumor board, including a senior pathologist, the
patient’s mass is felt to be benign. Which of the
following is most likely to be the diagnosis for this
patient?

A) Recurrent respiratory papilloma B)
Pseudoepitheliomatous hyperplasia C)
Mucoepidermoid D) Vocal cord polyp E) Reinke’s
edema

A

B; Pseudoepitheliomatous hyperplasia is a benign condition which demonstrates overgrowth of squamous epithelium on histology and can be confused for SCC. Mucoepidermoid carcinoma can also mimic SCC however it is not a benign condition. RRP, vocal cord polyp and Reinke’s are all benign conditions but are unlikely to be
confused for SCC. Necrotizing sialometaplasia is another benign condition that can be confused for SCC, however it is extremely rare in the larynx.

-Cummings 6th ed pg 1612

How well did you know this?
1
Not at all
2
3
4
5
Perfectly
58
Q

A 41 y/o female presents to your clinic with an large
neck mass. On exam her thyroid appears enlarged
and is symmetric, firm and nontender to palpation.
On review of her prior labs her primary care
provider had ordered, she is found to be euthyroid.
What additional lab work would most likely confirm
the suspected diagnosis?

A) Thyroglobulin B) Calcitonin C) Anti-TPO
antibodies D) TSH stimulating antibodies

A

C; The patient presents with symptoms most consistent w/ Hashimoto’s thyroiditis. It is most common in females age 30-50 and most patients are euthyroid. Presentation is usually a painless, firm, enlarged goiter. FNA will reveal lymphocytic infiltration with germinal
center formation, Hurthle cell metaplasia, fibrosis and follicular acinar atrophy. 70-90% of patients will have elevated thyroid peroxidase antibodies.

-See KJ Lee 10th ed pg 586

How well did you know this?
1
Not at all
2
3
4
5
Perfectly
59
Q

While performing a coronal approach for a
forehead lift you notice that the supraorbital
foramen is not complete and is instead a
supraorbital notch with an incomplete ring of bone.
What percentage of patients will have a
supraorbital notch instead of a true foramen?

A) 6% B) 27% C) 83% D) 98%

A

C; Per Fallucco 2012, 83% of patients will have a supraorbital notch instead of a foramen. It should be noted that these notches still have a fascial band holding the contents of the notch in place in the
majority of specimens. -Fallucco “The Anatomical Morphology of the Supraorbital Notch.”

Plastic and Reconstructive Surgery 130.6
(2012): 1227-233.

How well did you know this?
1
Not at all
2
3
4
5
Perfectly
60
Q

A 33 y/o male presents with sudden onset right
sided facial weakness. On exam he has facial
symmetry at rest but an obvious asymmetry when
he smiles. He is unable to completely close his eye.
What is the patient’s House-Brackmann score?

A) 1 B) II C) III D) IV E) V F) VI

A

D; I = normal facial function II = slight weakness and/or slight synkinesis but normal tone and symmetry at rest. III = obvious asymmetry with noticeable synkinesis, contracture or hemifacial spasm. There is normal symmetry and tone at rest and complete eye
closure with effort. IV = obvious asymmetry but normal tone and symmetry at rest with incomplete eye closure V = barely perceptible motion with asymmetry at rest VI = no movement

-See KJ Lee 10th ed pg 198

How well did you know this?
1
Not at all
2
3
4
5
Perfectly
61
Q

A 53 y/o female with recurrent headaches is
referred to your clinic after her primary care doctor
ordered an MRI brain as part of her workup. You
review her scan which demonstrates a small left
sided petrous apex lesion which is hyperintense on
T1 and T2. There is no appreciable enhancement
on T1 with contrast and T2 hyperintensity does not
change with fat saturation. What is the most likely
diagnosis?

A) Cholesteatoma B) Cholesterol Granuloma C)
Mucocele D) Retained Secretions

A

B; This is likely an incidental finding and not related to her
headaches. Although cholesterol granulomas can cause symptoms
as they expand and impinge upon other structures, they are often
asymptomatic. Cholesterol granulomas have very distinct MRI
findings. They do not enhance with contrast and demonstrate
hyperintensity on both T1 and T2. -Cummings 6th ed pg 2093

How well did you know this?
1
Not at all
2
3
4
5
Perfectly
62
Q

A 61 y/o male presents to clinic with severe right
sided ear pain. He states he noticed a firm lesion
on his ear which has become increasingly painful
over the past several weeks. Usually he sleeps on
his right side but because of the pain he has
switched to his left. On exam there is a firm, round
nodule on the right helical rim ~ 4mm in diameter
which is exquisitely tender to palpation. The
patient has no hx of smoking and minimal history of
sun exposure. What is the most likely diagnosis?

A) Squamous Cell Carcinoma B) Basal Cell
Carcinoma C) Winkler Disease D) Villaret
Syndrome E) Vail Syndrome

A

C;

This is a classic presentation for Winkler disease (i.e.
Chondrodermatitis Nodularis Helicis) which is a benign condition often mistaken for a neoplastic process on exam. The key to differentiating it from neoplasm is that these lesions are very painful, whereas skin cancer usually is not. Treatment involves complete
excision of the nodule.

Villaret Syndrome is jugular foramen syndrome with the addition of Horner syndrome. Vail syndrome is unilateral, nocturnal, vidian neuralgia associated with sinusitis.

-See KJ Lee 251-252

How well did you know this?
1
Not at all
2
3
4
5
Perfectly
63
Q

A 62 y/o male patient with a history of papillary
thyroid cancer s/p total thyroidectomy and
radioactive iodine ablation 1 year ago presents for
routine followup. He states he is doing well and
denies any new symptoms. His exam is
unrevealing. You order blood work which tests the
level of a substance which may indicate recurrent
cancer. Where is this substance normally stored?

A) Pituitary Gland B) Bone Marrow C) Parafollicular
Cells D) Follicular Cells E) Colloid

A

E; Thyroglobulin is used to monitor for recurrence of well
differentiated thyroid cancer after total thyroidectomy and radioactive
iodine ablation. Levels above 10 mg/dL are concerning for recurrent
disease. Thyroglobulin is made by follicular cells but then secreted
into the follicular lumen in the form of colloid. It is important to test for
anti-thyroglobulin antibodies at the same time, as elevated levels of
these antibodies invalidate the use of thyroglobulin as a tumor
marker. - See KJ Lee 10th ed pg 581.

How well did you know this?
1
Not at all
2
3
4
5
Perfectly
64
Q

Which of the following bones does not form part of the
medial wall of the orbit?

A) Palatine B) Maxilla C) Ethmoid D) Lacrimal E)
Sphenoid

A

A; The medial wall of the orbit is made up of the lacrimal bone,
sphenoid, lamina papyracea (of the ethmoid bone) and the frontal
process of the maxilla. The palatine bone contributes to the orbit as
part of the orbital floor. The frontal bone and zygoma also contribute
to the structure of the orbit. -See KJ Lee 10th ed pg 895

How well did you know this?
1
Not at all
2
3
4
5
Perfectly
65
Q

A 55 y/o male undergoes a partial mandibulectomy
for oral squamous cell carcinoma. As part of the
reconstruction, a superiorly based
sternocleidomastoid flap is used. What is the blood
supply for this flap?

A) Posterior Auricular Artery B) Occipital Artery C)
Superior Thyroid Artery D) Suprascapular Artery E)
None of the above

A

B; The blood supply of the sternocleidomastoid is made up of the
occipital artery (upper third), branches of the superior thyroid artery
(middle third) and branches of the thyrocervical trunk which gives off
the suprascapular artery (lower third). For a superiorly based flap the
corresponding vessel is the occipital artery. -See KJ Lee 10th ed pg
731.

How well did you know this?
1
Not at all
2
3
4
5
Perfectly
66
Q

A 33 y/o male with a history of three sets of
tympanostomy tubes as a child presents with right
sided hearing loss. Audiogram reveals a significant
conductive hearing loss and you decide to order
imaging. An MRI reveals a middle ear mass that is
hypointense on T1, hyperintense on T2 and
demonstrates rim enhancement with contrast. You
take the patient to surgery and remove the mass. 6
months later you obtain a repeat MRI to look for
residual disease. What is the sensitivity of using
MRI to detect residual disease?

A) 22% B) 54% C) 73% D) 91% E) 99%

A

D; The patient presents with classic symptoms and MRI findings
consistent with a cholesteatoma (hypointense T1, hyperintense T2
with rim enhancement). Although the question used an MRI to test
your knowledge, be aware that a noncontrast CT scan is the initial
imaging modality of choice for cholesteatoma workup. Canal wall up
mastoidectomy would be an appropriate surgical intervention for this
patient depending on the the extent of their disease and patient
preference. MRI scans can be used to followup these patients and
evaluate for residual/recurrent disease and carries a sensitivity of
91% and specificity of 96%. -See Cummings 6th ed Pg 2088-2093

How well did you know this?
1
Not at all
2
3
4
5
Perfectly
67
Q

A 40 y/o female is presented at tumor board. On
review of her imaging she is found to have a 5cm
thyroid nodule with extension into the strap
muscles as well as bilateral abnormally enlarged
lymph nodes the largest being 4 cm on each
side. On fine needle aspiration papillary thyroid
carcinoma was confirmed. A chest x ray
demonstrates a 3 cm mass which is hyperintense
on a thyroid uptake scan. What is the stage of this
patient’s thyroid cancer?

A) Stage I B) Stage II C) Stage III D) Stage IV

A

B; For well differentiated thyroid cancer, patients under the age of 45
have a much better prognosis than those older than 45 and therefore
can only be stage I or II. Stage II is for patients with distant
metastasis, and Stage I is for those without. The size of their primary
lesion and the location/size of their regional metastasis are irrelevant
to their staging. -See KJ Lee 10th ed pg 640.

How well did you know this?
1
Not at all
2
3
4
5
Perfectly
68
Q

A 33 y/o male presents to your clinic with
complaints that food gets stuck in his throat with
each meal. He denies hemoptysis, voice changes
or significant weight loss. His exam, including
flexible laryngoscopy, is unrevealing. An
esophagram is ordered which demonstrates a
“bird’s beak” esophagus. What is the etiology of
this condition?
A) Degeneration of Auerbach plexus B) Repetitive
nonperistaltic esophageal contractions C) Lower
esophageal sphincter relaxation D) Weakness of
the posterior esophageal wall E) Esophageal scar
tissue

A

A; This patient presents with classic symptoms of achalasia. This
disorder is characterized by failure of the lower esophageal sphincter
to relax and lack of peristalsis which leads to the class “bird’s beak”
appearance on esophagram. The underlying etiology is idiopathic
degeneration of Auerbach plexus (aka myenteric plexus) which
provides innervation to the muscular layer of the digestive tract.
Treatment options include calcium channel blockers and botox
injections into the lower esophageal sphincter however Heller
myotomy (aka lower esophageal sphincter myotomy) is the definitive
treatment method. -See KJ Lee 10th ed pg 524

How well did you know this?
1
Not at all
2
3
4
5
Perfectly
69
Q

A 26 y/o female presents with acute onset right
sided facial paralysis. She tells you that this has
happened to her several times in the past. On
exam she has a House-Brackmann grade III
weakness of her right face with edema of the facial
soft tissues and fissuring of the tongue. What is
the most likely diagnosis?
A) Ramsay Hunt Syndrome B) Melkersson-
Rosenthal Syndrome C) Heerfordt Disease D)
Lyme Disease E) Mobius Syndrome

A

B; Melkersson-Rosenthal syndrome is associated with recurrent
facial palsy, chronic/recurrent facial edema and fissuring of the
tongue. Patients tend to be in their 20s. The etiology of this
syndrome is unknown. Ramsay Hunt is facial paralysis due to
herpes zoster and presents with vesicular eruptions on the ear.
Heerfordt disease (aka uveoparotid fever) is a rare form of
sarcoidosis which can cause bilateral facial nerve paralysis, parotid
swelling and uveitis. Lyme disease is caused by borrelia burgdorferi
and is associated with erythema migrans. 10% of lyme disease
patients will have facial nerve palsy and bilateral involvement is not
uncommon. Almost 100% of patients will have a full recovery.
Mobius syndrome is a form of congenital facial paralysis which
presents with bilateral facial paralysis and unilateral or bilateral
abducens palsy. -See KJ Lee 10th ed pg 215

How well did you know this?
1
Not at all
2
3
4
5
Perfectly
70
Q

Cranial nerve IX exits the skull via what bony
opening?
A) Posterolateral Jugular Foramen B) Anteromedial
Jugular Foramen C) Anteromedial Foramen
Lacerum D) Posterolateral Foramen Lacerum E)
Foramen Spinosum

A

B; The jugular foramen is divided into anteromedial and
posterolateral segments called the par nervosa and par vasculara
respectively. CNs IX, X, XI and the inferior petrosal sinus pass
through par nervosa whereas the internal jugular vein and posterior
meningeal artery pass through the par vasculara. -See KJ Lee 10th
ed pg 235

How well did you know this?
1
Not at all
2
3
4
5
Perfectly
71
Q

A 39 y/o female presents to your clinic with left
sided hearing loss and pulsatile tinnitus. She
denies any other complaints and states that her
symptoms were insidious in onset. She has no
significant family history. On exam a left sided
middle ear mass is visualized on the promontory.
There is a positive Brown sign. Weber lateralizes
to the left with a negative Rinne on that side as
well. You decide to get a CT scan to better
delineate the mass. Given the most likely
diagnosis, what percentage of patients will have
multicentric disease?A) 1% B) 10% C) 25 D) 50% E) 80%

A

B; This patient has the classic presentation for a glomus tympanicum. These tumors are most common in caucasian females and present with pulsatile tinnitus and conductive hearing loss. They are often found on the promontory. Brown sign is blanching of the middle ear
mass on pneumatic otoscopy which is strongly suggestive of a glomus tumor. 10% of glomus tumors are multicentric so it is
important to evaluate the patient with imaging for potential additional
masses. -See KJ Lee 10th ed pg 681-683

How well did you know this?
1
Not at all
2
3
4
5
Perfectly
72
Q

A 35 y/o male presents for evaluation of an
incidentally discovered left thyroid nodule. On
ultrasound it measures 1.4 cm in largest dimension
and appears well circumscribed. No abnormal
appearing lateral neck nodes are found. To your
surprise the fine needle aspiration comes back
showing anaplastic thyroid cancer. What is the
stage of this patient’s thyroid cancer?A) Stage I B) Stage II C) Stage III D) Stage IV

A

D; All patients with anaplastic thyroid cancer are considered stage IV
due to the extremely poor prognosis. Tumor size, regional and
distant metastasis or irrelevant to their staging. -See KJ Lee 10th ed
pg 640

How well did you know this?
1
Not at all
2
3
4
5
Perfectly
73
Q

A 61 y/o male presents to your clinic after an MRI
for neck pain revealed multiple abnormal cervical
lymph nodes and thyroid nodules. You confirm
these findings on ultrasound and offer him a fine
needle aspiration (FNA). The patient is terribly
afraid of needles and only agrees to undergo an
FNA on a single site. Which of the following should
be biopsied to provide the most information?
A) A left sided 1.9 cm spongiform thyroid nodule B)
A 1.8cm right level IV node with a kidney bean
shape and echogenic hilum C) A 1.8cm left level III
lymph node with microcalcifications, round shape
and increased echogenicity D) A right sided 2.1 cm
solid hypoechoic nodule with irregular borders E) A
left sided 1.9 cm hypoechoic nodule which is taller
than it is wide

A

C; While both D and E likely represent well differentiated thyroid
carcinoma if an FNA is performed at these sites it does not provide
information as to regional metastasis and a second FNA will be
required for the suspicious cervical nodes. If you perform an FNA on
the suspicious lymph node (larger than 1 cm, microcalcifications,
abnormal round shape and increased echogenicity) and it is positive
for thyroid cancer then you know the patient has regional spread and
you can treat him appropriately. Spongiform thyroid nodules are
categorized as “very low suspicion” in the 2015 ATA thyroid nodule
guidelines. -See 2015 ATA Thyroid Nodule Guidelines

How well did you know this?
1
Not at all
2
3
4
5
Perfectly
74
Q

A 52 y/o male with a history of poorly controlled
diabetes presents with right sided otalgia. He
states that he has been taking vicodin for the pain
but that it hasn’t helped. On exam there is some
purulent otorrhea and granulation tissue present in
the EAC. You take a culture and admit the patient
for treatment. Which imaging study is best for
tracking the response to treatment in this patient?
A) MRI of the Skull Base B) Technetium
Radioisotope Scan C) CT Temporal Bone with
Contrast D) CT Temporal Bone Noncontrast E)
Gallium Scan

A

E; This patient presents with signs and symptoms of malignant otitis
externa. A noncontrast CT temporal bone scan can be used to
make/confirm the initial diagnosis as can an MRI of the skull base or
a technetium radioisotope scan. These scans are not great at
monitoring treatment however. It should be noted that once a
technetium scan becomes positive it will remain so indefinitely.
Gallium scans are best for detecting inflammatory response and a
reduction in uptake correlates with clinical improvement therefore
making them useful to track treatment response. -See KJ Lee 10th
ed pg 207-208

How well did you know this?
1
Not at all
2
3
4
5
Perfectly
75
Q

A 39 y/o female presents to your clinic with a right
sided parotid mass which she states has been
slowly growing over the past few years. The mass
is firm but mobile and does not cause her any pain.
She has no other significant medical history and
does not smoke. A fine needle aspiration does not
reveal any malignant cells. Given the most likely
diagnosis, what are the chances of eventual
malignant transformation if the patient decides not
to pursue any treatment.A)1%> B) 10% C) 25% D) 50% E) 75% F) >90%

A

C; The most common benign salivary gland neoplasm is pleomorphic
adenoma (benign mixed tumor), which most commonly presents in
the parotid gland. The rate of transformation to carcinoma ex-
pleomorphic adenoma is ~ 25%. -See KJ Lee 10th ed pg 501

How well did you know this?
1
Not at all
2
3
4
5
Perfectly
76
Q

A 32 y/o male presents to your clinic with
complaints of right sided hearing loss and vertigo
while lifting weights. The patient states this started
6 months ago after he suffered a concussion during
a car accident. On exam he demonstrates vertical
nystagmus during pneumatic otoscopy. An
audiogram demonstrates right sided low frequency
conductive hearing loss with an elevated bone
threshold. What scan should be ordered to confirm
the diagnosis?
A) CT in the plane of Ohngren B) CT in the plane of
Stenver C) CT in the plane of Frankfurt D) MRI T1
with contrast E) MRI T2 with contrast F) MRI
Diffusion Weighted Imaging

A

B; This patient has a presentation most consistent with superior
semicircular canal dehiscence. The most appropriate imaging
modality to order is a CT scan in the planes of Poschel and Stenver
as they transect the superior canal and allow the dehiscence to be
seen. The line of Ohngren connects the medial canthus of the eye to
the angle of the mandible and helps with prognostication of midface
tumors. The Frankfurt horizontal plane connects the inferior orbital
rim and the superior ear canal and helps define standard anatomic
positioning. MRI is not an appropriate scan to detect superior canal
dehiscence. -See KJ Lee 10th ed pg 355

How well did you know this?
1
Not at all
2
3
4
5
Perfectly
77
Q

Which of the following is the statistical definition of
“Power”?
A) Probability that the null hypothesis will be
rejected if it is indeed false B) Results observed in a
study, experiment, or test that are no different from
what might have occurred because of chance alone
C) Probability of making a type 1 error D) A zone of
compatibility within the data, which indicates a
range of values considered plausible for the
population from which the study sample was
selected

A

A; Factors that affect a study’s power include the statistical
significance criteria, the magnitude of the effect in the population and
the sample size used. B refers to the null hypothesis, C refers to the
P value, and D refers to the confidence interval. -See Cummings 6th
ed pg 16

How well did you know this?
1
Not at all
2
3
4
5
Perfectly
78
Q

An 11 month old male is brought to your clinic for
evaluation of his hearing. The mother states that
she is concerned because he doesn’t startle when
he is exposed to loud noises. The patient’s
delivery was without complications and he is
otherwise meeting his developmental milestones.
There is no family history of hearing loss and your
physical exam is unremarkable. What type of
hearing test is most appropriate for this patient?
A) Standard Audiometry B) Play Audiometry C)
Behavior Observation Audiometry D) Visual
Response Audiometry

A
D;  For children less than 6 months, behavior observation audiometry 
is preferred (in addition to ABR and OAE).  For children 6 months to 
3 years (as in our patient) visual response audiometry is preferred.  
Play audiometry can be used in children 3-6 years and conventional 
audiometry can usually be achieved in children over the age of 6.  -
See KJ Lee 10th ed pg 51-53
How well did you know this?
1
Not at all
2
3
4
5
Perfectly
79
Q

After drilling out a facial recess and identifying the
round window niche you perform a cochleostomy
and prepare to insert a cochlear implant electrode
array. What space will the electrode array enter?
A) Saccule B) Scala Media C) Scala Vestibuli D)
Scala Tympani

A

D; Cochlear implants are inserted into the scala tympani which can
be accessed via the round window or adjacent cochleostomy. The
scala vestibuli can be accessed via the vestibule through the oval
window (although this will likely cause a sensorineural hearing
loss). The scala media cannot be accessed surgically without
causing significant damage to the inner ear and is located between
the scala vestibuli and scala tympani. The saccule detects linear
acceleration and is not involved in hearing. -See KJ Lee 10th ed pg
157-158.

How well did you know this?
1
Not at all
2
3
4
5
Perfectly
80
Q

Which of the following bones of the skull is an
endochondral bone, not a membranous bone?
A) Frontal B) Ethmoid C) Zygoma D) Vomer E)
Lacrimal

A

B; Endochondral bones of the skull include the petrous, occipital,
ethmoid, mastoid and sphenoid bones. The remaining bones are all
membranous. During the ossification of endochondral bone cartilage
is present as an intermediary whereas this is not the case in
membranous bone. -See KJ Lee 10th ed pg 955.

How well did you know this?
1
Not at all
2
3
4
5
Perfectly
81
Q

A 48 y/o female presents with right sided pulsatile
tinnitus and an audiogram demonstrating a right
sided conductive hearing loss. On exam a reddish
mass is seen behind the TM which blanches during
pneumatic otoscopy. The rest of the exam is
unremarkable. After obtaining a CT scan to
confirm your diagnosis you take the patient to the
operating room for treatment. Which nerve gives
rise to this disease process? Which cranial nerve
is it a branch from?
A) Arnold’s Nerve; CN X B) Arnold’s Nerve; CN IX
C) Jacobson’s nerve; CN IX D) Jacobson’s nerve;
CN X E) Auriculotemporal nerve; CN V3

A

C; The patient presents with findings consistent with a glomus
tympanicum. These tumors arise from Jacobson’s nerve (aka
Tympanic nerve) which is branch of the glossopharyngeal nerve (CN
IX). Arnold’s nerve is also known as the auricular branch of the
vagus nerve (CN X) and it provides sensation to the ear canal, tragus
and auricle. The auriculotemporal nerve is a branch of the
mandibular branch of the trigeminal nerve (CN V3) and provides
sensation to the ear canal, tympanic membrane and auricle. It also
carries parasympathetic fibers to the parotid gland. -See KJ Lee 10th
ed pg 4

How well did you know this?
1
Not at all
2
3
4
5
Perfectly
82
Q

A 29 y/o male presents to clinic with voice changes.
He states that for the past 6 months he has
increased difficulty speaking which comes and
goes but is present enough to be disruptive to his
work performance. He does not have difficulty with
whispering or singing but notes frequent voice
breaks while speaking. On exam his voice sounds
strained but not breathy and he has particular
difficulty with words beginning in vowels. Flexible
laryngoscopy was largely unrevealing. Given the
most likely diagnosis what is the most effective
treatment modality?
A) Berke procedure B) Recurrent laryngeal nerve
transection C) Speech therapy D) Botox injection of
the thyroarytenoid E) Botox injection of the
posterior cricoarytenoid

A

D; This patient’s presentation is consistent with adductor spasmodic
dysphonia. Classic symptoms include strained voice with frequent
voice breaks and difficulty with words beginning in vowels. The
NIDCD established four diagnostic criteria for spasmodic dysphonia
including: 1) Increased effort with speaking 2) difficulty fluctuates
over time 3) Symptoms last more than 3 months 4) At least one
nonspeaking task is normal (laugh, whisper, sing, yawn, shout, cry)
Botox injections of the thyroarytenoid muscle is currently the most
effective management option. The Berke procedure involves
transection of the adductor branches of the recurrent nerve with
reinnervation using ansa cervicalis. Recurrent nerve transection was
previously used however this often leads to significant breathiness
and symptoms will often return within 3 years. Speech therapy is not
effective in patients with spasmodic dysphonia. Botox injections of
the posterior cricoarytenoid is used in patients with abductor
spasmodic dysphonia which is much less common and presents with
breathy voice and voice breaks during plosive sounds. -See KJ Lee 10th ed pg 543-544

How well did you know this?
1
Not at all
2
3
4
5
Perfectly
83
Q

A 62 y/o male with advanced glottis squamous cell
carcinoma presents to clinic for evaluation. It is
decided that the patient requires a total
laryngectomy with free flap coverage of the wound
bed. The patient has been losing weight for
several months due to dysphagia and aspiration
however 10 days ago he received a gtube and has
been able to receive adequate nutrition during this
time. Which of the following lab values is most
likely to be abnormal in this patient?
A) Prealbumin B) Albumin C) Transferrin D) Serum
Glucose E) None of the above are likely to be
abnormal

A

B; Albumin is a good marker for patients’ long term nutritional status
and has a half life of 20 days. Values less than 3.0 g/dL are
associated with increased morbidity. Prealbumin and transferrin are
both markers for a patient’s nutritional status but they have short half
lifes (2 and 10 days respectively) and are therefore more likely to be
normal in this patient who has been receiving adequate nutrition for
the past 10 days. Glucose is not a commonly used marker to
evaluate for malnutrition and would be expected to be normal in this
patient who is currently receiving adequate nutrition. -See KJ Lee
10th ed pg 921 - 922

How well did you know this?
1
Not at all
2
3
4
5
Perfectly
84
Q

A 42 y/o male presents with an incidental finding of a left
sided neck mass found on CT neck obtained during a
trauma workup two weeks ago. On the CT scan a “Lyre
sign” is appreciated. An MRI is then obtained to further
delineate this mass. What characteristic finding is
expected on the patient’s MRI scan?
A) Microcalcifications B) No enhancement with
contrast C) Thumbprint sign D) “Salt and Pepper”
on T1 E) Cystic component

A

D; The “Lyre sign” refers to the splaying of the internal and external
carotid artery that is caused by a carotid body tumor. This splaying
resembles the classic Greek instrument of the Lyre. These tumors
are paragangliomas and as such also demonstrate the classic “salt
and pepper” appearance caused by areas of slow flow (salt) and flow
voids (pepper). -Baser, Husniye, Baris Ayhan, Meryem Ilkay Eren
Karanis, Salih Baser, Deniz Karasoy, Kemal Kalkan, and Samil Ecirli.
“A Carotid Body Tumor Mimicking a Thyroid Nodule: A Case Report.”
Endocrine Abstracts EJEA (2014).

How well did you know this?
1
Not at all
2
3
4
5
Perfectly
85
Q

Which of the following nerves is not involved in the
parasympathetic innervation of the parotid gland?
A) Trigeminal Nerve B) Less Superficial Petrosal
Nerve C) Nervus Intermedius D) Jacobson’s Nerve

A

C; The pathway for parasympathetic innervation of the parotid gland
is as follows: Inferior salivary nucleus –> Jacobson’s nerve (CN IX)
–> Lesser superficial petrosal nerve –> Auriculotemporal nerve (V3)
–> Parotid gland. The nervus intermedius carries parasympathetic
fibers from the Superior salivary nucleus to the chorda tympani which
eventually innervates the submandibular and sublingual glands, but
not the parotid gland. -See KJ Lee 10th ed pg 488-489

How well did you know this?
1
Not at all
2
3
4
5
Perfectly
86
Q

A 53 y/o male presents to your clinic with
complaints of heartburn. He states his symptoms
are worse at night after having a big meal and is
associated with significant belching. You refer him
for endoscopic esophagoscopy which reveals 2
mucosal breaks which bridge the tops of the
esophageal folds involving 50% of the
circumference of the mucosal lining. How would
this finding be classified?
A) Lahey Stage 2 B) Los Angeles Grade C C)
Brombart Stage 3 D) Morton Stage 1 E) Van
Overbeek Stage 3

A

B; Erosive esophagitis is most commonly classified using the Los
Angeles classification scheme which is divided into 4 categories:
Grade A: >1 isolated mucosal breaks <5 mm long Grade B: > 1
isolated mucosal breaks >5 mm long Grade C: > 1 mucosal breaks
bridging the tops of folds but involving <75% of the circumference
Grade D: > 1 mucosal breaks bridging the tops of folds and involving
>75% of the circumference. The remaining answer choices are all
various classification schemes used to categorize Zenker’s
diverticulum. -See Cummings 6th ed pg 1008

How well did you know this?
1
Not at all
2
3
4
5
Perfectly
87
Q

Using your first paycheck as an attending
physician, you buy a porsche and speed out of the
dealership. As you accelerate to get up to speed
on the freeway, which vestibular organ detects the
acceleration?
A) Utricle B) Saccule C) Superior Semicircular
Canal D) Lateral Semicircular Canal E) Posterior
Semicircular Canal

A

A; The utricle detects linear acceleration (accelerating in a car)
whereas the saccule detects vertical acceleration (descending in an
elevator). The semicircular canals detect rotational acceleration. -
See KJ Lee 10th ed pg 83

How well did you know this?
1
Not at all
2
3
4
5
Perfectly
88
Q

An 18 y/o male presents to the trauma bay after
sustaining significant blunt trauma to the head. On
exam he is noted to have complete left sided facial
nerve paralysis. Which type of temporal bone
fracture is the most common cause of facial nerve
injuries? What is the most common cause of
posttraumatic vertigo?
A) Longitudinal; Labyrinthine Concussion B)
Longitudinal; BPPV C) Transverse; Labyrinthine
Concussion D) Transverse; BPPV E) Transverse;
Endolymphatic Hydrops

A

A; While transverse temporal bone fractures are more likely to cause
a facial nerve injury, there are far more longitudinal fractures and
therefore they are the most common cause of facial nerve injuries.
Labyrinthine concussion is a more common cause of posttraumatic
vertigo than BPPV. -See KJ Lee 10th ed pg 264-265

How well did you know this?
1
Not at all
2
3
4
5
Perfectly
89
Q

What is the location of the sphenopalatine
foramen?
A) Anterior to the crista ethmoidalis B) Posterior to
the crista ethmoidalis C) Posterior to the superior
attachment of the middle turbinate D) Anterior to
the inferior attachment of the middle turbinate E)
Anterior to the inferior attachment of the middle
turbinate

A

B; The sphenopalatine foramen can be located directly behind the
crista ethmoidalis in the majority of individuals. This is useful when
performing a sphenopalatine artery ligation to control epistaxis. It
should also be kept in mind that the artery branches medial to the
crista in almost all patients, with some patients having up to 10
individual branches. -See KJ Lee 10th ed pg 407 and Bolger “The
Role of the Crista Ethmoidalis in Endoscopic Sphenopalatine Artery
Ligation” American Journal of Rhinology

How well did you know this?
1
Not at all
2
3
4
5
Perfectly
90
Q

During a neck dissection a large artery is cut
accidentally and immediate ligation is required.
After the field is cleared of blood you realize that
the artery that was ligated was the fourth artery to
branch off the external carotid. Which of the
following is a branch of that artery?
A) Superior labial artery B) Superior laryngeal
artery C) Sublingual artery D) Middle temporal
artery

A

A; The fourth branch off the external carotid is the facial artery. Use
the mnemonic “Some Attendings Like Freaking Out Potential Med
Students” (Superior thyroid, ascending pharyngeal, lingual, facial,
occipital, posterior auricular, internal maxillary, superficial temporal).
The superior labial artery is a branch off the facial. The superior
laryngeal is a branch from superior thyroid, the sublingual is a branch
off of lingual and the middle temporal is a branch off of superficial
temporal. -See KJ Lee 10th ed 958-959.

How well did you know this?
1
Not at all
2
3
4
5
Perfectly
91
Q

Which of the sinus cavities is present at birth?
A) Maxillary and Ethmoids B) Maxillary and
Sphenoids C) Sphenoids and Frontals D) Ethmoids
and Frontals E) Sphenoids and Ethmoids
A 1 year old child is brought to your clinic for

A

A; The maxillary sinus develops out of a furrow in the lateral nasal
wall at ~ the 65th day of development and the ethmoid sinuses
develop in the second trimester. These are the only two sinuses you
are born with. The sphenoid sinuses are not present until ~ 2 years
of age and the frontals do not appear until the age of 7. -See KJ Lee
10th ed pg 783-784

How well did you know this?
1
Not at all
2
3
4
5
Perfectly
92
Q

A 1 year old child is brought to your clinic for
hearing loss. On review of his medical record the
patient was recently diagnosed with Fanconi’s
Anemia. The parents are considering having a
second child. What is the chance that a second
child will also have Fanconi’s Anemia?
A) 1:130,000 B) 25% C) 50% D) 100% if the child is
a male E) 100% regardless of gender

A

B; Fanconi’s Anemia is a rare congenital disorder characterized by
spontaneous bleeding, decreased platelets, bone marrow
megakaryocytes and anomalies of the inner, middle and external ear.
Approximately 1:130,000 children are born with this disorder. It is
autosomal recessive and therefore the chances that a second child
has this disorder is 25%. -See KJ Lee 10th ed pg 229-230.

How well did you know this?
1
Not at all
2
3
4
5
Perfectly
93
Q

A 37 y/o male presents with swelling of both
pinnas. He states that his ears are tender and
become inflamed for 1-2 weeks every so often. He
denies any otorrhea, tinnitus or vertigo but has a
small amount of otorrhea and muffled hearing. On
exam he is breathing comfortably and in no acute
distress but has impressive erythema and edema
of both auricles with severe tenderness to
palpation. You draw labs which reveal a normal
white blood cell count but an erythrocyte
sedimentation rate of 64. Given the most likely
diagnosis, what are the chances that this patient’s
disease will eventually progress to cause airway
symptoms?A) <1% B) 20% C) 50% D) 80% E) >95%

A

C;

This patient’s presentation is most consistent with relapsing polychondritis. This disorder is characterized by recurring inflammation of cartilaginous structures including the auricles, septum and airway. Symptoms develop rapidly and resolve after 1-2 weeks. Inflammatory markers such as ESR and CRP are often significantly elevated and treatment consists of oral steroids and immune modulating agents. ~ 50% of patients will develop progressive destruction of the airway cartilages which result in airway symptoms.

-See KJ Lee 10th ed pg 549

How well did you know this?
1
Not at all
2
3
4
5
Perfectly
94
Q

A 62 y/o female presents to the clinic with edema,
pruritis and clear discharge from her right ear. She
is prescribed some ear drops and sent home,
however, three days later she returns saying the
ear drops only made things worse. On exam she
has increased erythema and edema of the canal
with extension to the conchal bowl and ear lobe but
sparing the portion of the pinna superior to the
external ear canal. What ototopical medication was
she most likely prescribed?
A) Polysporin B) Acetic Acid C) Ofloxacin D)
Cortisporin E) Ciprodex

A

D; Cortisporin contains hydrocortisone, polymyxin B and neomycin
which is a common cause of contact dermatitis. The other ototopical
medications listed can cause allergic reactions but are less likely to
do so. Ciprodex contains ciprofloxacin and dexamethasone.
Polysporin contains polymyxin and lidocaine. -Epstein E. Allergy to Dermatologic Agents. JAMA. 1966;198(5):517-520.

How well did you know this?
1
Not at all
2
3
4
5
Perfectly
95
Q

A 62 y/o male presents with a left sided neck mass.
Imaging reveals a parapharyngeal space mass in
the prestyloid space. The patient is very hesitant to
undergo surgery and wants to know what would
happen if he elects observation. Which of the
following prestyloid structures is at risk if the mass
continues to grow?
A) Spinal accessory nerve B) Vagus nerve C)
Internal jugular vein D) Cervical sympathetic chain
E) Lingual nerve

A

E; All of the structures listed are poststyloid structures except for the
lingual nerve. In general, poststyloid masses are more concerning
given the importance of the structures in that region. Prestyloid:
Internal maxillary artery, inferior alveolar nerve, lingual nerve,
auriculotemporal nerve Poststyloid: Carotid artery, internal jugular
vein, CN IX, X, XI, XII, cervical sympathetic chain. -See KJ Lee 10th
ed pg 491 and 963.

How well did you know this?
1
Not at all
2
3
4
5
Perfectly
96
Q

A 73 y/o male with a history of diabetes and
hypertension presents to the emergency
department with severe left sided ear pain. On
exam he has moderate edema and erythema of the
EAC skin and a small amount of granulation tissue
present. You decide to order a nuclear imaging
study given the patient’s history and degree of pain.
If the scan is positive what is the most likely
organism responsible for this patient’s infection?
A) Pseudomonas B) Staph Aureus C) Staph
Epidermidis D) Aspergillus E) Candida

A

A; This patient demonstrates a classic presentation of malignant otitis
externa (MOE). They have a history of diabetes, pain out of
proportion to exam and granulation tissue (which is usually present at
the bony/cartilaginous junction). A technetium scan is the most
appropriate scan for initial diagnosis although CT temporal bones are
sometime ordered as well. Gallium scans and ESR can be used to
monitor response to treatment. The most common organism to
cause MOE is Pseudomonas (98% of cases) and the patient should
be treated with ciprofloxacin with a 3rd/4th generation cephalosporin
for at least 6 weeks. -See KJ Lee 10th ed pg 207-208

How well did you know this?
1
Not at all
2
3
4
5
Perfectly
97
Q

A 36 y/o male is referred for evaluation of his
sinuses. The patient endorses constant
headaches for the past 2-3 months as well as some
vision changes but denies nasal discharge or
obstruction. On exam his nasal passages are clear
but you notice a left sided abducens nerve palsy.
Given his abnormal exam you order a CT scan
which reveals a midline clival lesion which is well
circumscribed with some abnormal internal
calcifications, enhancement with contrast and a
positive “thumb sign”. Given the most likely
diagnosis, what is likely to be found on histology
upon resection of the lesion?
A) Antoni A Pattern B) Antoni B Pattern C)
Physaliferous Cells D) Psammoma Bodies E)
Orphan Annie Eye Nuclear Inclusions F) Zellballen

A

C; This patient presents with symptoms and imaging consistent with
a chordoma. These tumors, which develop from notochord
remnants, often present in the midline clivus and are locally
aggressive with involvement of Dorello’s canal and CN VI being
common. Surgical resection is the standard treatment but 5 year
survival is only ~ 50%. On imaging, projection of the mass
posteriorly such that it indents the pons is called a “thumb sign” and
is characteristic of these lesions. On histology cells with small round
nuclei and abundant vacuolated cytoplasm (physaliferous cells) are
seen. Antoni A and B patterns are present in vestibular
schwannomas. Psammoma bodies and orphan Annie eye nuclear
inclusions are seen in papillary thyroid carcinoma. Zellballen are
characteristic of paragangliomas. -See KJ Lee 10th ed pg 685-686

How well did you know this?
1
Not at all
2
3
4
5
Perfectly
98
Q

A 24 y/o female presents for workup of a thyroid
nodule. She denies any hyper or hypothyroid
symptoms and has no history of radiation
exposure, however multiple members of her family
have undergone thyroidectomies for both
cancerous and noncancerous lesions. On exam the
patient has multiple small masses of her skin and
oral mucosa but she states they have been
biopsied and she was told they are benign. What
is the most likely diagnosis?
A) Cogan syndrome B) Von Recklinghausen
disease C) Cowden disease D) Pendred syndrome
E) Rosai-Dorfman disease

A

C; This patient presents with a family history of goiter, multiple skin
masses (hamartomas) and concern for thyroid cancer which is
consistent with Cowden syndrome. Cowden syndrome (aka Multiple
Hamartoma Syndrome) is an autosomal dominant disorder caused
by mutations of phosphatase and tensin homolog (PTEN) gene. It is
important to make this diagnosis as these patients are at increased
risk of multiple malignancies including thyroid, colon, breast and renal
cancer. 89% of these patients will develop some form of cancer
during their lifetime. Cogan Syndrome = interstitial keratitis and
Meniere’s like symptoms likely related to periarteritis nodosa. Von
Recklinghausen disease = NF1, Multiple neurofibromas and cafe au
lait spots Pendred Syndrome = Congenital sensorineural hearing
loss and euthyroid goiter Rosai-Dorfman = Benign
lymphadenopathy which is often self limiting -See KJ Lee 10th ed pg
227 and 1056.

How well did you know this?
1
Not at all
2
3
4
5
Perfectly
99
Q

A 33 y/o female presents to your clinic for
evaluation of a nasopharyngeal mass. The patient
states she had an MRI performed for neck pain and
an abnormal mass was found in the back of her
nose. She occasionally experiences postnasal drip
but otherwise denies nasal obstruction, nasal
discharge, loss of sense of smell, fevers, voice
changes or weight loss. She has no significant
past medical history, social history or family history.
On review of the MRI there is a well circumscribed
1 cm lesion in the midline nasopharynx which is
bright on T2 and does not enhance with contrast.
On flexible nasal endoscopy there is a smooth
submucosal mass in the nasopharynx. What is the
most likely diagnosis?
A) Epstein Nodule B) Pyogenic Granuloma C)
Fibrous Dysplasia D) Thornwaldt’s Cyst E)
Osteoma

A

D; The most likely diagnosis is a Thornwaldt’s Cyst. These are
benign masses in the midline nasopharynx that represent a remnant
of the notochord. They can sometimes cause symptoms such as
neck pain, eustachian tube dysfunction and postnasal drip however
they are commonly asymptomatic and noticed incidentally on
imaging. If they are symptomatic they can be marsupialized,
otherwise no intervention is indicated. -See KJ Lee 10th ed pg 404

How well did you know this?
1
Not at all
2
3
4
5
Perfectly
100
Q

A 49 y/o female presents to the clinic with right
sided nasal congestion and occasional epistaxis.
On exam a necrotic appearing lesion of her inferior
turbinate is identified. A biopsy is obtained which
returns demonstrating mucosal melanoma with a
depth of 2mm and no ulcerations. Imaging is
obtained which does not reveal any bony erosion
or evidence of nodal/distant mets. What is the
TNM classification and staging for this patient?
A) T2aN0M0 - Stage Ia B) T2bN0M0 - Stage Ib C)
T2aN0M0 - Stage IIa D) T3N0M0 - Stage III E)
T3N0M0 - Stage IVa

A

D; Mucosal melanoma has a much worse prognosis than cutaneous
melanoma and is therefore staged differently. T stage begins at T3 if
it is limited to mucosa and does not involve any underlying structures.
Unlike cutaneous melanoma, depth does not matter (and often is not
reported). Lesions that involve cartilage or bone are considered T4a
and those that are locally advanced involving skull base, brain, dura,
cranial nerves, etc. are T4b. See staging below: T3N0 = Stage III
T4aNx = Stage IVa T4bNx = StageIVb M1 = Stage IVc If this was a
cutaneous melanoma the correct staging would be T2a (1-2mm
thickness w/o ulceration)N0M0 - Stage Ib. - See KJ Lee 10th ed pg
650.

How well did you know this?
1
Not at all
2
3
4
5
Perfectly
101
Q

A 44 y/o female with a past medical history of
diabetes presents with left ear pruritus, mild pain
and foul smelling discharge. She states that her
symptoms started three weeks ago. Initially she
had more pain and less itching, but this improved
after a course of ear drops from her primary care
provider. Once finishing the ear drops her current
symptoms began. On exam there is white fluffy
material in the external canal and weber lateralizes
to the left. What percentage of otitis externa is
caused by the class of organisms responsible for
this patient’s infection?A) 2% B) 24% C) 57% D) 73% E) 98%

A

A; This patient’s presentation is most consistent with fungal otitis
externa. This can commonly occur after a course of topical
antibiotics and classically has more pruritus than bacterial otitis
externa. Fungal elements are often visualized in the external canal
on exam. 2% (or less) of acute otitis externa is caused by fungus. -
See KJ Lee 10th ed pg 935.

How well did you know this?
1
Not at all
2
3
4
5
Perfectly
102
Q

A 44 y/o female with a history of Grave’s disease
refractory to medical management is taken to the
operating room for a thyroidectomy. The surgery is
completed without complication, however in the
postoperative recovery unit the patient develops
tachycardia to 130s and hyperthermia to 103.
Which of the following medications should not be
given immediately?
A) Propylthiouracil B) Methimazole C) Propranolol
D) Iodine E) Corticosteroids

A

D;

Tachycardia and hyperthermia in a patient with poorly controlled graves and an acute insult such as thyroid surgery is concerning for thyroid storm. Treatment of thyroid storm involves all of the listed medications however iodine should not be given until 1 hour after propylthiouracil and methimazole. Propylthiouracil and propranolol blocks T4 conversion to T3. Methimazole prevents new thyroid hormone synthesis. Corticosteroids are used as prophylaxis against adrenal insufficiency.

-See Bahn “Management Guidelines of the
ATA and AACE” pg 607.

How well did you know this?
1
Not at all
2
3
4
5
Perfectly
103
Q

A 28 y/o male presents to the trauma bay with a
facial laceration. You offer him a repair and explain
that you will use some local anesthetic agents to
numb the area. He states that several years ago
he went to the dentist and developed a rash after
they injected local anesthesia. Which of the
following agents should you use to minimize the
potential for an allergic reaction?
A) Lidocaine B) Cocaine C) Novocaine D)
Benzocaine E) They are all equally as likely to
cause an allergic reaction

A

A; Local anesthetic agents can be divided into esters and amides.
Esters more commonly cause allergies and it is recommended that if
a patient has a history of allergic reactions to one class of anesthesia
that the opposite class be used. Lidocaine is the only amide
anesthetic agent listed. Given the fact that previously this patient
most likely had a reaction to an ester medication, an amide such as
Lidocaine is the best answer choice. Remember that all amide
anesthetic agents have two “i”s in their names. -See KJ Lee 10th ed
pg 830-831

How well did you know this?
1
Not at all
2
3
4
5
Perfectly
104
Q

The recurrent laryngeal nerve enters the larynx
directly underneath which muscle?
A) Cricothyroid B) Posterior Cricoarytenoid C)
Inferior Constrictor D) Superior Constrictor E)
Cricopharyngeus

A

C; The recurrent laryngeal nerve (RLN) travels in the
tracheoesophageal groove as it moves superiorly entering the larynx
just posterior to the cricothyroid joint. Prior to entering the larynx it
dives underneath the inferior constrictor muscle. The RLN innervates
the inferior constrictor muscle and all the intrinsic muscles of the
larynx except the cricothyroid muscle. -See KJ Lee 10th ed Pg 577-
578

How well did you know this?
1
Not at all
2
3
4
5
Perfectly
105
Q

A 49 y/o female presents to the clinic with left sided
sudden sensorineural hearing loss. She states she
suddenly lost her hearing approximately 3 weeks
ago and it has not recovered since then. At first
she did experience some moderate vertigo but that
has subsided. She denies any head trauma or
family history of hearing loss. An audiogram is
obtained which demonstrates a unilateral profound
flat sensorineural hearing loss. Which of the
following is not a poor prognostic indicator for this
patient?
A) Severity of the loss B) Shape of the audiogram
C) History of vertigo D) Gender E) Age

A

D; There are four main factors that affect the prognosis of untreated
sudden sensorineural hearing loss. 1. Severity of the hearing loss
(the worse the loss the worse the prognosis) 2. Audiogram shape
(flat and down sloping audiograms are worse than mid frequency or
up sloping audiograms) 3. Presence of vertigo 4. Age of the patient
(children and patients over 40 have a worse prognosis) Additionally,
the longer a patient goes untreated the less effective treatment is and
many practitioners will not treat sudden sensorineural hearing loss
after 6 weeks due to the low rate of any meaningful recovery at that
time. Gender has not been shown to be a significant prognostic
factor in sudden sensorineural hearing loss. Men and women are
affected roughly equally. - See Cummings 6th ed pg 2332.

106
Q

A 27 y/o male presents to the facial plastic clinic for
evaluation. On exam he has a left sided deviated
septum, dorsal hump, internal nasal valve collapse
and a bulbous nasal tip. You offer him an open
septorhinoplasty and discuss the various portions
of the procedure including your plan to perform a
cephalic trim. When performing this maneuver how
much alar width must be left in order to avoid
postoperative external nasal valve collapse?A) 2mm B) 5mm C) 7mm D) 10mm

A

C; Removing the cephalic margin of the lateral crura of the lower
lateral cartilages (cephalic trim) is a maneuver used in rhinoplasty to
improve nasal tip shape and definition. It is recommended that at
least 7-9mm of alar width be left in place to prevent alar retraction
and external nasal valve collapse. -See KJ Lee 10th ed pg 759.

107
Q

A 14 y/o male presents to clinic for evaluation of
hearing loss. On exam he is 4’5” tall with a large
head and short extremities. He also has a
prominent brow and saddle nose deformity. His
audiogram demonstrates bilateral mixed hearing
loss. What is the mode of inheritance of this
patient’s condition?
A) Autosomal Dominant B) Autosomal Recessive
C) X-linked Dominant D) X-linked Recessive E)
Mitochondrial Inheritance

A

A; The patient demonstrates signs of dwarfism consistent with
achondroplasia. These patients often suffer hearing loss due to
fusion of the ossicles. The condition is inherited in an autosomal
dominant fashion. As a general rule (with several exceptions)
autosomally dominant hearing loss syndromes demonstrate some
external exam finding (such as white forelock for Waardenburg),
whereas recessive hearing loss syndromes have a relatively benign
exam (such as connexin 26 related hearing loss). - See KJ Lee
10th ed pg 129.

108
Q

A 63 y/o male is taken to the operating room for a
salvage total laryngectomy with pectoralis major
flap coverage. The anesthesiologist asks what
perioperative antibiotic you would like to give.
Which of the following perioperative antibiotics are
not recommended?
A) Cefazolin B) Unasyn C) Vancomycin D)
Cefuroxime E) Clindamycin

A

E; Multiple studies have demonstrated that clindamycin is a poor
choice for perioperative antibiotic coverage prior to total
laryngectomy. Both the rate of postoperative fistulas and infections
are increased with the use of this antibiotic compared to the other
antibiotics that are listed. - See Langerman “Laryngectomy
Complications Are Associated with Perioperative Antibiotic Choice”
and Mitchell “Antibiotic Prophylaxis in Patients Undergoing Head and
Neck Free Flap Reconstruction”

109
Q

A patient presents to clinic with a right sided
chronic otitis externa. He endorses mild hearing
loss in the right ear. On exam he is found to have
canal edema, foul otorrhea, tenderness of the
pinna and a large (50%) tympanic membrane
perforation. You treat the patient with topical
therapy in the office and have him follow up in two
weeks. On repeat exam his otitis externa has
improved but he reports worsening hearing and his
audiogram shows a significant sensorineural
hearing loss on the right side. What topical
medication did you use that could have led to this
complication?
A) Mastoid Powder B) Gentian Violet C) Ciprodex
D) Cipro Otic HC E) Cortisporin

A

B; Gentian Violet is known to be extremely ototoxic and should be
used very carefully (or not at all) in patients with TM perforations.
Mastoid powder (cipro, clotrimazole, dexamethasone and boric acid)
is safe to use in the middle ear and is not ototoxic, nor is ciprodex or
cipro otic. There is a hypothetical risk of ototoxicity with Cortisporin
due to the neomycin. Iatrogenic hearing loss has been shown in an
animal model, however the data on its ability to cause hearing loss in
humans is conflicting and it is not the most likely medication listed to
cause hearing loss. - See Cummings 6th ed pg 2128-2130.

110
Q

A 36 y/o male presents to clinic with tinnitus. He
states that it is present bilaterally, high pitched and
nonpulsatile. He denies hearing loss, aural
fullness, vertigo, otalgia or otorrhea. He tells you
“this noise is driving me nuts, I can’t take it”. He
has tried several herbal medications he found
online as well as noise masking devices without
success. An audiogram is obtained which
demonstrates normal hearing. Which of the
following is the next best step?
A) Trial of Gabapentin B) Hearing Aid C) Cognitive
Behavioral Therapy D) MRI IAC

A

C; High pitched nonpulsatile tinnitus can be distressing to patients
causing significant anxiety and even depression. Although many
treatment options are available very few have evidence supporting
their use. Cognitive behavioral therapy (CBT) is one of the few treatment modalities that is evidence based. A Cochrane review demonstrated tinnitus patients who underwent CBT, while not having
any decrease in tinnitus volume, did had improved quality of life scores and lower depressions scores compared to controls. Hearing aids can improve tinnitus but are not indicated in this patient given his
normal audiogram. An MRI IAC to rule out retrocochlear pathology is generally not recommended in patients with symmetric tinnitus and a
normal audiogram. Gabapentin has been used to treat tinnitus but
evidence for its use is conflicting and would not be the best treatment
option in this patient. - See Cochrane Database “Cognitive behavioural therapy for tinnitus.” 2010.

111
Q

A 22 y/o male patient presents with complaints of
right sided hearing loss. He endorses a hx of “ear
fullness” for the past several years and has
difficulty clearing his ears. He has not had any
previous ear surgeries. On exam, his right
tympanic membrane is intact but he has a
posterosuperior retraction pocket and Rinne is
negative on that side. What is the most likely
diagnosis for this patient?
A) Congenital Cholesteatoma B) Primary Acquired
Cholesteatoma C) Secondary Acquired
Cholesteatoma D) Otosclerosis E) None of the
above

A

B: Primary acquired cholesteatoma is due to persistent and deepening retraction pockets (usually in the pars flaccida) which cause accumulation of keratin debris and formation of a cholesteatoma. Secondary acquired is due to perforations and/or tympanostomy tubes which allow for migration of squamous epithelium into the middle ear space. The etiology of congenital cholesteatomas is unclear but by definition they originate in the middle ear in patients who have never had a perforation of the
tympanic membrane. - See Cummings 6th ed pg 2142-2144.

112
Q

A 35 y/o male presents to clinic after he noticed an
abnormally shaped mole on his left neck. You
perform a punch biopsy in clinic and the pathology
report demonstrates malignant melanoma that is .
82mm thick with ulceration but no mitoses. What is
the T stage for this patient’s melanoma?A) T1a B) T1b C) T2a D) T2b E) T3a

A

B; Malignant melanoma T staging is relatively straightforward.
Anything less than 1 mm thick is T1, between 1-2 mm is T2, 2-4 mm
is T3 and over 4mm is T4. “a” and “b” are determined by whether
there is ulceration (or mitosis in T1 lesions). If ulceration is present
then add a “b” to their stage. Therefore, this patients T stage is T1b.
-See KJ Lee 10th ed pg 650

113
Q

A 39 y/o female presents with an enlarging left
sided parotid gland. A discrete mass is palpable
and a fine needle aspiration is performed which
demonstrates adenoid cystic carcinoma. Which of
the following histological subtypes of adenoid cystic
carcinoma is associated with the best prognosis?A) Cystic B) Solid C) Tubular D) Cribriform

A

D; Adenoid cystic carcinoma is associated with perineural spread
and has three distinct histological subtypes which include tubular,
solid and cribriform. Tubular patterns represents a low grade tumor
and solid tumors are a high grade subtype. Cribriform subtypes
demonstrate a “swiss cheese” type pattern and is associated with the
best prognosis of all of the subtypes. - See KJ Lee 10th ed pg 500-
501.

114
Q

Which of the following is the most accurate
description of the layers of the scalp from
superficial to deep?
A) Skin, galea aponeurosis, subcutaneous tissue,
periosteum B) Skin, subcutaneous tissue, galea
aponeurosis, periosteum C) Skin, loose alveolar
tissue, galea aponeurosis, subcutaneous tissue,
periosteum D) Skin, subcutaneous tissue, loose
alveolar tissue, galea aponeurosis, periosteum E)
Skin, subcutaneous tissue, galea aponeurosis,
loose alveolar tissue, periosteum

A

E; Use the mnemonic SCALP (Skin, subCutaneous tissue, galea
Aponeurosis, Loose alveolar tissue, Periosteum) to remember the
order of the layers of the scalp. - See Tolhurst, “Surgical Anatomy of
the Scalp”; Plastics and Reconstructive Surgery, 1991.

115
Q

A 22 month old male with a history of Down’s
syndrome is referred to the clinic by his speech
therapist to evaluate his hearing. On exam the
patient has bilateral serous effusions and an in
office tympanogram confirms abnormal movement
of the tympanic membranes. He has never been
seen by an otolaryngologist in the past and has no
history of ear infections. What is the next best step
in the management of this patient?
A) Place bilateral tympanostomy tubes B) 14 day
course of amoxicillin C) Reexamine the child at 3
months D) Adenoidectomy

A

A; Statement 9 of the clinical practice guidelines for tympanostomy
tubes in children states that, “Clinicians may perform tympanostomy
tube insertion in at-risk children with unilateral or bilateral OME that is
unlikely to resolve quickly, as reflected by a type B (flat)
tympanogram or persistence of effusion for 3 months or longer.”
While the guidelines recommend waiting 3 months and reexamining
the patient in an otherwise healthy child to see if the effusion has resolved, given that this patient is at high risk for persistent effusion
because of his Down’s syndrome and currently experiencing a speech delay it is reasonable to place tympanostomy tubes. Antibiotics are never recommended for serous effusions. While an
adenoidectomy can decrease the rate of acute otitis media it is not
generally recommended as first line management of serous effusions. - See Rosenfeld “Clinical Practice Guideline - Tympanostomy Tubes in Children—Executive Summary” 2013.

116
Q

A 44 y/o female presents with left sided pulsatile
tinnitus and aural fullness. On exam she has a
reddish left sided middle ear mass on the
promontory. Which of the following is not
consistent with the most likely diagnosis?
A) Positive Brown sign B) Positive Aquino sign C)
Elevated urine vanillylmandelic acid D) Antoni A
pattern on histology E) “Salt and Pepper”
appearance on MRI

A

D; The patient has a presentation most consistent with a glomus
tympanicum (glomus tumor). The classic histology for glomus tumors
shows clusters of round/cuboidal cells called Zellballen (German for
balls of cells). Brown sign (blanching of the mass on pneumatic
otoscopy) and Aquino sign (blanching of the mass with compression
of the ipsilateral carotid) would both be positive. Urine
vanillylmandelic acid (VMA) and metanephrines could be elevated if
the tumor was functional although this is uncommon (less than 5%).
MRI demonstrates a characteristic “salt and pepper” appearance due
to flow voids within the tumor. Antoni A and B patterns are seen in
schwannomas and would not be consistent with the diagnosis of
glomus tympanicum. - See KJ Lee 10th ed pg 357 and 681-683.

117
Q

Which of the following is the most common subtype
of malignant melanoma?
A) Desmoplastic B) Acral Lentiginous C) Lentigo
Maligna D) Nodular E) Superficial Spreading

A

E; Superficial spreading melanoma is the most common subtype
accounting for 75% of all cases. It generally has a good prognosis as
it remains superficial for a significant amount of time and therefore is
often diagnosed at a relatively early stage. Desmoplastic is rare,
can be amelanotic and has high affinity for perineural spread. Acral
lentiginous is most often found on the soles and palms, is more
common in darker skinned individuals and is not related to sun
exposure. Lentigo maligna is the least common subtype and can
have a radial growth phase that lasts for decades. Nodular has no
radial growth phase and instead very quickly demonstrates vertical
growth leading to a late stage at diagnosis and poor prognosis. - See
KJ Lee 10th ed pg 645-646.

118
Q

A 44 y/o female presents with acute onset true
vertigo, tinnitus and fluctuating hearing loss. She
states these symptoms developed acutely over the
past month but denies any past otologic history.
On exam she has red and irritated appearing eyes
however the rest of her exam in unremarkable.
She has a past medical history of periarteritis
nodosa. What is the most likely diagnosis?
A) Syphilis B) Cogan Syndrome C) Meniere’s D)
Vogt-Koyanagi-Harada Syndrome E) Giant Cell
Arteritis

A

B; This patient’s presentation is most consistent with Cogan
syndrome which is characterized by nonsyphilitic interstitial keratitis,
vertigo, tinnitus, nystagmus and hearing loss. It is thought to be
related to periarteritis nodosa. While tertiary syphilis can cause
similar symptoms including interstitial keratitis there is not usually
evidence of active inflammatory changes. Meniere’s disease is often
confused for Cogan syndrome but does not involve interstitial
keratitis. Vogt-Koyanagi-Harada is similar to Cogan but also involves
alopecia and decreased melanin. Giant cell arteritis (aka temporal
arteritis) presents w/ fever, headaches, jaw claudication, vision loss
and polymyalgia rheumatica. It can be diagnosed with a temporal
artery biopsy. - See KJ Lee 10th ed pg 226.

119
Q

A 62 y/o male presents to the emergency
department with diplopia, deep facial pain and left
sided otorrhea. The patient has diplopia due to
involvement of which structure?
A) Meckel’s Cave B) Dorello’s Canal C) Huschke’s
Foramen D) Fissures of Santorini E) Huguier’s
Canal

A

B; The patient presents with the full triad of Gradenigo syndrome
which is due to petrositis. The patient’s diplopia is due to abducens
nerve palsy which is caused by involvement of Dorello’s canal which transmits CN VI. The deep facial pain is likely due to involvement of
CN V in Meckel’s cave. Huschke’s foramen is a nonossified portion
of the anterior external auditory canal that is present in a portion of the population. The Fissures of Santorini are vertical fissures in the cartilaginous external auditory canal through which infections can travel to the mastoid and surrounding tissue. Huguier’s canal is lateral to the roof of the protympanum and transmits the chorda tympani out of the temporal bone. - See KJ Lee 10th ed pg 3.

120
Q

A 62 y/o male with significant smoking history and
T3N2cM0 oral squamous cell carcinoma presents
to the clinic. On exam he has good dentition and a
large ulcerated lesion of the left anterior floor of
mouth. You offer him a composite resection
including anterior mandibulectomy. What
technique for reconstructing the defect would be
most appropriate in this patient?
A) Titanium reconstruction plate with primary
closure B) Titanium reconstruction plate with
anterolateral thigh free flap C) Titanium
reconstruction plate with pec flap coverage D)
Titanium reconstruction plate with osseous radial
forearm free flap E) Titanium reconstruction plate
with osseous fibular free flap

A

E; The key information in this question is that the patient’s mandibular defect is located anteriorly. While lateral mandibular defects can be closed with a variety of techniques, this is not the
case anteriorly. Anterior defects have a much higher rate of plate extrusion due to the tension of the soft tissue over the reconstructed
region. Therefore, covering an anterior reconstruction plate with primary closure, regional flap or soft tissue free flap leads to
unacceptable morbidity. Osseous free flaps are preferred and several factors determine which osseous free flap is best. In this
patient, dental reconstruction should be a consideration. While a radial forearm flap can be used to reconstruct the defect it is not thick enough to support future dental restoration and therefore is not the
best choice. The fibula free flap is a good choice as it allows for dental implants in the future, has minimal donor site morbidity, can be
accompanied by a large skin paddle and can provide up to 25 cm of donor bone. Other options include an iliac crest, lateral scapula
border or scapula tip graft. None of these would be ideal however due to donor site morbidity, decrease length and poor shape of the
harvested bone. - See Cummings 6th ed pg 1393-1394.

121
Q

A 25 y/o male is brought to the ED by his
roommate for altered mental status. The patient is
moderately obtunded but is able to tell you he has
a history of chronic ear infections and that he has
had a severe headache for the past 24 hours. On
exam he has a fever of 103.9, purulent discharge
from his left ear, nuchal rigidity and disruption of
conjugate gaze. Which of the follow is not an
appropriate next step?
A) Ophthalmoscopic exam B) Starting IV antibiotics
C) Neurosurgery consult D) CT Brain w/ contrast E)
Lumbar puncture

A

E; This patient demonstrates signs concerning for intracranial
complications of chronic otitis media. This could include meningitis,
lateral sinus thrombosis, epidural abscess, intraparenchymal abscess
or subdural empyema. It is inappropriate to perform a lumbar
puncture prior to ruling out elevated intracranial pressure (ICP) as
this can lead to tonsillar herniation therefore E is not an appropriate
next step. CT scans and ophthalmoscopic exams (looking for
papilledema) can help rule out elevated ICP. Neurosurgery should
be consulted in any patient with an intracranial complication and IV
antibiotics should be started as soon as possible. - See KJ Lee 10th
ed pg 325.

122
Q

A 78 y/o male presents to the clinic with complaints
of gradual onset hearing loss. He denies otalgia,
otorrhea, aural fullness or vertigo. His exam is
unrevealing. An audiogram is performed which
demonstrates the following pure tone averages and
speech discrimination scores of AD:45% and AS:
65%. Which of the following is the most likely
diagnosis?
A) Noise Induced Hearing Loss B) Strial
Presbycusis C) Neural Presbycusis D) Sensory
Presbycusis E) Mechanical Presbycusis

A

C; Age related hearing loss (aka presbycusis) can be divided into
four categories (Neural, Sensory, Mechanical, Strial). Neural
presbycusis is due to degeneration of the spiral ganglion and spiral
lamina nerves. It’s characteristic presentation is downward sloping
sensorineural hearing loss with speech discrimination scores lower
than what would be expected. Noise induced hearing loss is likely to
present with a “noise notch” at 4 kHz. Strial presbycusis is due to
breakdown of the stria vascularis and presents sensorineural hearing
loss that appears flat on audiogram. Sensory presbycusis is due to
atrophy of the inner hair cells and supporting cells in the organ of
corti. It presents with downward sloping sensorineural hearing loss
but speech discriminations that correlate with pure tone averages.
Mechanical presbycusis (aka inner ear conductive hearing loss) is
poorly understood and likely due to stiffening of the basilar
membrane. - See Cummings 6th ed pg 231-232.

123
Q

A 44 y/o male from Tennessee presents to your
office with long standing nasal congestion, loss of
sense of smell and facial pressure. On nasal
endoscopy he demonstrates thickened brownish
green nasal secretions emanating from the right
maxillary sinus which you sample and send to the
lab. Which of the following is not one of the criteria
to confirm the suspected diagnosis in this patient?
A) Presence of nasal polyps B) CT demonstrates
sinuses with hypointense rims surrounding a central
area of hyperdensity C) Presence of Charcot-
Leyden crystals D) Type 1 hypersensitivity on skin
test E) All of the above are criteria

A

E; Given this patient’s characteristic “peanut butter” like nasal
discharge, symptoms of chronic sinusitis and fact that he is from the
south, allergic fungal sinusitis (AFS) is the suspected diagnosis. In
1994 Bent and Kuhn delineated five criteria for diagnosis of AFS
which include: 1. Presence of eosinophilic mucin (which contains
Charcot-Leyden crystals) 2. Presence of noninvasive fungal hyphae
3. Presence of nasal polyps 4. Characteristic radiographic findings
(including hyperdense central material surrounded by hypointense
rim) 5. Type 1 hypersensitivity (by skin test, blood work or history) -
See KJ Lee 10th ed pg 409 and Bent “Diagnosis of allergic fungal
sinusitis.” 1994.

124
Q

You are in the operating room performing endoscopic
sinus surgery. After performing an uncinectomy your
attending asks you to proceed by removing the largest of
the anterior ethmoid cells. Which of the following is true
about this cell?
A) It forms the posterior border of the frontal recess
B) It is also the most anterior ethmoid cell C) When
present it is based on the orbit and extends into the
maxillary sinus D) When present can extend
superior to the sphenoid and involve the carotid
artery and optic nerve E) Lies directly posterior to
the basal lamella

A

A; The largest anterior ethmoid cell is the ethmoid bulla. It is also
one of the most consistent cells and is located in the middle meatus
directly anterior to the basal lamella and posterior to the uncinate
process. It often extends superiorly to the skull base in which case it
forms the posterior border of the frontal recess (if it does not then a
suprabullar recess is formed). The most anterior ethmoid cell is the
agger nasi. Haller cells are based on the orbit and extend into the
superior medial maxillary sinus potentially obstructing the
osteomeatal complex. An Onodi cell can extend superior to the
sphenoid and involve the carotid artery and optic nerve. The
posterior ethmoid cells begin directly behind the basal lamella. - See
KJ Lee 10th ed pg 366-371.

125
Q

A 62 y/o male presents to the clinic with hearing
loss. He states it has been gradual in nature,
bilateral and accompanied by high pitched tinnitus.
He denies otalgia, otorrhea, aural fullness or
vertigo. His audiogram is shown. Which of the
following is the most accurate diagnosis?
A) Noise Induced Hearing Loss B) Strial
Presbycusis C) Neural Presbycusis D) Sensory
Presbycusis E) Mechanical Presbycusis

A

B; Age related hearing loss (aka presbycusis) can be divided into
four categories (Neural, Sensory, Mechanical, Strial). Strial
presbycusis is due to breakdown of the stria vascularis and presents
as sensorineural hearing loss that appears flat on audiogram. Noise
induced hearing loss is likely to present with a “noise notch” at 4 kHz.
Neural presbycusis is due to degeneration of the spiral ganglion and
spiral lamina nerves. It’s characteristic presentation is downward
sloping sensorineural hearing loss with speech discrimination scores
lower than what would be expected by pure tone averages. Sensory
presbycusis is due to atrophy of the inner hair cells and supporting
cells in the organ of corti. It presents with downward sloping
sensorineural hearing loss but speech discriminations that correlate
with pure tone averages. Mechanical presbycusis (aka inner ear
conductive hearing loss) is poorly understood and likely due to
stiffening of the basilar membrane. - See Cummings 6th ed pg 231-232

126
Q

7 y/o male is taken to the operating room for
excision of a left parotid mass. You make your
incision, identify and preserve the facial nerve and
expose the tumor. While attempting to remove the
tumor you discover that the mass is much larger
than was anticipated and has invaded into the
carotid space. What structure separates the
parotid space from the carotid space?
A) Masseter muscle B) Stylohyoid muscle C)
Stylomandibular ligament D) Posterior belly of
digastric muscle E) Superior constrictor muscle

A

D; The parotid space contains the facial nerve and parotid gland. It
is located lateral to the carotid space and posterior to the masseter
space. It extends inferiorly to the angle of the mandible and
superiorly to the mid squamous temporal bone. The carotid space is
also known as the poststyloid parapharyngeal space. It contains the
carotid artery, jugular vein, lymph nodes, sympathetic plexus and
CNs IX-XII. The parotid space is separated from the carotid space
by the posterior belly of the digastric muscle. - See Cummings 6th
ed pg 124.

127
Q

Which of the following has a significant impact on a
patient’s prognosis across all stages of melanoma?
A) Location of primary lesion B) Clark’s level C)
Ulceration D) Mitotic rate E) History of sun
exposure

A

C; If you examine the 2010 AJCC cutaneous melanoma staging
criteria ulceration is the only criteria that is used across all stages. All
T stages are divided into “A” and “B”, without or with ulceration
respectively. This ultimately affects the staging of the patient and
their prognosis. Location of the primary melanoma is not significantly
correlated with prognosis in all stages of cutaneous melanoma.
Although the Clark’s level of invasion was previously used when
assessing T1 lesions, this was removed in the updated 2010
guidelines. Mitotic rate is still used in T1 lesions, but is not a
prognostic factor in all stages of disease. Sun exposure does not
significantly affect the patient’s prognosis although it is a risk factor
for developing cutaneous melanoma in the first place. The other
factor that significantly affects prognosis across all stages is tissue
depth. Deeper invasion is associated with poorer prognosis. Along
this line, subtypes of melanoma which rapidly invade vertically such
as nodular melanoma have a worse prognosis. Melanoma subtypes
such as lentigo maligna and superficial spreading melanoma have
long axial radial growth phases and therefore an overall better
prognosis. - See Cummings 6th ed pg 1166-1167.

128
Q

A 29 y/o male presents to your clinic for workup of
a thyroid nodule. On FNA he is found to have a
follicular lesion and you offer him surgery. On
review of his history you find that he has been
treated for multiple fibromas, osteomas and
epidermoid inclusion cysts. He also has a history
of colon cancer which has been treated. What is
the most likely diagnosis for this patient?
A) Pendred Syndrome B) Cowden Disease C)
Multiple Endocrine Neoplasia Type IIb D) Alport
Syndrome E) Gardner Syndrome

A

E; Gardner syndrome is an autosomal dominant disease
characterized by multiple osteomas of the mandible, maxilla and skull
as well as fibromas of the soft tissue and epidermoid inclusion cysts.
These patients also have multiple polyps of the colon that are
predisposed towards developing malignancy. They most commonly
present to the ENT clinic with well differentiated thyroid cancer to
which they are also predisposed. Pendred syndrome presents with
congenital sensorineural hearing loss and euthyroid goiter. Cowden
disease (aka multiple hamartoma syndrome) is characterized by
hypoplasia of the maxilla, mandible and soft palate as well as
microstomia, papillomatosis of the lips/pharynx, multiple thyroid
adenomas, pectus excavatum and scrotal tongue. Multiple
Endocrine Neoplasia Type IIb includes medullary thyroid cancer,
pheochromocytomas, and marfanoid habitus. Alport syndrome is
characterized by congenital hearing loss, end stage renal disease
and vision problems. - See KJ Lee 10th ed pg 231.

129
Q

A 6 y/o female presents to your clinic with a right sided ear infection and corresponding facial nerve weakness. You diagnose her with acute otitis media and her facial nerve function recovers after she is treated with a myringotomy, ciprodex and IV antibiotics. Afterwards the patient’s parents ask
you why their other children never had facial weakness when they had ear infections. You explain to them that the patient could have been at increased risk due to a bony dehiscence of the facial nerve in the middle ear. What percentage of the population have a bony dehiscence of the tympanic segment of the facial nerve?

A) Exceedingly rare B) 0.1% C) 1% D) 10% E) 20%

A

E; Bony dehiscence of the fallopian canal is quite common and makes facial nerve dysfunction due to middle ear infections more likely. Estimates of how common this dehiscence is ranges widely. KJ Lee states that 20% of the population has a dehiscence however
Baxter dissected 535 temporal bones at Mass Eye and Ear in 1971 and found 294 dehiscences (55%). The take home message is that
you should expect a dehiscence of the facial nerve in a large number
of patients. - See KJ Lee 10th ed pg 321.

130
Q

A 63 y/o male presents with dysphagia. He states
his symptoms have slowly progressed over the
past few years but he finally sought care now that
he is losing weight. He denies any pain associated
with his symptoms but does endorse borborygmi,
halitosis and globus sensation. He has no
significant findings on a thorough head and neck
exam including flexible laryngoscopy. Barium
swallow esophagram is ordered and confirms the
most likely diagnosis. Hyperactivity of which
muscle is most likely responsible for this patient’s
symptoms?
A) Cricopharyngeus B) Cricoarytenoid C)
Esophageal smooth muscle D) Superior constrictor
E) Inferior constrictor

A

A; This patient’s presentation is most consistent with a Zenker’s
diverticulum which is a dehiscence in Killian’s triangle (midline,
inferior constrictor muscles and cricopharyngeus). The most
common presenting symptom is progressive dysphagia however
other symptoms include regurgitating undigested food, weight loss,
borborygmi (noisy swallowing), halitosis, aspiration, choking, globus,
and hoarse voice. It is thought that overactivation of the
cricopharyngeus muscle prevents distal migration of food boluses
and leads to pressure on the pharyngeal wall at Killian’s triangle,
eventually leading to dehiscence. The cricoarytenoid muscle is
actually two separate muscles, the posterior cricoarytenoid and the
lateral cricoarytenoid. The posterior cricoarytenoid is the only muscle
which abducts the vocal folds. While the inferior constrictor muscle
does form one border of Killian’s triangle, overactivation does not
cause a dehiscence. The superior constrictor muscle is well above
this region and is not involved in this process. While overactivation of
the esophageal musculature can cause dysphagia this is typically a
painful process and is less common than Zenker’s. - See KJ Lee
10th ed pg 523.

131
Q

You are examining a patient who has high fever
and neck stiffness in the setting of an acute ear
infection. As part of your exam you lay the patient
on their back with their thighs at a right angle to
their trunk and then attempt to extend their leg at
the knee. What is the name of this exam
maneuver?
A) Griesinger Sign B) Bruns Sign C) Kernig Sign D)
Brudzinski Sign E) None of the above

A

C; This is a description of Kernig sign. Meningitis should be
suspected if the patient has significant pain or restriction on
extending the knee. Brudzinski’s sign is when when the examiner
flexes the neck of the patient which causes reflexive flexion of the
legs as well. Bruns sign is vertigo and headache associated with
sudden movement that is caused by tumors of the fourth ventricle.
Griesinger sign is edema of the mastoid tip due to venous sinus
thrombosis. - See KJ Lee 10th ed pg 254-256.

132
Q

A 59 y/o male presents to the clinic with a 2.4 cm
right sided thyroid nodule found incidentally on CT
scan. You perform an ultrasound which
demonstrate a >2 cm nodule with irregular borders
and microcalcifications but no extrathyroidal spread
and no evidence of lateral neck lymphadenopathy.
A fine needle aspiration (FNA) is performed which
demonstrates a Bethesda V result. What is the
next best step in the management of this patient?
A) Repeat FNA B) Repeat Thyroid Ultrasound In 1
Year C) Radioactive Iodine Ablation D) Thyroid
Lobectomy E) Total Thyroidectomy

A

D; The Bethesda grading system is used to report thyroid FNA
results. Bethesda V indicates suspicion for malignancy (60-75%) and
therefore warrants intervention. Assuming this patient has well
differentiated thyroid carcinoma they would be categorized as Stage
II (T2N0M0). The most recent 2015 guidelines suggested a thyroid
lobectomy for patients with low risk well differentiated cancer
(unifocal disease less than 4cm w/o extrathyroidal extension) due to
substantial evidence that performing a total thyroidectomy does not
improve the already excellent prognosis. Given the high chance of
malignancy for Bethesda category V there is no need for a repeat
FNA. Observation with repeat ultrasound in 1 year is not appropriate
for a nodule that has a 75% chance of being malignant. Radioactive
iodine ablation cannot be performed in a patient who still has their
thyroid. - See ATA Thyroid Nodule Guidelines 2015.

133
Q

A 36 y/o male presents with right sided throat pain,
ear pain and globus sensation. He states these
symptoms have been present for several months
and it is sometimes worse when he turns his head
to the side. On exam he has 2+ tonsils and you
are able to palpate a firm mass in the right tonsillar
fossa. You make the diagnosis of Eagle syndrome
but decide to confirm it with use of a CT scan.
What is the shortest length of the styloid process
which would be considered abnormal?A) 2 cm B) 2.5 cm C) 3 cm D) 3.5 cm E) 4cm

A

C; Eagle syndrome is thought to be caused by an elongated styloid
process (or calcified stylohyoid ligament) which compresses the
glossopharyngeal nerve (or perhaps the carotid, the etiology is not
well understood). A normal styloid process is 2.5cm or less.
However, it should be noted that ~4% of the population has a styloid
process greater than 3cm, yet only a small fraction of those patients
have symptoms consistent with Eagle syndrome. Treatment involves
tonsillectomy and excision of the distal end of the elongated styloid
process. - See Cummings 6th ed pg 1350.

134
Q

You are reviewing your clinic schedule for the next
day when you notice that one of your patients is a 2
week old male born at term who is being brought in
to evaluate his stridor. Assuming that this patient
has the most common cause of stridor, which of the
following is an appropriate way to counsel the
parents?
A) The stridor will likely become worse in the next
few months B) Supraglottoplasty will likely be
recommended for this patient C) Diagnosis can be
made based on the history and characteristics of
the stridor D) There is a 75% chance of the patient
having a synchronous lesion E) The patient’s stridor
will likely improve while feeding

A

A; By far the most common cause of stridor in the neonatal period is
laryngomalacia. Despite sounding concerning to the parents, the
majority of these patients will do well with observation and will not
require any intervention as the stridor resolves spontaneously,
usually within a year. If the patient’s stridor does not improve or they
demonstrate severe laryngomalacia with failure to thrive then
supraglottoplasty can be considered. Diagnosis of laryngomalacia
can only be made using a flexible laryngoscope which will
demonstrate floppy arytenoid tissue and infolding of the epiglottis.
An omega shaped epiglottis is often found but can be present in
healthy patients as well. There is a significant chance of
synchronous lesions in laryngomalacia patients, but the incidence is
lower than 75% (estimates range from 8-58%). The laryngomalacia
stridor often worsens with feeding and but can improve with crying.
While this patient’s stridor will likely resolve spontaneously, as the
child becomes more active, oxygen demands increase and they will
often have subjective worsening of the stridor initially. - See
Cummings 6th ed pg 3123-3124.

135
Q

A patient who underwent primary bilateral
endoscopic sinus surgery at an outside hospital 6
months ago presents to the clinic with continued
sinus symptoms. He states that his previously
bilateral congestion and facial pressure are
significantly improved but he routinely develops
intermittent left sided congestion and thick but
nonpurulent discharge. On exam he has some
residual posterior ethmoid cells, a left sided septal
deviation, a large left sided antrostomy posterior to
the natural os and a lateralized middle turbinate.
Which of the following is the next best step in
management?
A) Increased Nasal Saline Irrigation B) Septoplasty
C) Place Middle Meatus Spacer D) Middle Meatus
Culture and Antibiotics E) Revision Maxillary
Antrostomy

A

E; The patient’s exam is concerning for a maxillary antrostomy not
connected to the natural os. The mucociliary clearance pathway
directs mucous through the natural os however it can fall back into an
unconnected antrostomy and be recirculated leading to intermittent
nasal congestion and discharge. This can be addressed with
revision maxillary antrostomy to connect it to the natural os. While
increasing nasal saline irrigation may help clear some of the nasal
discharge it will not address the underlying issue. Septoplasty and
medialization of the middle turbinate with a spacer can both improve
nasal airflow but would not address nasal discharge. Middle meatus
culture and antibiotics would be the appropriate management if the
patient had evidence of an infection but his discharge is nonpurulent
and he has no other signs of acute sinus infection. - See Cummings
6th ed pg 767.

136
Q

A 53 y/o female with a long history of chronic ear
infections presents with severe headache, left
sided otorrhea and + Griesinger sign. A CT brain
with contrast reveals a positive “Delta sign” on the
left. What other sign/symptoms would you expect
in this patient given the most likely diagnosis?
A) Synkinesis of the left facial nerve B) Diplopia and
retro orbital pain C) Brudzinski Sign D) Right gaze
nystagmus E) Recurrent afternoon fevers

A

E; This patient presents with signs and symptoms concerning for
lateral sinus thrombosis which classically has severe headaches,
papilledema, septic emboli to the lungs, otorrhea and
edema/tenderness over the mastoid (Griesinger sign). CT brain with
contrast will often show a rim enhancement of the sinus with central
hypodensity known as a “Delta sign”. Another common sign is
“picket fence” fevers which are fevers that tend to cluster during a
particular part of the day. - See KJ Lee 10th ed pg 321-325.

137
Q

A 24 y/o male is referred to your clinic with a known
HPV+ squamous cell carcinoma(SCC) of the
oropharynx. What is the most likely presenting
symptom in this patient?
A) Odynophagia B) Dysphagia C) Dysphonia D)
Asymptomatic oropharyngeal lesion E)
Asymptomatic neck mass

A

E; HPV+ SCC most commonly affects the oropharynx and is
becoming increasingly prevalent. It tends to present with small
primary lesions and advanced nodal disease making them far more
likely to present as asymptomatic neck masses with unknown
primaries. On imaging these neck masses often demonstrate cystic
nodes that can be misdiagnosed as branchial cleft cysts. HPV+
SCC does not usually present with pain or significant obstruction
therefore odynophagia and dysphagia are incorrect. Dysphonia
would not be expected from a lesion of the oropharynx. While HPV+
SCC can present as an asymptomatic oropharyngeal lesion this is
less commonly noticed than a neck mass. - See Cummings 6th ed
pg 1084.

138
Q

A 50 y/o female is taken to the operating room for bilateral upper lid blepharoplasties for dermatochalasis. In the preoperative area you identify the relevant superficial landmarks and mark out your proposed incision. During this process you measure the distance from the upper eyelid margin to the upper eyelid crease. In this patient, what is the expected distance between these two structures?

A) 7mm B) 9mm C) 11mm D) 13mm E) 15mm

A

C; The upper lid crease is formed by insertion of the levator aponeurosis and orbital septum into the orbicularis oculi muscle and adjacent skin. It is an important landmark that needs to be identified preoperatively as it forms the border of the resectable tissue. In women the crease is usually 10-12 mm above the lid margin, however in men it is only 7-8 mm. - See Cummings 6th ed pg 442-443.

139
Q

An 18 y/o male suffers blunt trauma to the head
and is appreciated to have complete left sided
facial paralysis on presentation to the trauma bay.
A CT scan reveals a transverse temporal bone
fracture coursing through the otic capsule. You
follow the patient closely and perform serial ENOG
exams. 1 week after his trauma he demonstrates
left sided function which is 55% of the normal right
side. The decision is made to take the patient to
the operating room for decompression of the nerve.
Which region of the nerve should be
decompressed for optimal results?
A) Tympanic Segment B) Mastoid Segment C)
Labyrinthine Segment D) Perigeniculate Region E)
Two of the above F) Three of the above G) All of
the above

A

E; The most commonly injured segment of the facial nerve during
temporal bone trauma is the perigeniculate region. It is thought that
this is due to the greater superficial petrosal nerve (GSPN) which
may cause an avulsion injury to the nerve in this area during trauma.
However, a significant number of patients will have a second lesion in
the mastoid segment and therefore this region must be explored and
decompressed as well for optimal results. - See Cummings 6th ed
pg 2226.

140
Q

Which of the following is not a disadvantage of
lateral graft technique tympanoplasty?
A) Limited to perforations that are 50% or less B)
Extended healing time C) More technically
demanding for the surgeon D) Risk of blunting E)
Risk of lateralization of the graft

A

A; One of the advantages of the lateral graft technique is that it can
be used for any size tympanic membrane perforation. Extending
healing time, increased demand on the technical proficiency of the
surgeon and risk of blunting/lateralization of the graft are all risks of
the lateral technique. Regardless of technique however, in
experienced hands closure can be expected in ~90% of cases. - See
Cummings 6th ed pg 2180-2183.

141
Q

Both erythroplakia and leukoplakia are common
lesions evaluated in the clinic. What percentage of
erythroplakia and leukoplakia will demonstrate
severe dysplasia or carcinoma on biopsy?
A) 50% / 10% B) 50% / 50% C) 25% / 10% D) 25%
/ 25% E) 10% / 25%

A

A; Erythroplakia is a much more concerning finding on examination
of the oral cavity. On biopsy, approximately 50% of lesions will
demonstrate severe dysplasia or carcinoma in situ. Leukoplakia is
less concerning, however there is still a 10% chance of severe
dysplasia/carcinoma and therefore biopsy is warranted. - See KJ
Lee 10th ed pg 516.

142
Q

A 19 y/o male is brought to the trauma bay for
evaluation of blunt head trauma. On exam he is
noted to have left sided House Brackmann V facial
weakness, left sided conductive hearing loss and
hemotympanum. He is admitted and stabilized by
the neurosurgical team. What is the best initial test
to determine the need for operative intervention for
this patient’s facial weakness?
A) EMG B) EEG C) ENOG D) Maximum stimulation
test E) None of the above

A

E; None of the above tests are warranted because the patient
demonstrates some facial nerve function. Surgery is only indicated
for patients with immediate onset total facial nerve paralysis in
patients who demonstrate >90% degeneration on ENoG within 2
weeks of injury. However, since this patient is HB V (instead of total
paralysis which is HB VI) no further testing or intervention other than
steroids and observation is recommended. - See Cummings 6th ed
pg 2227.

143
Q

A 9 y/o male is brought to clinic by his mother with
complaints that his ears are too big. You offer the patient
surgical intervention to correct the issue. Unfortunately,
at his post op visit the patient’s mother is displeased with
the result stating that his ears have a “telephone”
appearance. What surgical maneuver likely led to this
complication?
A) Ely technique B) Monks technique C) Furnas
technique D) Mustarde technique E) Cartilage
sculpting

A

C; Furnas technique refers to conchal setback which involves
placing permanent sutures to approximate the conchal cartilage
against the mastoid periosteum. Overly aggressive conchal setback
can create a telephone ear deformity by pulling the middle third of the
auricle more posterior compared to the upper and lower poles. Ely
and Monks techniques are both historical otoplasty techniques that
are no longer commonly used. Mustarde technique is used to create
an antihelical fold and involves placing 3-6 sutures along the scapha.
Cartilage sculpting involves scoring or excising cartilage along the
area where an antihelix is desired in order to allow for the cartilage to
conform to the desired shape. In both the Mustarde and cartilage
sculpting techniques, overcorrection of the antihelix can result in the
antihelix being the most lateral structure on the frontal view (aka
hidden helix deformity) or buckling of the helical rim creating a
vertical scaphal fold (aka vertical post deformity). - See Cummings
6th ed pg 468-472.

144
Q

You are performing botox injections in the clinic for
glabellar crease when you realize you have run out
of Botox®. You are able to find some Dysport in
your medication refrigerator. You were planning on
injecting 20 units of botox into your next patient.
How many units of Dysport should you inject as an
equivalent?
A) 10 units B) 20 units C) 50 units D) 75 units E)
100 units

A

C; Both Dysport and Botox® are botulinum toxin A which can be
used for therapeutic and cosmetic muscle paralysis. Botox® is
produced by Allergan (USA) whereas Dysport is made by Galderma
(UK) and although they have similar functions their dosing is
different. In general 4 units of Botox® is equivalent to 10 units of
Dysport. - See Cummings 6th ed pg 436.

145
Q

A 31 y/o male presents to the clinic with an oral
lesion. He first noticed it ~ 6 months ago and it has
not changed significantly since then. He denies
pain, discharge, fatigue or weight loss. He has no
significant past medical history and does not have
a history of smoking. On exam there is a 7mm
pedunculated, nonulcerated, raised lesion on the
left posterior oral tongue. You perform an
excisional biopsy and send the lesion to pathology. What
is the most likely result of the biopsy?
A) Squamous papilloma HPV subtype 16 B)
Squamous carcinoma HPV subtype 16 C)
Squamous papilloma HPV subtype 11 D)
Squamous carcinoma HPV subtype 11 E)
Squamous papilloma HPV subtype 33 F)
Squamous carcinoma HPV subtype 33

A

C; This patient’s lesion is most consistent with a benign papilloma
given its stable size, asymptomatic nature and exam findings that do
not demonstrate ulceration. Oral papillomas are HPV related lesions
most commonly caused by HPV subtypes 6 and 11. Squamous cell
carcinoma is most commonly caused by HPV subtypes 16, 18, 31
and 33. It is thought that the HPV antibodies to E6 and E7 are
oncogenes which downregulate p53 and retinoblastoma protein (Rb)
respectively leading to the development of neoplasia. - See
Cummings 6th ed pg 3143.

146
Q

Which of the following glands is not innervated by
the facial nerve?
A) Nasal Gland B) Lacrimal Gland C) Sublingual
Gland D) Submandibular Gland E) Parotid Gland

A

E; The parotid gland obtains its parasympathetic innervation via the
glossopharyngeal nerve. The preganglionic nerve bodies originate in
the inferior salivatory nucleus, travel along the glossopharyngeal
nerve, through the lesser petrosal nerve and into the otic ganglion.
At this point they synapse with postganglionic fingers and travel via
the auriculotemporal nerve to the parotid gland. The facial nerve
does not have a role in this pathway. The lacrimal and nasal glands
are innervated by the facial nerve via the greater superficial petrosal
nerve. The sublingual and submandibular glands are also innervated
by CN 7 via the chorda tympani and the lingual nerve. - See KJ Lee
10th ed pg 194-196.

147
Q

A patient presents to the clinic with diffuse oral
mucosal blistering that developed over the last
several weeks. He states the blisters are painful
and that his oral intake has been significantly
limited. He has a family history of various
autoimmune disorders but can’t remember which
ones specifically. On exam he has multiple painful
blisters of the oral mucosa which slough off with
gentle palpation as well as several similar blisters
on the skin of his arms and chest. You obtain a
biopsy which reveals blisters between the dermis
and epidermis with destruction of the basement
membrane. Pathology sends the specimen for
further immunofluorescent testing which reveals an
autoimmune disorder. What is the most likely
diagnosis?
A) Pemphigus vulgaris B) Bullous Pemphigoid C)
Systemic Lupus Erythematosus D) Syphilis E)
Wegener’s Granulomatosis

A

B; This patient’s presentation of painful blistering of the skin and oral
mucosa is most consistent with either pemphigus vulgaris or bullous
pemphigoid. Systemic lupus erythematosus, syphilis and wegener’s
are unlikely to cause such blisters. Differentiating pemphigus from
pemphigoid cannot be done clinically. Pemphigus vulgaris blisters
are caused by desmogleins which break connections between the
epithelial cells. Bullous pemphigoid blisters are due to destruction of
the basement membrane and therefore is the most likely diagnosis in
this patient given the biopsy results. An easy way to remember this
is that the “Bs” stick together, Bullous pemphigoid attacks the
Basement membrane. - See KJ Lee 10th ed pg 549.

148
Q

The combination of the lever mechanism of the
ossicles and the ratio of the TM to the oval window
provide how much of a gain in hearing?A) 1.3 dB B) 23 dB C) 26 dB D) 28 dB

A

B; The malleus and incus lever mechanism is estimated to create
1.3 dB of gain however the ratio of the round window to oval window
creates 26 dB of gain. In combination there is a theoretical gain of 28
dB gain however in reality inefficiencies in the system cause a final
average gain of only 23 dB. - See KJ Lee 10th ed pg 148.

149
Q

A 66 y/o male with a long history of chewing
tobacco use presents with an upper lip lesion. He states it has been growing for the past several months but denies any pain. On exam there is a 4 cm ulcerative lesion which abuts but does not involve
the commissure on the right. He has no palpable lymphadenopathy. A biopsy is obtained to confirm the diagnosis. Which of the following is true with regards to this patient’s disease?

A) There is a >20% risk that this patient has occult cervical nodal metastasis B) Basal cell carcinoma is more common on the lower lip compared to the upper lip C) The location of this patient’s carcinoma decreased his 5 year survival by 10-20% D) Lip carcinoma is the second most common site for oral
cavity carcinoma after oral tongue carcinoma E) The most likely diagnosis for an upper lip lesion is basal cell carcinoma

A

C; While 95% of lip cancers are located on the lower lip, those located on the upper lip or involving the oral commissure have a 10-
20% lower 5 year survival. Overall survival for lip carcinoma is 91% however this drops down to 30-70% with advanced stage disease. Lip carcinoma develops regional metastases later than other site of cancer and has ~ 10% chance of regional metastasis overall. Basal cell carcinoma is more common on the upper lip compared to the
lower lip, however, it should be pointed out that the most common cancer of the upper lip is still squamous cell carcinoma given that fact that it represents 95% of all lip cancer. Lip carcinoma is the most common site for oral cavity cancer (oral tongue is the second most common comprising 30%). - See KJ Lee 10th ed pg 700-702.

150
Q

A 35 y/o male is involved in a car accident and is
found to have a right sided longitudinal temporal
bone fracture during his trauma workup. 6 weeks
later he presents for his recommended follow up
audiogram. He continues to have hearing loss at
this time and on tuning fork exam has a Weber
which lateralizes to the right and a negative Rinne.
What is the most likely finding on tympanogram?A) Ad B) As C) A D) B E) C

A

A; The patient has a classic presentation for ossicular discontinuity
which occurs commonly after longitudinal temporal bone fractures. In a conductive loss, Weber will localize to the affected side and Rinne
will be negative (i.e. BC>AC) if the loss is large enough. Ossicular discontinuity creates a type Ad tympanogram which demonstrates
increased tympanic compliance. Remember that the “d” stands for
discontinuity. “As” indicates decreased tympanic compliance and is
seen in otosclerosis (s for sclerosis). Type A is present in healthy
ears. Type B indicates a middle ear effusion or a perforation. Type
C indicates persistent negative pressure. - See KJ Lee 10th ed pg
40.

151
Q

Which of the following regarding the laryngeal
framework is false?
A) The cricoid is made of hyaline cartilage B) The
cricoid is the narrowest portion of the airway in
adults C) Ossification of the thyroid cartilage first
occurs at the inferior horn D) The larynx has three
paired and three unpaired cartilages E) The
laryngeal prominence is typically 120 degrees in
females

A

B; While the cricoid/subglottis is the narrowest portion of the airway
in the pediatric population as the laryngeal framework grows into
adulthood the glottis becomes the narrowest portion. All of the
cartilages of the airway are made of hyaline cartilage (except the
cuneiform and corniculate). Ossification of the laryngeal framework
starts with the inferior margin of the thyroid cartilage at ~ age 20
years old. The three paired cartilages include the corniculate,
cuneiform and arytenoids. The three unpaired cartilages include the
epiglottis, thyroid and cricoid cartilages. The laryngeal prominence is
120 degrees in women and 90 degrees in men (creating an Adam’s
apple) - See KJ Lee 10th ed pg 794.

152
Q

A 33 y/o female presents with left sided pulsatile
tinnitus. A CT scan is performed which
demonstrates irregular destruction of the left jugular
foramen. Given the most likely diagnosis, what is
the vascular supply for this tumor?
A) Internal Maxillary B) Ascending Pharyngeal C)
Artery of Pterygoid Canal D) Middle Meningeal E)
Occipital

A

B; Glomus tumors are on the differential for pulsatile tinnitus and demonstrate irregular destruction of the jugular foramen (unlike
schwannomas which create smooth widening of the foramen). The blood supply to glomus jugulares is the ascending pharyngeal artery.
- See Cummings 6th ed pg 2757.

153
Q

A 46 y/o female presents to the emergency room
with clear nasal discharge after undergoing
endoscopic sinus surgery at an outside
facility. She is diagnosed with a CSF leak and is
taken to the operating room for surgical
repair. Intraoperatively it is decided that intrathecal
fluorescein will be used to help identify the
leak. You are about to inject the fluorescein into
the patient’s lumbar drain when you remember that
fluorescein is neurotoxic and decide to double
check the dose. What is the appropriate dose of
intrathecal fluorescein?
A) 1mg B) 10mg C) 100mg D)1,000mg E) Dose
varies based on patient’s weight F) Continue
injecting until skull base defect identified

A

B; Intrathecal fluorescein is a useful tool to help identify the site of
small CSF leaks in the operating room. The standard dose that is
considered safe and effective is 10mg. This is typically given by
drawing up 0.1cc of 10% fluorescein and mixing it with 10cc of the
patients CSF before reinjecting it into a lumbar drain. The dose does
not vary based on the patient’s weight and it should not be injected
continuously or multiple times if no leak is identified. Fluorescein is
neurotoxic and can cause seizures if an inappropriate dose is
given. More common side effects include lower extremity weakness
and numbness. It should be noted that intrathecal fluorescein is not
a foolproof method for identifying CSF leak and failure to identify
fluorescein endoscopically does not rule out a CSF leak as there is a
26% false negative rate with this technique. - See Seth “The utility of
intrathecal fluorescein in cerebrospinal fluid leak repair” 2010.

154
Q

During excision of an acoustic schwannoma the
vein of Labbe is accidentally ligated. This is most
likely to cause infarction of what structure?
A) Frontal lobe B) Temporal lobe C) Parietal lobe
D) Cochlea E) Posterior Semicircular Canal

A

B; The vein of Labbe (aka the inferior anastomotic vein) provides a
connection between the transverse sinus and middle cerebral vein. It
is not always present and is found more commonly on the left (77%
vs. 66%). Ligation of this vessel can lead to poor venous outflow of
the temporal lobe causing edema and potential infarction. - See
Cummings 6th ed pg 2721.

155
Q

A 66 y/o male presents with multiple small masses
of the left face. He states that the masses have
been slowly growing over the past 15 years. He
also relays a history of surgery over 20 years ago
for a tumor on that side of his face but is not sure of
the final diagnosis. On exam he has a well healed
modified blair incision on the left as well as multiple
~1cm rubbery and mobile palpable masses in the
parotid bed. There is no cervical
lymphadenopathy. You explain to the patient that
these masses are likely due to the same disease
process as his original tumor but there is a chance
that it has progressed to a malignant lesion. What
is the risk that this patient has had malignant
transformation of his tumor?A) <1% B) 1.5% C) 10% D) 25% E) 50%

A

C; This patient’s presentation is most consistent with a pleomorphic
adenoma which is the most common neoplasms of the salivary
glands. If the capsule of the tumor is violated in surgery and tumor
contents spill into the wound bed there is a risk of seeding and
patients can present with multiple small recurrences in the parotid
bed. Pleomorphic adenomas do have a small but significant risk of
malignant transformation. This risk is ~ 1.5% within the first 5 years
of diagnosis, however tumors that are neglected and do not receive
treatment for 15 years or more have a 10% chance of malignant
transformation. - See Cummings 6th ed pg 1246.

156
Q

You perform Mohs surgery on a patient with a
nasal tip basal cell carcinoma. The best
reconstruction option is felt to be a full thickness
skin graft. You harvest and place the graft after
which the patient asks you how long it will take for
blood flow to return to the grafted tissue?
A) 24 hours B) 48 hours C) 3-5 days D) 7-10 days
E) 2-3 weeks

A

C; This question tests your knowledge of the stages of healing for
skin grafts. Initially, skin grafts obtain their nutrients via plasmatic
imbibition which supports the graft for the initial ~24 hours. The next
stage in healing is inosculation (24-48 hours), where the vessels from
the graft and the graft bed grow together end to end. Lastly, new
capillaries form and begin to establish a permanent blood supply for
the graft. This last steps begins 72 hours after grafting and is
completed during the first week. - See Pasha 3rd ed pg 465.

157
Q

What structure creates the inferior border of the
sinus tympani?
A) Mastoid Segment of Facial Nerve B) Tympanic
Segment of Facial Nerve C) Posterior Semicircular
Canal D) Ponticulus E) Subiculum

A

E; The borders of the sinus tympani are as follows: Inferior =
Subiculum Superior = Ponticulus Lateral = Mastoid Segment of
Facial Nerve Medial = Posterior Semicircular Canal The sinus
tympani is a difficult location to visualize during surgery for
cholesteatoma and is a common area for recurrence and residual
disease. - See KJ Lee 10th ed pg 3.

158
Q

You decide to take a Meniere’s patient to the
operating room for a labyrinthectomy. While drilling
out the bone contained within the superior
semicircular canal you encounter a vessel which
causes a significant amount of bleeding. What is
the name of this vessel?
A) Aberrant Carotid Artery B) Subarcuate Artery C)
Labyrinthine Artery D) Auriculotemporal Artery E)
Middle Meningeal Artery

A

B; The subarcuate artery can be found in a canal running through
the bone surrounded by the superior semicircular canal. This artery
is frequently used as a rough guide during labyrinthectomy by
keeping this area of bleeding in the center of the surgical field. - See
Cummings 6th ed pg 2666.

159
Q

A patient comes to the clinic to receive her routine
allergy immunotherapy injection for an allergy to
timothy grass. 15 minutes after the injection she
develops wheezing, urticaria and an impending
sense of doom. Which of the following is not a risk
factor for this condition?
A) Treatment during the pollen season B) First
injection from a new vial C) Poorly controlled
asthma D) Concomitant use of Lisinopril E)
Concomitant use of Metoprolol

A

D; This patient has symptoms concerning for anaphylaxis. This
occurs very rarely, some have estimated the incidence to be as low
as 5 events in every 1 million injections. Risk factors for the
development of anaphylaxis or increased severity of anaphylaxis
include asthma (especially if poorly controlled), dosing errors,
concomitant medications (especially B-blockers such as metoprolol),
injections during pollen season, the first injection from a new vial,
high sensitivity to the offending allergen, a history of previous
systemic reactions and large local skin reactions. Use of ACE
inhibitors (such as lisinopril) are not a risk factor for development of
anaphylaxis during immunotherapy. - See Lieberman “The risk and
management of anaphylaxis in the setting of immunotherapy” 2012.

160
Q

With regards to obstructive sleep apnea in the
pediatric population, which of the following is false?
A) Adenotonsillectomy cures 60% of patients B) An
AHI >5 is the threshold for diagnosis C) Daytime
fatigue is not an accurate indicator of sleep apnea
D) Most patients are not obese E) Males are
affected more commonly than females

A

B; In adults an AHI under 5 is normal, 5-15 is mild, 15-30 is
moderate and 30+ is severe. However, in children an AHI between
1-4 is mild, 5-10 is moderate and 10+ is severe. Adenotonsillectomy
is the first line therapy recommended for pediatric patients with sleep
apnea and cures ~ 60% of patients. Daytime fatigue is not a
common indicator in children and is present in only 13-20% of these
patients. Behavioral changes and poor school performance are
much more common. Only 25-40% of pediatric patients with OSA
are obese. Males are affected much more commonly than females
for reasons that are not entirely clear. - See Cummings 6th ed ch
184.

161
Q

A patient is seen in the clinic to discuss a total
laryngectomy for an advanced stage glottic
carcinoma. He is concerned about his ability to
communicate postoperatively. You discuss options
including a tracheoesophageal prosthesis (TEP).
What is the maximum phonation time for a patient
using a TEP?
A) Less than 5 seconds B) 16-17 seconds C) 20-22
seconds D) 55-65 seconds E) Indefinite

A

B; Phonation times using TEPs are pulmonary based and therefore
reach near normal levels. Normal maximum phonation is ~ 20
seconds and TEP users can voice for ~ 16-17 seconds. Use of
esophageal air for voicing produces very short phonation times
(usually 1-2 seconds) due to limited availability of air (just 60-80 ml).
Use of an electrolarynx leads to indefinite phonation time in theory. -
See Cummings 6th ed pg 1734-1735.

162
Q

While preparing to take a patient to the operating room
for functional endoscopic sinus surgery you review their
CT scan. You measure the lateral lamella to be 6 mm
deep. What Keros classification does this patient have?
A) Keros Type I B) Keros Type II C) Keros Type III
D) Keros Type IV

A

B; The Keros classification is divided into three categories (there is
no Keros Type IV). Each is based on the height of the lateral lamella.
Keros Type I = 1-3 mm Keros Type II = 4-7 mm Keros Type III = 8-16
mm This classification is used to help assess for the risk of skull
base injury and CSF leak. Patients with a high Keros classification
have more exposure of the lateral lamella that is susceptible to injury
and surgeons should be aware of this preoperatively in order to
mitigate this risk. - See KJ Lee 10th ed pg 371.

163
Q

With regards to the oncogenic pathway of HPV, E7

exerts its effect on what protein?A) p53 B) VEGF C) rB D) Raf Kinase E) Merlin

A

C; HPV is thought to promote oncogenesis via two proteins E6 and E7. E7 degrades the retinoblastoma protein (rB) which releases
transcription factor E2F and causes cell cycle proliferation. E6 degrades p53 which normally acts as a tumor suppressant. VEGF
and Raf Kinase are not involved in the HPV pathway. Alterations in the Merlin protein are responsible for NF2. - See Cummings 6th ed
pg 1080.

164
Q

A 46 y/o male presents with left sided ear pain and
facial nerve paralysis. On exam he is found to
have otitis media. You inform the patient that his
facial nerve weakness is likely due to irritation of
the nerve from the infection which is possible due
to a bony dehiscence of the fallopian canal. What
is the most common site of dehiscence of the facial
nerve?
A) Inferior to the oval window B) Superior to
the round window C) Just proximal to the first genu
D) Just distal to the second genu E) Just proximal
to the branch point with chorda tympani

A

B; The rate of facial nerve dehiscence varies from 20-50%. 80% of
the time, this dehiscence is found in the area superior to, or adjacent
to, the round window. 7% of the time it is dehiscent in the anterior
epitympanic space and 1% of the time it is dehiscent in the mastoid
segment. Patients over the age of 19 are 3.6 times more likely to
have facial nerve dehiscence than patients under the age of 18. -
See Cummings 6th ed pg 2198.

165
Q

Odontogenic infections which go untreated can
easily spread into surrounding soft tissue spaces.
Infections above which structure will spread into the
floor of mouth instead of the neck?
A) Mylohyoid B) Tooth Roots C) Digastric D)
Mandible E) Oral Mucosa F) Inferior Alveolar Nerve

A

A; The location of an odontogenic infection in relation to the
mylohyoid determines which space it can spread to. Infections above
the mylohyoid attachment to the mandible will spread into the
sublingual space whereas infection below it will spread into the
submandibular space. - See Cummings 6th ed pg 168.

166
Q

You are rounding on your post operative day #2
neck dissection patient when you notice that their
drain has some milky fluid in it. Over the next 24
hours there is 1.2 liters of similar milky output.
Which of the following is the next most appropriate
management option?
A) Medium chain fatty acid diet B) Short chain fatty
acid diet C) Compression dressing D) Octreotide
injections E) None of the above

A

E; Management of high output chyle leaks almost always requires
surgical intervention. There is no clear definition of how much output
is required to be considered, “high output” however various authors
have used 500, 600 or 1000cc over a 24 hour period. Chyle leaks
are more common on the left due to the presence of the thoracic duct
but up to 25% will occur on the right. A valsalva can be used
intraoperatively to assess for presence of a chyle leak and should be
performed at the end of a neck dissection if lymphatic duct injury is
suspected. Medium chain fatty acid diets, compression dressings
and octreotide injections all aim to decrease the flow of chyle and
help heal seal the chyle leak. These options would be more
appropriate in a patient with low flow leaks. Short chain fatty acid
diet is not a treatment for chyle leaks. - See Cummings 6th ed pg
1869-1870.

167
Q

A 72 y/o male presents to the clinic for evaluation
of an abnormal imaging finding. On review of his
chart he underwent a CT neck after minor trauma
which noticed a 1cm indentation in the
retropharyngeal space. The mass has well defined
borders and there is no abnormal
lymphadenopathy on the scan. The patient denies
dysphagia, voice changes, fevers, weight loss or
fatigue. He has no personal history of malignancy
and has never smoked. What is the most likely
diagnosis?
A) Tortuous Carotid Artery B) Foreign Body C)
Lipoma D) Reactive Lymphadenopathy E) Cervical
Osteophyte

A

A; Many processes can involve the retropharyngeal space, however
this patient is asymptomatic indicating a benign process. One of the
more common “pseudotumors” in this region is a tortuous common
carotid artery which can be pressed up against the retropharyngeal
space giving the impression of a mass. - See Cummings 6th ed pg
133.

168
Q

A patient with chemical workup consistent with
hyperparathyroidism is taken to the operating room
for parathyroidectomy. Intraoperatively the bilateral
upper and left lower parathyroid glands are found.
Frozen pathology is consistent with normal
parathyroid tissue. Exploration of the right lower
pole does not reveal a parathyroid gland. What is
the most likely location of the remaining parathyroid
gland?
A) Retroesophageal B) Intrathyroidal C) In the
carotid sheath D) Thyrothymic ligament E)
Retrolaryngeal

A

D; It is important to know both the normal and abnormal locations of
the parathyroid glands in parathyroid surgery. The inferior
parathyroid glands are derived from the 3rd branchial arch and
descend with the thymus. Therefore, glands that are not in their
typical location are most commonly found in the thyrothymic ligament
or further down in the thymus itself. Abnormal locations of superior
parathyroid glands include the retroesophageal and retrolaryngeal
regions as well as inside the carotid sheath. Parathyroid glands can
also occasionally be found in the thyroid capsule or in the thyroid
gland itself, however this is extremely rare. - See KJ Lee 10th ed pg
607.

169
Q

You are called to evaluate a patient with diffuse
facial trauma and concern for a naso orbital
ethmoid (NOE) fracture. Which of the following
exam findings indicate an NOE fracture?
A) Intercanthal distance 1/2 of interpupillary
distance B) Intercanthal distance 2/3rds of
interpupillary distance C) Intercanthal distance
equal to horizontal palpebral width D) Intercanthal
distance 1/2 of horizontal palpebral width E)
Intercanthal width of 30mm

A

B; Telecanthus (widening of the intercanthal distance) is a sign of
NOE fractures. This is caused by fracture of the lacrimal bone/frontal
process of the maxilla or complete avulsion of the medial canthal
tendon from these structures. Typically the intercanthal distance is
1/2 of the interpupillary distance and equal to the horizontal palpebral
width. In most people this is ~30mm. In NOE fractures the
intercanthal distance is widened making B the correct answer.
Another test that can be used to assess for the process of an NOE
fracture is to pull laterally on the medial canthal tendon and the
palpate it. If a “guitar string” is felt the tendon is attached however if
there is no resistance and no palpable tendon, suspicion for a NOE
fracture is heightened. - See Cummings 6th ed pg 332.

170
Q

In an adult patient, a lymph node in what area is
considered abnormally enlarged at 6mm?
A) Level IV B) Submental C) Retropharyngeal D)
Paratracheal E) Perifacial

A

C; In general cervical lymph nodes can enlarge up to 1cm and still
be considered normal. Jugulodigastric nodes can range up to 1.5cm
before they become concerning based on size criteria alone. The
exception to this rule is the retropharyngeal nodes which are
considered abnormally enlarged in adults if they are greater than
5mm. In children retropharyngeal nodes can range up to 1 cm and
still be considered normal. - See Cummings 6th ed pg 141.

171
Q

You evaluate a patient for vertigo in the clinic. On
Dix-Hallpike maneuver he demonstrates horizontal,
geotropic nystagmus with a short latency period.
On repeat exam his nystagmus does not
demonstrate fatigue. What structure is responsible
for this patient’s vertigo?
A) Superior Semicircular Canal B) Posterior
Semicircular Canal C) Lateral Semicircular Canal
D) Utricle E) Saccule

A

C; Benign paroxysmal positional vertigo (BPPV) can be evaluated
with use of the Dix-Hallpike maneuver. Most patients have posterior
semicircular canalithiasis which will demonstrate vertical upbeating
and torsional nystagmus, a latency period of ~10 seconds and
decrease nystagmus on repeat testing (fatigable). ~12% of BPPV is
caused by lateral semicircular canalithiasis which demonstrates
horizontal nystagmus (that can be geotropic or ageotropic), shorter
latency period and less fatigability. The superior semicircular canal is
responsible for only 2% of BPPV. Treatment for PC-BPPV consists
of the Epley and Semont maneuvers. LC-BPPV can be treated with
the log roll or lying on ones side with the affected ear up for several
hours. - See Cummings 6th ed pg 2551-2553.

172
Q

A 39 y/o male presents with left sided hearing loss
after suffering blunt head trauma. He also
complains of autophony and dizziness with
exposure to loud noises. On tuning fork exam
Weber lateralizes to the left, Rinne is positive b/l.
Given the most likely diagnosis which of the
following would you not expect to be true in this
patient?
A) Abnormally high VEMP thresholds B) No
abnormalities found on middle ear exploration C)
Acoustic reflexes present D) An air bone gap
primarily below 2000 Hz E) Negative bone
threshold below 2000 Hz

A

A; This patient presents with signs and symptoms consistent with
superior semicircular canal dehiscence (SSCD) including a
conductive hearing loss, autophony and Tullio’s phenomenon. Given
this diagnosis we would expect a normal middle ear to be found
intraoperatively, acoustic reflexes to be present (which differentiates
SSCD from otosclerosis), a low frequency conductive hearing loss
and suprathreshold bone lines. VEMP (Vestibular-evoked myogenic
potential) thresholds are typically low in these patients. - See KJ Lee
10th ed pg 248-249.

173
Q

A 4 y/o male is brought to the clinic for workup of
cervical lymphadenopathy. The patient’s mother
states he has had neck swelling present for the last
two weeks along with a persistent fever. His
symptoms have not resolved despite a course of
amoxicillin. On physical exam the child has
bilateral palpable lymphadenopathy, the largest of
which is ~ 2 cm. He also has erythematous
desquamated palms, conjunctivitis and a skin rash
over his torso. What is the next best step in the
management of this patient?
A) Admit for IV antibiotics B) Fine needle aspiration
C) Excisional biopsy D) CT neck with contrast E)
Echocardiogram

A

E; This patient’s presentation is consistent with Kawasaki disease
(aka mucocutaneous lymph node syndrome). Kawasaki is a
multisystem vasculitis most commonly seen in children less than 5.
Diagnostic criteria include a fever for 5 days or more and 4 or more of
the following: Conjunctivitis Strawberry tongue/fissured lips
Polymorphous exanthem (i.e. a widespread rash) Palmar
erythema/desquamation around the fingernails/nonpitting edema
Nonsuppurative cervical adenopathy >1.5cm Treatment involves
aspirin and IVIG however an echocardiogram should also be ordered
to evaluate the patient’s heart as Kawasaki is the most common
cause of acquired heart disease in the pediatric population. IV
antibiotics are not appropriate in Kawasaki patients. Further workup
with needle aspiration/biopsy/CT are not warranted given the patient
meets diagnostic criteria for Kawasaki disease. - See KJ Lee 10th ed
pg 822-823.

174
Q

A 21 y/o female presents with right sided hearing
loss which she noticed a few weeks after delivering
her first child. She denies any vertigo, tinnitus,
otalgia, otorrhea or aural fullness. On exam her
Weber lateralizes to the right and Rinne is
negative. What is the mode of inheritance of this
patient’s condition?
A) Mitochondrial Inheritance B) X- Linked
Recessive C) Autosomal Recessive D) Autosomal
Dominant E) This is not a genetically inherited
condition

A

D; This patient’s presentation is classic for otosclerosis. Pregnancy
has been found to accelerate the process of otospongiosis and many
patients will present with conductive hearing loss during or shortly
after their first pregnancy. Otosclerosis is an autosomal dominant
condition, however it has variable penetrance and expressivity. -See
KJ Lee 10th ed pg 345.

175
Q

An 8 y/o male is brought to the clinic for evaluation
of microtia. On physical exam he has down
slanting palpebral fissures, grade I microtia
bilaterally, and malar hypoplasia. An audiogram
demonstrates bilateral conductive hearing losses.
Which of the following is false with regards to
patients with this disorder?
A) Autosomal dominant inheritance B) 35% of
patients will also have a cleft palate C) 45% of
patients will have cognitive developmental delays
D) Increased rate of kyphosis E) All of the above
are true

A

C; Downslanting palpebral fissures and malar hypoplasia describes
the classic face of a patient with Treacher Collins. Microtia and
ossicular abnormalities causing conductive hearing loss are also
associated with this disorder and help confirm the suspected
diagnosis. Treacher collins is an autosomal dominant disorder with
variable penetrance. It is associated with palate clefting in 35% of
patients and some patients will have significant airway compromise
requiring surgical intervention due to the abnormal development of
the facial bones. Kyphosis is common in this population. This is
thought to be due to normal brain volume causing increased
resorption of the clivus during development causing flexion at the
skull base. These patients have normal intelligence. - See
Cummings 6th ed pg 2886-2887 and 2900.

176
Q

A 14 month old male with a history of prolonged
intubation is taken to the operating room for
evaluation of stridor. Your attending performs a
direct laryngoscopy and bronchoscopy and states
the child has a Meyer-Cotton Grade IV airway
before handing the instruments over to you. What
do you expect to see when inspecting the patient’s
airway?
A) 50-75% obstruction of the subglottis B) 75-99%
obstruction of the subglottis C) Complete
obstruction of the subglottis D) Partial obstruction of
the subglottis with extension of lesions into the
glottis E) Bilateral cricoarytenoid fixation

A

C; The Meyer-Cotton grading system is the most frequently used
method to describe subglottic stenosis. It is divided into 4 grades:
Grade I = 0-50% obstruction Grade II = 51-70% Grade III = 71-99%
Grade IV = 100% obstruction/No detectable lumen The McCaffrey
system (answer choices D and E) can also be used for
laryngotracheal stenosis and is based on subsite and length of
obstruction instead of lumen obstruction. - See Cummings 6th ed pg
983.

177
Q

A 2 year old with bilateral serous otitis media is
brought to the clinic for evaluation. On exam she
has frontal bossing, sunken bridge of the nose and
shortened limbs. Prior genetic testing revealed a
mutation in FGFR3. What is the mode of
inheritance of this disorder?
A) Sex-Linked Recessive B) Sex-Linked Dominant
C) Autosomal Recessive D) Autosomal Dominant
E) None of the above

A

D; This patient’s physical exam is consistent with achondroplasia.
These patients have mutations in the Fibroblast Growth Factor
Receptor 3 (FGFR3) gene. They will also have midface hypoplasia
but a normal sized mandible which makes the mandible appear
abnormally large. Remember that as a general rule of thumb,
disorders with visible physical stigmata tend to be autosomal
dominant. - See Cummings 6th ed pg 2886-2887.

178
Q

Which of the following syndromes can present with

multiple mucosal neuromas?A) MEN I B) MEN IIA C) MEN IIB D) MEN III

A

C; Multiple endocrine neoplasia (sometimes called multiple endocrine adenomatosis) has 3 variations. MEN I is made up of the
three Ps: Pituitary Adenomas, Pancreatic islet cell tumors,
Parathyroid hyperplasia. MEN IIA and IIB both present with
medullary thyroid cancer and pheochromocytomas. MEN IIA also has parathyroid adenomas (you can remember this because it is
“carried over” from MEN I). The unique feature of MEN IIB is
marfanoid habitus and mucosal neuromas. - KJ Lee 10th ed pg 239.

179
Q

Which of the following is not one of the medications
used in triple therapy to treat Helicobacter Pylori?
A) Omeprazole 20 mg BID B) Amoxicillin 500 mg
BID C) Clarithromycin 500 mg BID D)
Metronidazole 500 mg TID E) None of the above

A

E; All of the listed medications can be used as part of standard “triple
therapy” for H Pylori. A proton pump inhibitor such as omeprazole is
always used. Clarithromycin is preferred as well but can be replaced
by metronidazole if there is evidence of clarithromycin resistance. A
third antibiotic is also added which can be metronidazole or
amoxicillin. Note that only 3 medications are used in any given
treatment but that there are several options from which to choose to
create the treatment regime. Duration of treatment is controversial
but 14 days is most commonly used. - See http://emedicine.
medscape.com/article/2172395-overview?
pa=U9fXbjJqN0s0xTdakAgbvBksPaPbxUW5kLIi4AXD9XwMcboPY1
HB0GeYrkg902dPJyGvMX%2Fu%2BWdIXoARf%2FT0zw%3D%

180
Q

While performing both upper and lower lid
blepharoplasties on a patient you reduce the
volume of the orbital fat pads with bipolar cautery.
You are careful to avoid injury to an important
structure between the fat pads. Which of the
following accurately describes the relationship of
this structure?
A) Inferior rectus between the medial and central
inferior fats pads B) Superior rectus between the
medial and central superior fat pads C) Inferior
oblique between the medial and central inferior fat
pads D) Superior oblique between the medial and
central superior fat pads E) Lacrimal gland between
lateral and central superior fat pads

A

C; There are two superior (medial and central) and three inferior
orbital fat pads. Remember that the lacrimal gland fills in the space
where the third upper fat pad would otherwise go. The important
structure to be aware of when reducing the size of these fat pads is
the inferior oblique muscle which runs between the medial and
central fat pads. Damage to the inferior oblique can lead to diplopia.
- See Cummings 6th ed pg 444

181
Q

A 62 y/o male presents for workup of a thyroid
nodule. On ultrasound there is a 4 cm solid,
hypoechoic, ovoid mass with calcifications in the
left lobe. A fine needle aspiration (FNA) contains
cells which stain positive for amyloid. Which of the
following peripheral blood tests is most likely to be
abnormal in this patient?
A) Complete Blood Count B) Calcitonin C) Thyroid
Stimulating Hormone D) T3 E) T4 F) Serum
Amyloid A Precursor

A

B; A FNA with cells containing amyloid (birefringence green on
congo red staining) is pathognomonic for medullary thyroid
carcinoma (MTC). It should be noted that the absence of amyloid
does not rule out MTC. The diagnosis of MTC can also be made with
a peripheral blood test which is likely to show elevated levels of
carcinoembryonic antigen (CEA) and calcitonin. In fact, it is
polymerization of the calcitonin that creates the amyloid deposits. A
complete blood count is likely to be abnormal in a patient with
lymphoma which can also present with a thyroid mass but will not
show amyloid on FNA. TSH, T3 and T4 can be abnormal in MTC but
often times the patient is euthyroid. Serum Amyloid A Precursor can
be detected in patients with amyloidosis, but they are unlikely to have
a thyroid mass with amyloid in it. - See Cummings 6th ed pg 1917

182
Q

An 18 y/o presents with left sided hearing loss. On exam
a red blush is seen on the promontory, weber lateralizes
to the left and Rinne is negative AS. What is the most
common size of prosthesis that will improve this patient’s
hearing?
A) 4.25 x 0.6mm B) 4.75 x 0.6mm C) 4.25 x 0.7mm
D) 4.75 x 0.7mm

A

A; This patient has conductive hearing loss and demonstrates
Schwartze sign (reddish blush over promontory) which is caused by
active otospongiosis. Stapes piston prostheses are 0.6mm in
diameter (the drill used to make the stapedotomy is 0.7mm). Ideally
the prosthesis extends into the vestibule just 0.25mm. The most
common length is 4.25 mm although this is variable from patient to
patient. - See Cummings 6th ed pg 2215.

183
Q

A 49 y/o patient with male pattern baldness
presents for hair transplantation. The trichion
should be placed how far above the glabella for the
most natural look?A) 4 cm B) 6 cm C) 9 cm D) 11 cm

A

C; The trichion is the lowest point of the midfrontal hairline and is
typically positioned between 7-10 cm above the glabella. The
distances between the trichion and the glabella makes up the
superior 1/3rd of the face. In patients with more advanced hair loss
and/or limited donor reserves consider making the hairline higher. -
See Cummings 6th ed pg 376-378.

184
Q

A 25 y/o male presents to the trauma bay after
suffering blunt head trauma. During his workup he
is found to have a left sided temporal bone fracture
and corresponding facial paralysis. Which of the
following is the best prognostic indicator with
regards to this patient’s facial nerve?
A) Pt is otherwise healthy and a nonsmoker B)
Fracture involves the otic capsule C) Fracture is
categorized as transverse D) Rapid onset of facial
paralysis on the second day after admission E)
Facial paralysis noted by first responders in the
field

A

D; Delayed facial nerve paralysis almost always experiences a
complete recovery. Immediate onset paralysis is more likely to
represent a higher degree of injury to the nerve and is less likely to
recover. Otic capsule involving transverse fractures are more likely
to cause facial nerve paralysis. The fact that the patient has no past
medical history and does not smoke is unlikely to affect the course of
his facial nerve recovery. - See Cummings 6th ed pg 2224-2225

185
Q

A 2 y/o female patient is admitted to the intensive
care unit after undergoing an extensive surgical
repair of a congenital heart defect. The patient has
been intubated for 5 days and is expected to
require prolonged ventilation support. You are
consulted for tracheostomy placement. You
receive the consult on Friday afternoon but do not
have any block time in the operating room until the
following Wednesday. What is the increase in risk
that this patient will develop subglottic stenosis if
their tracheostomy is delayed until your block time? A) <5% B) 25% C) 50% D) 75% E) >90%

A

C; For every 5 days a child is intubated their risk of developing
subglottic stenosis increases by ~50%. 90% of subglottic stenosis in
the pediatric population is caused by intubation related trauma with
an incidence of 1- 8%. The subglottis is prone to trauma in children
due to the delicate pseudostratified respiratory epithelium and
submucosal areolar tissue which can quickly develop edema as well
as the complete cricoid ring which contains the edema in the airway.
Major factors that influence subglottic stenosis include length of
intubation, size of the endotracheal tube and movement of the tube.
- See Manica 2013.

186
Q

A 46 y/o male presents to clinic with complaints of
dizziness. After taking a detailed history you
discover his main symptom is Tullio phenomenon.
All of the following conditions can present with
Tullio phenomenon except…?
A) Syphilis B) Barotrauma C) Otosclerosis D)
Superior Semicircular Canal Dehiscence E)
Perilymphatic Fistula

A

C; Tullio phenomenon is vertigo caused by loud noise exposure. It
is caused by abnormal movement of the endolymphatic fluid in
response to pressure changes caused by loud noises. Hennebert is
similar but evoked by direct pressure changes. For Tullio
phenomenon to be present there must be an intact conductive
mechanism including tympanic membrane, ossicular chain and
footplate. Therefore, if a patient’s stapes footplate is fixed (as in
otosclerosis) they cannot have Tullio’s phenomenon. Syphilis,
superior semicircular canal dehiscence, barotrauma (that does not
interrupt the conductive mechanism) and perilymphatic fistula can all
present with Tullio’s. - See KJ Lee 10th ed pg 259.

187
Q

What is normal intraocular pressure?
A) <20 mm Hg B) 20-30mm Hg C) 30-40mm Hg D)
40-50mm Hg

A

A; Normal intraocular pressures are typically less than 20mm Hg
(and typically greater than 10 mm Hg). Anything over 20mm Hg is
considered elevated. In the setting of retrobulbar hematoma
pressure can rise significantly, compressing the ophthalmic arteries
and creating ischemia. Some authors have suggested that
intraocular pressures over 30mm Hg is an indications for surgical
intervention. Options for interventions include lateral
canthotomy/cantholysis, medial orbital wall decompression and use
of diuretics such as mannitol. - See Mohammadi Jama Oto 2015.

188
Q

Which of the following is false with regards to nasal
dermoids?
A) Nasal dermoids account for 1-3 % of all
dermoids B) Dermoids do not enlarge with crying C)
Only 50% of cases have a dimple at the rhinion D)
Hair protruding from a midline nasal mass is
pathognomonic for dermoids E) Nasal dermoids are
inherited in an autosomal recessive fashion

A

E; Nasal dermoids occur sporadically and are not inherited. There is
a slight male>female predominance. Nasal dermoids account for 1-3
% of dermoids however they represent ~ 10% of head and neck
dermoids. Dermoids do not enlarge with crying and do not
transilluminate which is important when trying to differentiate them
from encephaloceles. About half of nasal dermoids will demonstrate
a dimple however there is a wide spectrum of presentation.
Dermoids contain ectoderm and mesoderm and therefore often
contain hair, teeth, sebaceous glands and other structures. Unlike
dermoids, teratomas contain all three germ layers. - See Cummings
6th ed pg 2947.

189
Q

A 19 y/o male presents to clinic 2 months after
sustaining a head injury during a motor vehicle
accident. He complains of persistent hearing loss
since the accident but denies vertigo, otalgia, or
otorrhea. On exam his weber lateralizes to the
right, Rinne negative AD, positive AS. You decide
to obtain a CT temporal bone for further evaluation.
What will be the most likely finding?
A) Fracture of the stapes B) Fracture of the incus
C) Hemotympanum D) Incudostapedial dislocation
E) Incudomalleal dislocation

A

D; The most common cause of conductive hearing loss after
temporal bone trauma is hemotympanum, however by two months
post injury this should have resolved. The most common ossicular
chain injury is incudostapedial dislocation (82%) and is the most
likely source of this patient’s persistent conductive hearing loss. -
See Cummings 6th ed pg 2230.

190
Q

A 12 month old male presents to the clinic with
stridor. He is found to have a subglottic
hemangioma on exam with 60% obstruction of the
airway. After a discussion with the parents it is
decided to treat him with propranolol. The patient
weighs 8 kilograms. What is the appropriate
starting dose for this patient?
A) 1 mg QD B) 2.7mg QD C) 8 mg QD D) 2.7 mg
TID E) 8mg TID

A

D; The use of propranolol has radically changed the management of
subglottic hemangiomas given how effective it is at treating these
masses and preventing the need for surgical intervention. The
standard dosing is 1 mg/kg/day divided into three doses (i.e. 0.33
mg/kg/dose TID). This can then be titrated up to a maximum of 2-3
mg/kg/day (still divided into TID dosing) pending decrease in size of
the hemangioma. Common side effects include hypoglycemia,
bradycardia, hypotension, gastric reflux and bronchospasm. Patients
under the age of 8 weeks should initially be treated as inpatients, but
most others can be treated in an outpatient setting. - See Cummings
6th ed pg 3127-3128.

191
Q

A 52 y/o female presents for evaluation of sinusitis.
On review of her past medical history it is noted
that she has been diagnosed with systemic
vasculitis and asthma. A recent workup by her
primary care doctor revealed elevated eosinophils.
On exam she demonstrates diffuse nasal
polyposis. Which of the following is false with
regards to this patient?
A) Treatment may involve cyclophosphamide B)
The vasculitis involves small and medium vessels
C) The patient will likely have a positive C-ANCA
test D) The disease process is related to the gene
HLA-DRB4 E) The disease process can lead to
Loeffler syndrome

A

C; This patient has the classic triad for Churg-Strauss, eosinophilia,
asthma and vasculitis. The disease usually starts with a prodrome of
asthma and rhinitis followed by gastroenteritis and/or chronic
eosinophilic pneumonia (Loeffler’s syndrome) and ultimately results
in systemic vasculitis of small and medium vessels. Treatment is
similar to that of Wegner’s and involves corticosteroids and
sometimes cyclophosphamide. Mutations of HLA-DRB4 is the
underlying cause. C-ANCA is specific (but not sensitive) for
Wegner’s and will not be positive in Churg-Strauss patients. - See
Cummings 6th ed pg 203-204.

192
Q

A 52 y/o male is taken to the operating room for
treatment of chronic sinusitis refractory to medical
management. He is found to have diffuse nasal
polyposis and a full functional endoscopic sinus
surgery is performed. 1 week later the pathology
results return demonstrating inverting papilloma in
the left nasal cavity. You discuss these results with
the patient and explain that he will require a second
operation. Which of the following is true with
regards to inverting papilloma?
A) Recurrence rates are ~25% at 5 years postop B)
Dissection should be carried out in the
supraperiostal plane C) The incidence is 1 per
10,000 people annually D) The maxillary sinus is
the most commonly affected area E) There is a
<10% chance of concomitant squamous cell
carcinoma

A

E; Inverting papilloma (aka schneiderian papilloma) is a benign
tumor of the sinonasal tract. It is present in a small portion (1-5%) of
excised nasal masses and is important to identify on pathology due
to its association with synchronous malignant lesions. It was
previously thought that ~ 50% of inverting papillomas were
associated with malignant squamous cell carcinoma however more
recent data shows that it is far less. Current estimates range from
3.4-9.7%. Recurrence rates given modern endoscopic techniques is
~ 10%. A key aspect of the technique to remove inverting papilloma
is to dissect in the subperiosteal plane and drill down any involved
bone. The incidence is 0.6-1.5 per 100,000 people per year. The
lateral nasal wall in the posterior fontanelle is the most commonly
affected site, the maxillary sinus is the second most affected region. - See Cummings 6th ed pg 741-743.

193
Q

A 34 y/o heavy metal singer presents with
persistent hoarse voice. The hoarse voice has
been present for over 6 months. On flexible
laryngoscopy he has a right sided pedunculated
mass at the junction of the anterior and middle
1/3rd of the fold. The mass appears smooth
without erosion and he has no palpable cervical
lymphadenopathy. He would like to return to
performing as quickly as possible. What is the next
best step in management?
A) Voice rest, omeprazole and repeat exam in 6
weeks B) Voice therapy C) CT Neck with contrast
D) 10 days of Augmentin E) CO2 laser excision

A

E; The patient’s mass is most consistent with a vocal fold polyp.
These are often caused by vocal fold trauma (singing in a heavy
metal band) and can cause persistent hoarse voice. They are most
commonly located at the junction of the anterior 1/3rd and middle
1/3rd of the vocal fold. Unlike vocal fold nodules which are always
bilateral, polyps are unilateral. While vocal fold nodules can often be
treated with voice therapy, polyps rarely resolve with voice therapy
alone and surgery is often required. - See Cummings 6th ed pg
3141.

194
Q

A patient with an isolated zygomatic arch fracture
presents to the emergency room. You decide to
take him to the operating room for a Gillies
approach for fracture reduction. In preparation for
the case you review the anatomy in this region.
Which of the following describes the location of the
temporal branch of the facial nerve?
A) Deep to the deep layer of the deep temporal
fascia
B) Superficial to the deep layer of the deep
temporal fascia
C) Deep to the superficial layer of the deep
temporal fascia
D) Superficial to the superficial layer of the deep
temporal fascia
E) Deep to the superficial layer of the superficial
temporal fascia

A

D; Where exactly the facial nerve is located can become confusing
but it is important to know that the nerve lies above the upper layer of
the deep temporal fascia in the superficial fascia (temporal fascia).
The deep temporal fascia splits to form deep and superficial layers
(of the deep temporal fascia) wrapping around the temporal fat pad
and temporalis muscle. For this reason, when performing a Gillies
approach one must incise down to the temporalis muscle and stay
under the superficial layer of the deep temporal fascia in order to
avoid damage to the nerve. - See Cummings 6th ed pg 2681.

195
Q

A 32 y/o female presents for workup of a recurrent
thyroiditis. On flexible laryngoscopy she is found to
have a small opening in the left pyriform sinus and
you diagnose her with a branchial cleft anomaly.
Which of the following structures is derived from
the same branchial arch?
A) Handle of the Malleus B) Greater Horn of the
Hyoid C) Posterior Belly of the Digastric D) Tensor
Veli Palatini E) Corniculate Cartilages

A

B; Third branchial arch anomalies are much less common than
either 1st or 2nd branchial arch anomalies. They often present as
recurrent thyroiditis with fistula tracts opening into the pyriform sinus
(with the left side being more common). The third arch is the
embryological origin for the stylopharyngeus muscle, greater horn
and lower body of the hyoid bone, thymus, glossopharyngeal nerve,
common and internal carotid arteries. The handle of the malleus is
derived from the second arch whereas the head is from the 1st arch.
The posterior belly of the digastric has its origin in the second arch
(anterior belly is 1st). Tensor veli palatini is the only muscle of the
soft palate that is derived from the 1st arch, all the others come from
the 4th arch. The corniculate and cuneiform cartilages come from
the 6th arch. - See Cummings 6th ed pg 2828.

196
Q

A 44 y/o female presents to the clinic stating that
she does not like the appearance of her nose. You
perform a thorough facial analysis prior to offering
her a septorhinoplasty. Which of the following is
false with regards to the ideal aesthetic nose in this
patient?
A) The nasolabial angle should be 95-105 degrees
B) The nasofacial angle should be 115-135 degrees
C) There should be 2-4mm of columellar show D)
Nasal projection should be 60% of nasal length E)
The root of the nose should be at the level of the
superior palpebral crease

A

B; Complete facial analysis is an important part of the workup of any
nasal deformities. It is also a rich source of inservice and board
questions. A common mistake is to confuse the nasofacial angle with
the nasofrontal angle. The nasofrontal angle is between the nasal
dorsum and a line tangent to the glabella and this is ideally 115-135
degrees. However, the nasofacial angle is between the place of the
face (usually glabella to pogonion) and the nasal dorsum. The ideal
nasofacial angle is 36 degrees (although it can vary from 30-40
degrees). They are easy to confuse on an exam so make sure to
know the difference. All the other facial analysis measurements are
correct and are also common topics on board exams. - See KJ Lee
10th ed pg 757-758.

197
Q

You are consulted to evaluate a 31 y/o male who
suffered blunt head trauma. On exam his right
facial nerve is nonfunctional. Given the mechanism
of injury, what is the most likely area of injury to the
nerve?
A) Distal to the Stylomastoid Foramen B) 1st genu
C) Tympanic Segment D) Mastoid Segment E) 2nd
genu

A

B; The facial nerve is most susceptible to injury from blunt trauma at
the geniculate ganglion (1st genu). This may be due to several
factors including thin bone in this area which is susceptible to
fracturing as well as tethering of the genu by the greater superficial
petrosal nerve (GSPN) which anchors the nerve and may lead to
shearing or stretch injuries when trauma occurs. - See Cummings
6th ed pg 1985.

198
Q

A 52 y/o male presents to clinic with chronic right
sided otorrhea. He has a history of sudden
sensorineural hearing loss with complete deafness
in his left ear. On exam the left ear appears normal
however in the right ear there is keratin debris
inside a deep attic retraction pocket. A CT
demonstrates a small cholesteatoma limited to the
attic. The patient lives 4 hours away and you are
concerned about his ability to follow up with you
postoperatively. Which of the following is the most
appropriate surgical approach?
A) Simple Mastoidectomy B) Canal Wall Up
Mastoidectomy C) Canal Wall Down
Mastoidectomy D) Radical Mastoidectomy E)
Modified Radical Mastoidectomy

A

E; In a patient with a cholesteatoma limited to the attic in an only
hearing ear the best approach is a modified radical mastoidectomy,
also known as a Bondy procedure. In this surgical approach the
superior and posterior canal next to the cholesteatoma is removed
allowing it to be exteriorized, however the middle ear is not
manipulated in any way. This approach is only applicable to small
cholesteatomas limited to the epitympanum but is ideal in patients
where hearing preservation is needed. A simple mastoidectomy
(removal of mastoid cortex and limited air cells) is not appropriate for
cholesteatoma surgery as it does not provide access to the diseased
region. The fact that the patient lives far away and may have poor
follow up does help determine the need for canal wall up vs canal
wall down but both involve entering the middle ear which is not ideal
in this patient’s only hearing ear. Radical mastoidectomy involves
removing the ossicles and tympanic membrane as well as plugging
the eustachian tube leading to significant hearing loss and is not
appropriate in this patient’s remaining hearing ear if it can be
avoided. - See Cummings 6th ed pg 2190.

199
Q

A 22 y/o female develops a urinary tract infection
while traveling abroad and is treated with
gentamicin. She immediately develops significant
hearing loss and some disequilibrium. Upon her
return home she presents to your clinic for
evaluation. It is discovered that she has a positive
family history of hearing loss and you suspect that
she has a genetic mutation that causes increased
susceptibility of the inner ear to aminoglycosides.
What is the most likely mode of inheritance for this
patient’s genetic mutation?
A) Mitochondrial B) X-linked recessive C) X-linked
dominant D) Autosomal dominant E) Autosomal
recessive

A

A; While aminoglycoside medications can cause hearing loss in any
patient, there are some individuals with genetic point mutations (1555
A-G) in the mitochondrial 12S ribosomal RNA which makes them
significantly more sensitive to their effects. As this is a defect of the
mitochondrial RNA its mode of inheritance is mitochondrial and we
would expect all of these patients children to have this same
mutation. - See KJ Lee 10th ed pg 943.

200
Q

A 39 y/o male presents to the clinic with vertex
headaches and a CT scan demonstrating isolated
left sphenoid sinus opacification. You decide to
take this patient to the operating room for exam
under anesthesia including sphenoidotomy and
biopsy. Which of the following accurately describes
the location of the sphenoid ostium?
A) 5 cm from the nasal spine at 15 degree angle B)
7 cm from the nasal spine at a 30 degree angle C)
3 cm superior to choanal floor D) 1.5 cm inferior to
skull base E) 1 cm below posteroinferior aspect of
superior turbinate

A

B; There are many ways one can find the sphenoid ostium (besides
using imaging guidance). One classic description of its location is its
angle and distance from the nasal spine. This has been described as
~7 cm (6.2-8.0cm) at a ~30 degree angle (can range up to 34
degrees). The sphenoid ostium can also be located 1.5 cm superior
to the floor of the choana or 1 cm superior to the posteroinferior
aspect of the superior turbinate. It is almost always located medial to
the superior turbinate. There is not a described distance of the
sphenoid os relative to the skull base (likely because in most cases
the sphenoid is located prior to defining the skull base). - See
Cummings 6th ed pg 758.

201
Q

An 8 y/o female presents for evaluation of right
sided cervical lymphadenopathy. Her mother
states that she first noticed the mass 3 months ago
and it has roughly doubled in size since that time.
The patient is experiencing mild weight loss and
has had some intermittent fevers. She denies any
local trauma to the region. On exam there is a 4
cm firm, nontender mass anterior to the
sternocleidomastoid in level II with a violaceous
hue to the overlying skin. An ultrasound is
performed which demonstrates liquefactive
necrosis in the center of this neck mass. You
confirm the suspected diagnosis with a skin test.
Which of the following is not a reasonable next step
in the management of this patient?
A) Treatment with antibiotics only B) Observation
C) Incision and drainage followed by antibiotics D)
Excisional biopsy

A

C; This patient’s presentation is concerning for an atypical
mycobacterial infection. A patient with fevers/weight loss and
unilateral, enlarging, nontender neck mass with overlying
purple/erythematous discoloration should raise suspicion of this
diagnosis. A skin test can usually diagnose mycobacterial infections.
As lesions grow they can progress to suppuration. There is
controversy as to the ideal management however incision and
drainage is not recommended as it often creates a chronically
draining wound which prolongs the patient’s clinical course.
Antibiotic therapy with two antibiotics (including clarithromycin) is
often performed and with good results. Surgical excision and
curettage is often the treatment of choice and can significantly
shorten the patient’s clinical course. Observation will often lead to a
longer clinical course however it is a reasonable option in a stable
patient who is otherwise resistant to therapy. - See KJ Lee 10th ed
pg 822.

202
Q

A 3 year old male is brought to the clinic for evaluation of
hoarse voice. On flexible laryngoscopy multiple small
papillomatous lesions are found throughout the larynx.
You take him to the operating room for excision of these
lesions and send tissue for pathology. If PCR was used
on these specimens which HPV subtypes are most likely
to be found?
A) 16 and 18 B) 42 and 43 C) 6 and 11 D) 31 and
33 E) 45 and 52

A

C; Juvenile recurrent respiratory papillomatosis (RRP) is the most
common benign neoplasm of the larynx in children and the second
most common cause of hoarseness. 30% of patients will display
extralaryngeal involvement and providers should consider the
possibility of sexual abuse in children who present after the age of 7
(most patients present between 2-4 years old). RRP is caused by
HPV subtypes 6 and 11 most commonly. HPV 42/43 are low risk
HPV subtypes but are not commonly associated with RRP. Subtypes
16/18, 31/33 and 45/52 are known high risk subtypes for malignancy
but do not cause RRP. - See KJ Lee 10th ed pg 798-799.

203
Q

An 8 y/o male is brought to the clinic for evaluation
of hearing loss after head trauma. A CT scan is
ordered demonstrating a left sided vestibular
aqueduct with a diameter of 2.1 mm at the
midpoint. What gene is responsible for this child’s
condition?
A) SLC26A4 B) DFNB1 C) GJB-2 D) DFNA1 E)
COL2A1

A

A; Hearing loss after head trauma and a CT scan demonstrating a
vestibular aqueduct larger than 1.5mm at its midpoint (the width of
the horizontal canal) is consistent with enlarged vestibular aqueduct.
SLC26A4 is the gene responsible for this mutation. SLC26A4
mutations are also the cause of Pendred syndrome (hearing loss and
euthyroid goiter) and these two conditions are often seen together.
Both DFNB1 and GJB-2 are associated with connexin related
nonsyndromic autosomal recessive hearing loss. DFNA1 was the
first gene associated with hearing loss and is relatively rare.
COL2A1 causes the most common variant of Stickler syndrome
(hearing loss, ocular changes and joint disease). - See Cummings
6th ed pg 2565.

204
Q

A 21 y/o male with chronic sinusitis presents to the
clinic. On review of his chart you notice that he has
been symptomatic for many years despite maximal
medical therapy and 3 revision sinus surgeries. On
exam he has thick, purulent secretions sitting in his
maxillary sinuses despite widely patent ostiomeatal
complexes. You suspect a ciliary dyskinesia and
decide to perform a saccharine test. You place a
small amount of blue stained saccharine on the
head of his left inferior turbinate and wait to see
how long it takes for the patient to taste something
sweet. How long can transport of the saccharine to
the oropharynx take and still be considered within
normal limits?
A) 10 minutes B) 30 minutes C) 45 minutes D) 1
hour E) 12 hours

A

B; The saccharine test is a low tech way to assess mucociliary
clearance. Mucociliary flow moves at ~ 1 cm/minute. In normal
patients transport of the sweet tasting saccharin from the head of the
inferior turbinate to the oropharynx takes ~ 10 minutes, however the
test is still considered within normal limits up to 30 minutes. Blue dye
can be added to the saccharine and used to confirm the rate of
mucociliary transport with nasal endoscopy. - See Cummings 6th ed pg 645

205
Q

A 51 y/o female presents to your clinic with nasal
congestion, thick nasal discharge and loss of sense
of smell. She states these symptoms have been
present for 6 months now despite use of nasal
saline irrigation, flonase and loratadine. You offer
this patient endoscopic sinus surgery and order a
CT scan as part of her preoperative planning. On
review of the scan she has pansinusitis with
complete opacification of all of her sinuses. What
is her Lund-MacKay score?A) 12 B) 16 C) 20 D) 24 E) 28

A

D; The Lund-MacKay scoring system is the most widely used method to objectively evaluate radiographic studies of sinus disease.
6 sites are evaluated on each side including: Anterior Ethmoids Posterior Ethmoids Maxillary Sinus Sphenoid Sinus Frontal Sinus Ostiomeatal Complex Each site is given a score of 0, 1 or 2 for normal, partial opacification and complete opacification respectively.
Therefore in a patient with pansinusitis and complete opacification of
all her sinuses the maximum score is 24. -See Cummings 6th ed pg 666

206
Q

A 56 y/o male presents to the clinic with a left
parotid mass. Appropriate imaging and a fine
needle aspiration are performed and a final
diagnosis of T3N2bM0 mucoepidermoid carcinoma
is made. Regarding this patient, which of the
following is most likely true?
A) The tumor is less than 4cm in diameter B) The
tumor does not demonstrate extracapsular spread
C) There is bilateral lymphadenopathy, none
greater than 6cm in diameter D) Cancer stage is
IVA E) Histology may show tubular, cribriform or
solid growth patterns

A

D; T3 salivary gland neoplasms are stage III or higher. N1 lesions
are stage III but N2 bumps this up to Stage IVA. N3 lesions are
stage IVB and M1 disease is IVC. T3 lesions are larger than 4cm
and/or demonstrate extracapsular spread macroscopically
(microscopic extracapsular spread on final path does not count).
N2b disease is multiple ipsilateral lymph nodes none greater than
6cm. Bilateral lymphadenopathy is categorized as N2c disease.
Tubular, cribriform or solid growth patterns are descriptions of
adenoid cystic carcinoma, not mucoepidermoid carcinoma.
Mucoepidermoid carcinoma is classified as low, intermediate or high
grade based on multiple criteria (cystic components, tumor front
invasion, nuclear atypia, lymphatic/vascular invasion, neural invasion,
necrosis, 4+ mitosis by high powered field and bony invasion). - See
Cummings 6th ed pg 1261-1264.

207
Q

A 32 y/o female with extensive inverting papilloma
is taken to the operating room for endoscopic sinus
surgery. Surgery goes well and the patient’s nasal
airway is improved postop however at her 6 week
follow up she mentions that since surgery her left
eye has been feeling dry and she notices
decreased tearing on that side. Which of the
following structures was most likely damaged
during surgery?
A) Lamina Papyracea B) Lacrimal Duct C) Lacrimal
Sac D) Vidian Nerve E) Maxillary Nerve

A

D; The vidian nerve is formed by the greater petrosal nerve and
deep petrosal nerve. It synapses at the sphenopalatine ganglion and
then sends parasympathetic fibers to the nose and lacrimal glands to
stimulate secretions. Vidian neurectomy can be offered for
intractable rhinorrhea, however damage to the nerve can also cause
decreased lacrimation due to its innervation of the lacrimal gland.
This can occur in extensive sinus surgery, especially if bone is drilled
down as is often the case in treatment of inverting papilloma.
Damage to the lamina papyracea can place the contents of the orbit
at risk but would not cause decreased lacrimation. Damage to the
lacrimal duct is likely to cause epiphora. Given the lateral position of
the lacrimal sac it would be difficult to injure it during sinus surgery.
The maxillary nerve (V2) provides sensation but is not involved in
lacrimation. - See Cummings 6th ed pg 700.

208
Q

A 58 y/o female presents with unilateral nasal
obstruction. On exam a grey polypoid mass is
seen filling the left nasal cavity. A biopsy is
obtained and returns showing schneiderian
papilloma, exophytic subtype. Which of the
following is true with regards to this patient’s
diagnosis?
A) Arise most commonly from the maxillary sinus B)
Accounts for ~50% of sinonasal papillomas C)
~15% rate of malignant transformation D) Radiation
is the primary treatment modality E) 60%
recurrence rate

A

B; It is important to recognize that Schneiderian papillomas come in
3 subtypes, exophytic, oncocytic and inverting. We are most familiar
with inverting papillomas due to their high rate of malignant
transformation (~15%) however ~ 50% of sinonasal papillomas
consist of the exophytic subtype which is not known to undergo
malignant transformation (although there are a few case reports).
The oncocytic subtype is rare accounting for only 3-5% of these
tumors. Although inverting papilloma most commonly arises from the
maxillary sinus and lateral nasal wall, exophytic papillomas are
almost always based on the septum. Treatment for all of these
papillomas is surgical unless otherwise contraindicated. Inverting
papillomas have a recurrence rate of up to 60% however exophytic
subtypes are less likely to recur (~20%). - See Vorasubin
“Schneiderian papillomas: Comparative review of exophytic,
oncocytic, and inverted types” 2013.

209
Q

A 44 y/o male presents to the clinic with persistent
voice complaints after Type 1 thyroplasty. On
exam he is found to have a persistent posterior
glottic gap and he wants to know what can be done
to address this issue? Which of the following is
true with regards to this patient’s surgical options?
A) Arytenoid adduction mimics the action of the
lateral cricoarytenoid B) Arytenoid adduction cannot
address differences in the level of the vocal folds C)
Revision Type 1 thyroplasty is unlikely to result in a
significant improvement D) <3% of patients will
have incomplete glottic closure after type 1
thyroplasty E) Injection laryngoplasty is not a viable
option in this patient

A

A; Arytenoid adduction involves placing suture between the
muscular process of the arytenoid through the paraglottic space and
into the inferior thyroid ala. This internally rotates the arytenoid
causing adduction of the cord. This motion mimics the action of the
lateral cricoarytenoid. Done in conjunction with type 1 thyroplasty it
is felt to improve voice outcomes but can also increase the rate of
airway obstruction. Arytenoid adduction (unlike type 1 thyroplasty or
injection medialization) can address differences in the level of the
folds as its action moves the vocal fold caudally. Revision type 1
thyroplasty is a reasonable option for this patient and repositioning of
the existing implant or replacement with a larger implant will often
improve vocal quality. <3% of patients will suffer a significant
complication after type 1 thyroplasty, however 10-15% of patients will
have incomplete glottic closure. Injection laryngoplasty can also be
performed in patients such as this with counseling that the effect is
temporary. - See Cummings 6th ed pg 945-948

210
Q

A 34 y/o female with recurrent acute sinusitis
presents to the clinic for evaluation. She refuses
any surgical intervention due to a prior poor
experience with general anesthesia however her
symptoms are not currently controlled with maximal
medical therapy. She agrees to in office balloon
sinuplasty. During the procedure there is difficulty
inserting the balloon into the frontal sinus and you
review the CT scan before a second attempt.
Which of the following statements is true with
regards to the drainage pathway of the frontal
sinus?
A) Uncinate attachment to the skull base diverts
frontal sinus drainage to the middle meatus B)
Uncinate attachment to the lamina papyracea
diverts frontal sinus drainage to the middle meatus
C) Uncinate attachment to the middle turbinate
diverts frontal sinus drainage to the middle meatus
D) The anterior boundary of the frontal recess is the
ethmoid bulla E) The posterior boundary of the
frontal recess is the agger nasi

A

B; The superior extent of the uncinate process can attach to the
lamina papyracea, skull base or middle turbinate. This attachment
determines whether the frontal sinus drains laterally (into the
infundibulum) or medially (into the middle meatus). Attachment to
the lamina papyracea is the most common anatomic formation and
leads to frontal sinus drainage medial to the uncinate into the middle
meatus. Both attachment to the skull base or middle turbinate cause
frontal sinus drainage to be diverted laterally into the infundibulum.
This is an important relationship when attempting to locate the frontal
sinus ostium. The frontal recess boundaries are the agger nasi
anteriorly, ethmoid bulla posteriorly, lamina papyracea laterally and
middle turbinate medially. - See Cummings 6th ed pg 752-754.

211
Q

The ampullae of which two semicircular canals are
adjacent to each other?
A) Posterior and Lateral B) Posterior and Superior
C) Superior and Lateral D) None of the ampullae
are adjacent to each other

A

C; The superior and lateral semicircular canals end in ampullae that
are adjacent to each other in the anterior vestibule. The ampullae of
the posterior canal is located on the opposite end of the vestibule. It
should be noted that the superior and posterior canal share a
common crus where the nonampullated ends enter the vestibule. -
See Cummings 6th ed pg 2010.

212
Q

Which of the following is a symptom used in the
diagnosis of chronic rhinosinusitis in the adult
population but not in the pediatric population?
A) Anosmia B) Nasal Congestion C) Purulent Nasal
Discharge D) Facial Pressure E) Cough

A

A; By definition adult chronic sinusitis must present with at least 2
out of four of the following: Anosmia/hyposmia Nasal congestion
Purulent nasal discharge Facial pressure/pain In addition, these
symptoms must be present for at least 12 weeks and the patient
must have endoscopic or radiographic evidence of sinusitis. In the
pediatric population anosmia/hyposmia is not a common presenting
symptom and is difficult to evaluate. Instead, post nasal drip and
cough are much more common and should be considered symptoms
consistent with a diagnosis of chronic sinusitis. -See KJ Lee 10th
ed pg 409 and 785.

213
Q

What is the narrowest portion of the airway in a
neonatal patient?
A) Oropharynx B) Supraglottis C) Glottis D)
Subglottis E) Trachea

A

D; For patients age 0-5 the narrowest portion of their airway is the
subglottis (which corresponds to the cricoid cartilage). In older
children and adults the cricoid expands and the narrowest portion of
the airway becomes the glottis itself. This is thought to be part of the
reason neonatal patients are at elevated risk of developing subglottic
stenosis after intubation. - See KJ Lee 10th ed pg 794.

214
Q

A 22 y/o male presents to the clinic with complaints
of nasal obstruction. He also endorses clear nasal
discharge, watery eyes and sneezing that seems
worse in the early spring time. On exam he
demonstrates hypertrophic inferior turbinates and
boggy nasal mucosa. He has been treated with
flonase and claritin with limited success. You
decide to send him for allergy testing. Based on
your findings, which of the following is this patient
most likely allergic to?
A) Ragweed B) Oak Trees C) Timothy Grass D)
Dust Mites E) Cat Dander

A

B; The patient’s symptoms are consistent with allergic rhinitis
however these symptoms do not vary significantly amongst different
allergens. However, the seasonal nature and timing of his symptoms
are a clue as to a possible causative allergen. Allergies to tree
pollens present most commonly in the winter and spring (Feb - May)
and therefore is the most likely source of this patient’s allergic
symptoms. Grasses (Timothy, Johnson, Bermuda) cause allergies in
the late spring and summer. Weeds (Ragweed, Sage, Lamb’s
quarter) cause allergies in late summer and fall. Dust mites and cat
dander are perennial allergies and would be less common in a
patient with seasonal variance. - See KJ Lee 10th ed pg 462-463.

215
Q

A 62 y/o female with a history of Parkinson’s
disease presents for voice evaluation. On exam
she has a soft, monopitch, breathy voice with poor
articulation. On flexible laryngoscopy with
stroboscopy the vocal folds demonstrate bowing
and phase asymmetry. Which of the following is
the next most reasonable step in the management
of this patient’s voice complaints?
A) Lee Silverman Voice Treatment B)
Augmentation of Levodopa Dosing C) Injection
Vocal Fold Augmentation D) Pallidotomy E) Type 1
Thyroplasty F) Speech Therapy

A

A; Lee Silverman Voice Treatment (LSVT) was developed in the
1980s with the goal of improving speech in patients with Parkinson’s
disease but patients with severe depression and dementia have
found success with it as well. Treatment is intense (four times per
week for a month) and centers around increasing phonatory effort.
LSVT is the vocal training program of choice for patients with
Parkinson’s disease. Although this patient should receive treatment
(often with levodopa) for her Parkinson’s disease, changing this dose
is unlikely to significantly affect vocal quality. Injection vocal fold
augmentation or Type 1 Thyroplasty can be performed for vocal fold
bowing but would not be first line in this patient. Pallidotomy
(destroying the globus pallidus) is a neurosurgical procedure
sometimes used for the treatment of Parkinson’s but it has not been
shown to be effective in improving these patient’s voice. Speech
therapy would likely not be successful in managing a voice disorder.
- See Ramig “Parkinson’s Disease: Speech and Voice Disorders and
Their Treatment with the Lee Silverman Voice Treatment” 2004.

216
Q

A 33 y/o male with Stage IV squamous cell
carcinoma of the oral tongue presents to your clinic
to discuss treatment options. He is a well
respected chef and is strongly opposed to
glossectomy given his profession. You discuss the
risks of radiation therapy including the possibility of
osteoradionecrosis (ORN). Which of the following
is true of ORN?
A) The maxilla is the most commonly affected bone
B) ORN is more common with hyperfractionated
radiation C) Severe ORN occurs in 10% of patients
receiving conventional radiation therapy D) Healthy
teeth in the area closest to the tumor should be
removed prior to radiation to decrease the risk of
ORN E) Hyperbaric oxygen therapy has been
shown to have a small, but statistically significant,
improvement in ORN outcomes

A

B; Osteoradionecrosis is a feared side effect of radiation therapy and
most commonly presents as exposed bone in the oral cavity with or
without pain. ORN has been shown to be most common with
hyperfractionated radiation therapy (23%) compared to traditional
radiation (9-10%) especially if the doses are timed close together.
The mandible is the most commonly affected bone, likely due to its
unilateral blood supply and fact that the mandible is more commonly
directly in the field of radiation. While ORN occurs in 9-10% of
patients receiving traditional radiation therapy, only 2% have severe
ORN. While patients should receive dental care prior to radiation
including removal/repair of damaged teeth, healthy teeth do not need
to be removed. Hyperbaric oxygen is often used to treat ORN but to
date the data is mixed as to whether it is effective. - See Cummings
6th ed pg 1108. - See also http://www.newyorker.
com/magazine/2008/05/12/a-man-of-taste

217
Q

A 22 y/o male presents with a rapid onset left sided
neck mass. He states that this mass enlarged over
the last week and is acutely painful. On exam he
has a 3 cm soft and tender mass which lies just
anterior to the SCM in the mid neck. A CT scan is
ordered which indicates a cystic structure. You
suspect the origin is congenital and review the
scan to find the rest of the fistula tract. Which of
the following is consistent with the most likely
congenital fistula?
A) Medial to cranial nerve IX B) Passes through the
thyrohyoid membrane C) Terminates in the palatine
tonsil fossa D) Terminates in the pyriform sinus

A

C; A lateral neck mass anterior to SCM in the mid neck which
appears to arise due to an infection and is cystic in nature is most
consistent with a second branchial arch cleft cyst. Second arch
cyst/fistulae are the most common. They course from anterior to the
SCM then penetrate platysma and travel superiorly between the
internal/external carotids, over CN IX and XII and below the styloid
ligament to end in the palatine tonsillar fossa. Third branchial cleft
fistulas travel from the medial edge of SCM, lateral to the common
carotid but medial/posterior to the internal carotid, over CN XII and
under CN IX, through the thyrohyoid membrane and into the piriform
sinus. First branchial cleft anomalies present as preauricular
cysts/sinuses or duplications of the external auditory canal and can
be divided into Type 1 (duplication of ectodermal EAC) or Type II
(duplication of ectoderm and mesoderm). Fourth branchial cleft
abnormalities are exceedingly rare. - See Cummings 6th ed pg
2828.

218
Q

A 5 year old male is brought to your clinic for evaluation
of moderate bilateral mixed hearing loss. When reviewing
his chart he is noted to have an extensive ophthalmologic
history and is being treated for severe myopia and retinal
detachment. On exam he has a cleft palate, small chin
and bilateral serous effusions. The mutation of which
gene causes this patient’s condition?A) TCOF1 B) NF2 C) COL2A1 D) EYA1 E) PAX3

A

C; This patient has a presentation consistent with Stickler syndrome.
These patients develop early onset myopia, retinal detachment,
cataracts and blindness. They also have significant micrognathia
which can lead to clefting and eustachian tube dysfunction. Most
patients will have some degree of sensorineural hearing loss as well.
The responsible gene is COL2A1 and is inherited in an autosomal
dominant fashion. TCOF1 causes Treacher Collins syndrome. NF2
is the gene which is responsible for the protein merlin which is
abnormal in patients with neurofibromatosis type 2. EYA1 defects
cause Branchio-oto-renal syndrome. Both type 1 and type 3
Waardenburg syndrome are caused by PAX3 mutations. - See KJ
Lee 10th ed pg 120-122.

219
Q

A 62 y/o female presents for workup of asymmetric
hearing loss and chronic pain. An MRI is ordered
which reveals a 4x5mm left cerebellopontine angle
(CPA) tumor concerning for vestibular
schwannoma. The right CPA is clear. On review
of her history she was previously diagnosed with
several schwannomas of the spinal cord which had
previously been biopsied and are currently being
monitored. Which of the following genes is most
likely responsible for this patient’s vestibular
schwannoma?
A) NF1 B) GJB2 C) Merlin D) SMARCB1 E)
SLC26A4

A

D; This patient has a presentation consistent with Schwannomatosis
(sometimes referred to as NF-3). Schwannomatosis is a relatively
new diagnosis whose diagnostic criteria were first set forth in 2005
and were updated in 2011. The clinical diagnosis can be made two
ways: 1. “Two or more non-intradermal schwannomas, one with
pathological confirmation including no bilateral vestibular
schwannoma by high-quality MRI.” 2. “One pathologically confirmed
schwannoma or intracranial meningioma and affected first-degree
relative” The diagnosis can also be considered in patients with more
than one likely schwannoma but without tissue diagnosis if the
patient has concurrent chronic pain associated with the tumor. It is
thought that schwannomatosis arises due to genetic mutations in the
SMARCB1 and NF2 genes and requires four separate mutations for
the schwannoma to develop. NF1 is associated with Von
Recklinghausen Syndrome (neurofibromatosis type 1). GJB2 is the
gene responsible for connexin deficiencies leading to congenital
hearing loss. Merlin is the protein made from the NF2 gene which is
responsible for neurofibromatosis type 2. SLC26A4 is mutated in
Pendred syndrome and patients with enlarged vestibular aqueducts.
- See Plotkin “Updates From the 2011 International
Schwannomatosis Workshop: From Genetics to Diagnostic Criteria”
2011.

220
Q

In which method of facial analysis should the upper
lip be 4mm behind a tangent connecting the point
of maximal nasal tip projection with the pogonion?
A) Ricketts B) Goodes C) Burstone D) Steiner E)
Crumley

A

A; In the Ricketts method of facial analysis a line is drawn between
the nasal tip and the pogonion. On the ideal face the upper lip
should be 4mm behind this line and the lower lip should be 2mm
behind this line. Burstone analysis = Line is drawn between
subnasale and pogonion, upper lip is 3.5mm anterior to this line.
Lower lip is 2.2 mm anterior to this line. Steiner = If a line is drawn
between the inflection point on the columella and the pogonion, the
upper and lower lips should just touch this line. Goodes and Crumley
methods are for evaluating nasal projection. Goodes states that tip
projection should be 55-60% of nasal length. Crumley states that tip
projection to nasal height to nasal length should be 3:4:5. - See
Cummings 6th ed pg 454.

221
Q

A 36 y/o male is referred to clinic for evaluation of
sinusitis. He denies nasal obstruction or loss of sense of
smell but states that he has severe left sided headaches.
His headaches are always on the left side and have been
present for the last 4 months without remission. He
states the pain fluctuates and when his headache is
particularly bad his eyes water and he develops
rhinorrhea. Which of the following is the next best step in
management?
A) MRI Brain B) CT Sinus C) Sumatriptan trial D)
Indomethacin trial E) High flow oxygen

A

D; The patient has symptoms consistent with hemicrania continua
which is a primary headache disorder. Diagnostic criteria include:
Headache for more than 3 months fulfilling the other 3 criteria:
1. All of the following characteristics:
Unilateral pain without side-shift
Daily and continuous, without pain-free periods
Moderate intensity, but with exacerbations of severe pain
2. At least one of the following autonomic features occurs during
exacerbations and ipsilateral to the side of pain:
Conjunctival injection and/or lacrimation
Nasal congestion and/or rhinorrhea
Ptosis and/or miosis
3. Complete response to therapeutic doses of indomethacin,
although cases of hemicrania continua that do not resolve with
indomethacin treatment have been documented. It is important to
recognize hemicrania continua and paroxysmal hemicrania (similar
symptoms but repeated attacks for 2-30 minutes) because it is easily
treatable with indomethacin. - See Cummings 6th ed pg 250.

222
Q

Which of the following is true about chronic serous
otitis media but not of acute otitis media?
A) Otorrhea can be the only presenting symptom B)
Pseudomonas is commonly the cause C) Patients
are often afebrile D) Patients often do not have
significant pain E) None of the above

A

B; While chronic serous otitis media can be caused by many
organisms, Pseudomonas and Staph aureus are the most common
causes. Acute otitis media is most commonly caused by
Streptococcus pneumoniae (in addition to Haemophilus influenzae
and Moraxella catarrhalis; Streptococcus pyogenes). - See
Cummings 6th ed pg 2401 and 3021.

223
Q

An 8 month old female is brought to clinic with
concerns for hearing loss. The mother states that
her daughter passed her newborn hearing
screening but doesn’t seem to respond to speech
the way her other children did. You order an
Auditory Brainstem Response (ABR) which
demonstrates a normal response on the left but
complete absence of signal on the right. Which of
the following is the next best step in the
management of this patient?
A) MRI B) CT C) Repeat ABR D) Audiogram E)
Reassure Parents

A

A; In a newborn who passes a newborn hearing screening (often
performed using otoacoustic emissions (OAE)) but has absent ABRs
the most common diagnosis is auditory neuropathy. This is a rare
condition which presents with poorer word recognition than would be
expected based on pure tone averages. OAEs are often present as
this is a disorder of the auditory nerve and not the hair cells. It can
be unilateral or bilateral. A significant number (64%) of these
patients will demonstrate an abnormal finding on MRI and therefore
this is the next best step in the management of this patient. CT
scans can be ordered as well but are less likely to detect an
abnormality. Repeating an ABR would not be recommended as it is
unlikely to be a false positive and would only delay the patient’s care.
A full audiogram is a reasonable next step, however it would be
unlikely to provide you more information than you already have with
the ABRs and OAEs at this time. Reassuring the parents without
further workup is not a reasonable alternative. - See Roche “Imaging
characteristics of children with auditory neuropathy spectrum
disorder.” 2010

224
Q

Which branchial cleft sinus tract will pass under the
glossopharyngeal nerve?
A) First branchial arch cleft Type 1 B) First
branchial arch cleft Type 2 C) Second branchial
arch cleft D) Third branchial arch cleft

A

D; Third branchial cleft fistulas travel from the medial edge of SCM,
lateral to the common carotid but medial/posterior to the internal
carotid, over CN XII and under CN IX, through the thyrohyoid
membrane and into the piriform sinus. They are the only branchial
cleft tract that passes under the glossopharyngeal nerve. Second
arch cyst/fistulae are the most common. They course from anterior
to the SCM then penetrate platysma and travel superiorly between
the internal/external carotids, over CN IX and XII and below the
styloid ligament to end in the palatine tonsil. First branchial cleft
anomalies present as preauricular cysts/sinuses or duplications of
the external auditory canal and can be divided into Type 1
(duplication of ectodermal EAC) or Type II (duplication of ectoderm
and mesoderm). Fourth branchial cleft abnormalities are exceedingly
rare. -See Cummings 6th ed pg 2828.

225
Q

A 22 y/o male presents to the trauma bay status
post blunt trauma to the face. On exam he is found
to have telecanthus and imaging reveals a right
sided nasal orbital ethmoid (NOE) fracture. He is
taken to the operating room where you find a
comminuted fracture of the NOE complex and a
small piece of bone attached to the medial canthal
ligament. How would you classify this injury?

A) Chandler Group I B) Chandler Group II C)
Manson Type I D) Manson Type II E) Manson Type
III

A

D; The most common classification scheme for NOE fractures was
introduced by Markowitz and Manson in 1991. It revolves around the
connection between the medial canthal ligament and the NOE
complex. Class I = Medial canthus is attached to a large piece of the
NOE complex. Class II = NOE complex is comminuted but the medial
canthus is attached to a small piece of bone. Class III = The medial
canthal tendon is avulsed entirely from the bone. The Chandler
classifications refer to orbital complications from sinusitis. Chandler
group I refers to preseptal cellulitis and group II refers to
postseptal/orbital cellulitis. - See Cummings 6th ed pg 334.

226
Q

A 22 y/o male suffers blunt trauma to the left orbit
during a bar fight. On his CT scan obtained during
his trauma workup there is concern for entrapment
of the inferior rectus muscle. Which of the following
is the best test to confirm entrapment?
A) Forced duction B) Measure intraocular pressure
C) MRI D) Snellen vision test E) “H” test

A

A; Forced duction testing involves firmly grasping the sclera and
pulling the eye superior, inferior, medial and lateral. The eye should
move freely however if there is entrapment significant resistance will
be encountered. Occasionally, the entrapment can resolve while
performing this test. It is the best way to confirm entrapment
however it is uncomfortable for the patient so topical anesthetic eye
drops are required. Measuring intraocular pressure can provide
information regarding injuries to the globe but do not help confirm the
presence of entrapment. An MRI is not a practical test to confirm
entrapment and would not be expected to provide significantly more
information than a CT scan. The Snellen vision test measures visual
acuity but would not reflect the function of an extraocular muscle. An
“H” test would detect entrapment of the inferior rectus as the patient
would demonstrate diplopia on upward gaze, however this diplopia
could be due to other sources as well (hematoma, edema, nerve
damage, severed extraocular muscle) and is not specific to
entrapment. - See Cummings 6th ed pg 505.

227
Q

A 44 y/o female with chronic eustachian tube
dysfunction presents with left ear pruritus, otorrhea
and discomfort. She has a history of bilateral
tympanostomy tubes and has been treated on and
off for the past few months with topical antibiotic
drops for persistent otitis media. On exam she has
a patent tympanostomy tube on the left side with
black fluffy material present in the lateral canal.
Which of the following is a reasonable treatment
option for this patient?
A) Ofloxacin otic drops B) Vosol HC C)
Clotrimazole 1% otic drops D) Acetic acid 2% drops

A

C; This patient has persistent otitis externa after multiple rounds of
antibiotic otic drops and an exam consistent with fungal elements in
the ear canal. All of the answer choices except ofloxacin can treat
fungal otitis externa but this patient has a tympanostomy tube and
should only be given topical medications that are safe to enter the
middle ear. Acetic acid has been found to be ototoxic. Vosol HC is
made of acetic acid with hydrocortisone. Clotrimazole has been
shown to be safe in the middle ear of guinea pigs. - Tom
“Ototoxicity of common topical antimycotic preparations.” 2000

228
Q

A 44 y/o female is being treated with subcutaneous
immunotherapy (SCIT) injections in the clinic.
Shortly after her injection she develops a severe
headache, bounding pulse and impending sense of
doom. Which of the following is not an appropriate
measure to take in the management of this patient?
A) Place tourniquet proximal to SCIT site B) 8 mg
IV Dexamethasone C) 0.6 mg subcutaneous
Epinephrine D) Lay the patient supine E) Provide
oxygen via nonrebreather

A

C; While early administration of epinephrine is the most crucial
aspect in management of anaphylaxis the standard dose for adults is
0.3 mg, not 0.6. This is the dose in a standard EpiPen. The pediatric
dose is 0.15mg. Additional treatment of anaphylaxis include,
continuous monitoring of vital signs and airway, intubation if
necessary, laying the patient down, providing O2, placing a
tourniquet proximal to the injection site, obtaining IV access,
providing benadryl 50 mg IV, providing dexamethasone 8mg IV and
transferring to the ED/ICU. - See KJ Lee 10th ed pg 482.

229
Q

What does acoustic rhinometry measure?
A) Transnasal Pressure B) Peak Nasal Airflow C)
Nasal Resistance D) Cross Sectional Area

A

D; Acoustic rhinometry is an objective measure of the nasal airway.
It measures the cross sectional area of the nasal cavity by evaluating
changes in reflected sound waves after playing a “click” in the nose.
It is essentially nasal sonar. It is mostly used in research and does
not have a strong clinical role. It has been found to be useful in
identifying the presence of nasal obstruction but not its severity.
Transnasal pressure and nasal resistance are both evaluated by
rhinomanometry which measures pressure changes at the front and
the back of the nose. Peak nasal airflow is a measure of the
maximum flow through the nasal cavity but this is not measured by
acoustic rhinometry. - See Cummings 6th ed pg 648-653.

230
Q

A 59 y/o female presents with a large inverting
papilloma that requires extensive dissection of the
anterior skull base. During resection a CSF leak in
the region of the posterior ethmoid cells is
discovered. You decide to repair this defect with a
pedicled nasoseptal flap. Which of the following
endoscopic maneuvers would compromise the
viability of this flap?
A) Excision of the anterior middle turbinate B)
Excision of the posterior middle turbinate C)
Septectomy D) Removal of anterior sphenoid wall
to expose lateral wall of sphenoid E) Removal of
anterior sphenoid wall to expose floor of sphenoid

A

E; The nasoseptal flap, commonly used to repair CSF leaks, is a
pedicle flap that is able to cover most defects of the anterior skull
base. Its vascular supply is the posterior septal artery (branch of
sphenopalatine artery) which travels from lateral to medial across the
anterior wall of the sphenoid. Removing this wall in an inferior
direction to expose the floor of the sphenoid risk damage to the
vascular supply of this flap. Removing the anterior sphenoid wall in a
lateral direction is dissection in parallel to the artery and less likely
compromise the flap. A septectomy is commonly performed for
improved access and is not expected to interrupt this blood supply
nor is excision of any part of the middle turbinate. - See Cummings
6th ed pg 1198.

231
Q

Which of the following is not an ultrasound
characteristic of high risk thyroid nodules?
A) Solid B) Tall>Wide C) Hypoechoic D) Absence of
Halo sign E) Peripheral Vascularity

A

E; There are several ultrasound characteristics of high risk thyroid
lesions. Use the mnemonic SHITM&M. Solid Halo sign absent
Iso/hypoechoic Tall>Wide (on axial view) Margins irregular
Microcalcifications Vascularity can also be used to assess thyroid
nodules for malignancy, but the data is less robust and it is not used
commonly. Peripheral or absent vascularity is felt to be a sign that
the nodule is benign, whereas intranodular vascularity may be a sign
of neoplasm. - See 2015 ATA Thyroid Guidelines.

232
Q

A 48 y/o female presents to your clinic to discuss a
thyroid nodule found incidentally on PET scan. The
patient has a history of metastatic breast cancer
and underwent the PET as part of her post
treatment surveillance. A followup ultrasound
demonstrated a nodule measuring 1.1cm in largest
dimension. Which of the following characteristics, if
found on ultrasound, would indicate the patient
does not require a fine needle aspiration?
A) Nodule is 50% cystic B) Isoechoic C) Halo sign
is present D) The nodules Anterior-Posterior
dimension is larger than its Medial-Lateral
dimension E) None of the above

A

E; Many thyroid nodules are found incidentally on a PET scan.
Focal uptake in the thyroid gland is found on 1-2% of all PET scans
and warranties US evaluation of the gland as part of the workup.
Approximately 1 in 3 PET positive thyroid nodules will harbor
neoplasm and therefore fine needle aspiration is warranted in all PET
positive thyroid nodules greater than 1 cm regardless of the nodules
other characteristics. - See 2015 ATA Thyroid Guidelines.

233
Q

58 y/o male presents for evaluation of a left sided
thyroid nodule found on routine examination by the
patient’s primary care provider. On ultrasound the
nodule measures 1.9 cm in largest dimension and
is hypoechoic with irregular margins. A fine needle
aspiration is performed and the pathology report
states “Atypia of undetermined significance”. What
is the risk of malignancy in this patient’s thyroid
nodule?
A) 0-3% B) 1-4% C) 5-15% D) 15-30% E) 60-75%
F) 97-99% G) Requires additional information

A

C; The Bethesda system is used to report the results of thyroid fine
needle aspirations and categorizes FNA results into 6 categories
which have corresponding risks of malignancy. 1. Nondiagnostic = 1-
4% 2. Benign = 0-3% 3. Atypia of undetermined significant/Follicular
lesion of undetermined significance = 5-15% 4. Follicular neoplasm =
15-30% 5. Suspicious for malignancy = 60-75% 6. Malignant = 97-
99% - See Cibas “The Bethesda System for Reporting Thyroid
Cytopathology” 2009.

234
Q

A 45 y/o male with asymptomatic primary
hyperparathyroidism presents to your clinic for
evaluation of possible surgical intervention. He
denies any history of kidney stones, bone pain or
gastrointestinal difficulties. On review of his lab
values his Ca is 11.9 mg/dL, PTH is 151, creatinine
clearance is decreased by 25% and his 24 hour
urine calcium is 350 mg/dL. A DEXA scan is
ordered demonstrating a T-score of 2.2. Which of
the following is an indication for parathyroidectomy
in this patient?
A) Urine Calcium Level B) Creatinine Clearance C)
Serum Calcium Level D) Age E) Bone Density F)
PTH

A

D; Patients presenting with a workup consistent with primary
hyperparathyroidism (elevated serum calcium and PTH, normal
Vitamin D levels, decreased/low-normal phosphate) and symptoms of
hyperparathyroidism (stones, groans, and psychic overtone) are clear
candidates for surgery. However, in the asymptomatic patient (which
is more common) the indications are less clear. In 2002 the NIH
provided guidelines for indications for parathyroidectomy in these
patients. They include: 1. Serum calcium is greater than 1 mg/dL
above the upper limit of normal. 2.Creatinine clearance is reduced
more than 30% for age in the absence of another cause. 3.
Measurement of 24-hour urinary calcium is greater than 400 mg/dL.
4. Patients are younger than 50 years of age. 5. Bone mineral density
measurement at the lumbar spine, hip, or distal radius is reduced
more than 2.5 standard deviations (by T-score). 6.Patients request
surgery, or patients are unsuitable for long-term surveillance. Given
this patient is less than 50 surgery is indicated. His urine calcium,
creatinine clearance, serum calcium and bone density do not meet
criteria. PTH is used to diagnose hyperparathyroidism but is not
used to determine whether or not a patient is a candidate for surgical
intervention. -See Cummings 6th ed pg 1943.

235
Q

What is the name of the line that extends inferiorly
from the lateral semicircular canal and bisects the
posterior semicricular canal?

A

Donaldson Line The Donaldson line extends inferiorly from the
lateral semicircular canal and bisects the posterior semicircular canal.
It is used in ear surgery as a guide to find the endolymphatic sac.
The endolymphatic sac is always inferior to this line. The other
borders of the endolymphatic san include the sigmoid sinus
posteriorly, the jugular bulb anteroinferiorly and the mastoid segment
of the facial nerve laterally. - See Cummings 6th ed pg 2586.

236
Q

What seperates a standard facial recess drillout

from an “extended” facial recess drillout?

A

Sacrifice of the Chorda Tympani The facial recess is the area
defined by the facial nerve, incus butress and chorda tympani. This
allows access to the middle ear. When icreased exposure is required
the chorda tympani can be sacrificed to enlarge the opening and this
is referred to as an “extended” facial recess. - See Cummings 6th ed
pg 2725.

237
Q

Name the boundaries of the frontal recess.

A

Lateral = Lamina Papyracea Medial = Midde Turbinate Anterior =
Posterior Wall of Agger Nasi Posterior = Anterior Wall of Ethmoid
Bulla - See KJ Lee 10th ed pg 269

238
Q

The stalk of most juvenile angiofibromas can be

found eminanting from what structure?

A

Sphenopalatine Foramen Juvenile angiofibromas are benign, highly
vascular tumors which affect teenage males. They commonly
present with epistaxis and nasal obstruction and should not be
biopsied in clinic due to risk fo significant hemmorhage. Diagnosis
whould be made with imaging and the history. Treatment involves
rpeoperative embolization and excision (usually endoscopic). - See
KJ Lee 10th ed pg 782-783

239
Q

Name the three structures marking the posterior

boundary of the oral cavity.

A
  1. Tonsillar Pillars 2. Circumvallate Papillae 3. Junction of Hard and
    Soft Palate The oral cavity extends from the cutaneous vermilion
    junction of the lips anteriorly to the above structure posteriorly. There
    are 7 subsite of the oral cavity which include the lips, oral tongue,
    floor of mouth, buccal mucosa, alveolar ridge, retromolar trigone and
    hard palate. - See KJ Lee 10th ed 696.
240
Q

Describe the common findings on flexible

laryngoscopy in a patient with laryngomalacia?

A
  1. Omega shaped epiglottis 2. Epiglottis falls backwars on inspiration
  2. Collapse of the arytenoids Laryngomalacia is the most common
    cause of stridor in newborns and aften presents with intermittent
    inspiratory stridor that slowly resolves over the first few months of life.
    It is important to note that an omega shaped epiglottis can also be
    found in healthy infants and is not pathognomonic for laryngomalacia.
    Up to 50% of these patients will have a synchronous airway lesion so
    bronchoscopy may be indicated. Acid reflux is also more common in
    this group and treatment with proton pump inhibitors may improve
    symptoms. Most patients can be observed but for patients with
    aspiration, feeding diffculties and/or failure to thrive supraglottoplasty
    is the treatment of choice. - See Cummings 6th ed 2132-3124
241
Q

What is the innervation for Muller’s muscle and

Levator Superioris?

A
  1. Muller’s Muscle = Sympathetic Chain 2. Oculomotor Nerve (CN III)
    Muller’s muscle attaches from the undersurface of the levator
    aponeurosis to the superior tarsal plate and produces 2-3 mm of
    eupper eyelid lift. The levator superioris originates from the lesser
    wing of the sphenoid, extends anteriorly to Whitnall’s ligament where
    it makes a 90 degree turn and inserts on the superior tarsal plate. It
    is responsible for the majority of upper eyelid retraction. - See
    Cummings 6th ed pg 444.
242
Q

What is the rate of mucociliary flow in the nasal

cavity?

A

1 cm/min This knowledge is useful when performing a saccarine test
for ciliary dysfunction. A small amount of sweet tasting saccarine
placed on the inferior turbinate should take ~10 minutes to reach the
oropharynx (although up to 30 minutes is still considered normal). -
See Cummings 6th ed pg 645.

243
Q

The anterior ethmoid artery is a branch of what

vessel?

A

Internal Carotid –> Opthalmic –> Anterior (and posterior) ethmoid
artery This is important to know with regards to epistaxis
management. Embolization of the anterior ethmoid artery is
contraindicated due to the risk of retrograde flow potentially occluding
the opthalmic artery and resulting in blindness. Additionally,
performing a sphenopalatine ligation will not control epistaxis if it’s
origin is the ethmoidal arteries so consider ligation of these vessels
via a transorbital approach at the same time. - See https://en.
wikipedia.org/wiki/Anterior_ethmoidal_artery

244
Q

What is Poiseuille’s law?

A

Q = Pi x r^4 x (delta P) / 8 x n x l Q = Flow r = radius of the tube
delta P = difference in pressure between the begining and the end of
the tube n = dynamic viscosity l = tube length This equation is
important specifically for airway flow and blood flow. The key aspect
that is frequently tested on the inservice and boards is the fact that
even a small increase in the radius of the tube increases flow
significantly given that its impact is r ^ 4. - See Cummings 6th ed pg
1124.

245
Q

Name the four buttresses of the face that are
crucial to repair during open reduction internal
fixation of facial trauma.

A

1.Upper Transverse Midface Buttress 2.Lower Transverse Midface
Buttress 3. Lateral Vertical Midface Buttress 4. Medial Maxillary
Butress There are two vertical and two horizontal butresses: -Upper
Transverse Midface Buttress = Extends from the squamosal portion
of the temporal bone to the nasofrontal junction across the zygomatic
arch and inferior orbital rim. It extends posteriorly into the orbital floor.
-Lower Transverse Maxillary Buttress = Extends along the maxilla
above the alveolar ridge with posterior extension into the hard palate.
-Lateral Vertical Midface Buttresses = Vertical columns of bone from
the posterior maxillary molars across the zygomaticomaxillary suture
and body of zygoma extending superiorly along the lateral orbital rim
and across the zygomaticofrontal suture. It extends posteriorly into
the lateral orbital wall and lateral maxillary sinus.
-Medial Maxillary Butresses = Extends from the anterior nasal spine
along the rim of the piriform aperature, up to the frontal process of
the maxilla and across the nasofrontal junction to the frontal bone. It
projects posteriorly to the medial orbital wall. - See Cummings 6th ed
pg 339-341.

246
Q

Choanal stenosis is defined as a narrowed, yet

patent, choana measuring less than what width?

A

6mm While bilateral choanal atresia often presents early with
respiratory distress and difficulty feeding which improves withe
crying, unilateral atresia or stenosis can often present later in life with
rhinorrhea, recurrent sinonasal infections, mouth breathing,
eustachian tube dysfunction, and failure to thrive. While atresia can
be diagnosed with failure to pass a catheter, stenosis is better
delineated on a CT scan. - See Cummings 6th ed pg 2953.

247
Q

What is the name of the constriction at the superior
end of the superior constrictor muscle where the
palatopharyngus muscle fibers meet?

A

Passavant’s Ridge This ridge can be seen on the posterior
pharyngeal wall just inferior and posterior to the opening of the
eustachian tubes. It forms the inferior border of the adenoids. It can
assist in anterior-posterior closure of the palate. Damage to this
ridge may result in velopharyngeal insufficiency (VPI) and patients
with VPI may benefit from procedures to enlarge this ridge. - See KJ
Lee 10th ed pg 510.

248
Q

A 9 y/o male presents to the clinic for evaluation of
sensorineural hearing loss. On a detailed history
the patients mother states that the patient has
frequent fainting spells but no other medical issues.
What test should be ordered to confirm the
diagnosis? What medication should the child be
started on?

A

Electrocardiogram / Propranolol Sensorineural hearing loss and
syncopal episodes should raise concerns for Jervell Lange-Nielsen
Syndrome. This diagnosis can be confirmed with an EKG which will
reveal large T waves and prolongation of the QT interval. Because
this is potentially fatal, providers should have a low threshold for
ordering EKGs in cases of congenital hearing loss. The patients
cardiac condition should be treated with propranolol. - See KJ Lee
10th ed pg 123.

249
Q

What percentage of NF1 patients develop acoustic
neuromas? What percentage of NF2 patients
develop acoustic neuromas?

A

5% / 95% Neurofibromatosis type 1 is characterized by cafe-au-lait
spots, cutaneous neurofibromas, plexiform neuromas, lisch nodules
of iris and optic gliomas. It occurs in ~1:3000 people and 5% of
patients develop acoustic neuromas (usually unilateral).
Neurofibromatosis type 2 is much less common and occurs due to
deletions of the NF2 gene on chromosome 22q12. 95% of patients
with NF2 develop bilateral acoustic neuromas. - See KJ Lee 10th ed
pg 120-121.

250
Q

Which diagnosis has a poorer prognosis?

Hodgkins or non-hodgkins lymphoma.

A

Non-Hodgkin Lymphoma While Hodgkins lymphoma has a 5 year
survival rate around 90-95%, non-hodgkin lymphoma is much lower
at ~67%. Hodgkins lymphoma is more common in adults and
histology demonstrates the classic Reed-Sternberg cells. Non-
Hodgkin lymphoma is seen most commonly in children age 2 to 12
and is more ommon in males ompared to females. - See KJ Lee
10th ed pg 820-821.

251
Q

Name at least three benefits tracheotomy has over

intubation for prolonged airway management.

A

Advantages include all of the following: -Decreased dead space - Improve pulmonary toilet -Increased patient comfort and decreased need for sedation - Easier weaning from a ventilator -Decreased long
term airway complications such as subglottic stenosis It is generally recommended that tracheostomy be performed if more than 21 days of intubation are anticipated and that intubation is preferred if less
than 10 days of airway support is needed. The timing of when to perform a tracheostomy is controversial with some studies demonstrating decreased sedation requirements and shorter ICU stays if tracheostomy is performed earlier, while others did not find
this difference.

  • See Cummings 6th ed pg96.
252
Q

What are the two absolute contraindications to

cochlear implantation?

A

Michel Deformity and Cochlear Nerve Agensis Michel deformity involves congenital agenisis of the cochlea and therefore there is no neural elements which can be stimulated with a cochlear implant. Similarly, a narrowing of the internal auditory canal can represent absence of the cochlear nerve which also indicates absence of the necessary neural pathway. Most other congenital defects of the inner ear have at least some neural elements present making them relative contraindications to implantation. Labyrinthitis ossificans is not a contraindication to implantation, although it may significantly limit the depth of insertion. Anomalous facial nerves and thin cribiform region should be identified preopreratively and may increase the risk of surgery, but are not absolute contraindications.

  • See KJ Lee 10th ed pg 155.
253
Q

What are the four Centor Criteria?

A

1) Fever 2) Anterior Cervical Lymphadenopathy 3) Tonsillar Exudates 4) Absence of Cough If 3 of 4 are positive then the positive predictive value is 40-60% that Group A B-hemolytic Strep Pyogenes (GABHS) infection is present and treatment with antibitics is warranted (although many will still do a rapid strep test prior to
treating) . If only 1 of the 4 is positive there is a negative predictive value of 80% that the patient does not have GABHS and no further treatment is needed (again, some will still perform a rapid strep test.

  • See Cummings 6th ed pg 155.
254
Q

Name two advantages and one disadvantage to
using a Laryngeal Mask Airway (LMA) in securing
the airway.

A

Advantages: 1. Can be inserted blindly and quickly. 2. No neck movement required 3. 95-99% success rate 4. Easy to learn 5. Less postoperative sore throat, cough and laryngeal injury. Disadvantage: 1. Increased risk of aspiration of gastric contents compared with standard endotracheal intubation.

  • See Cummings 6th ed pg 71-72.
255
Q

What percentage of nasal fracture patients who
require intervention are satisfied with the results of
closed reduction and do not require additional
intervention?

A

87% Several studies support closed reduction as initial modality for nasal fractures. The timing of closed reduction is important as excessive delay (past 10 days) can make realigning the bones difficult due to the formation of scar tissue. Younger patients and athletes can heal particularly quickly whereas older patients can be reduced as time points more removed from their initial injury. Reduction within the first 1-2 days can also be difficult due to edema. 91% of pts are satisfied with the results of closed approach at 3 months and 87% at 3 years.

  • See Cummings 6th ed pg 498.
256
Q

What is the other name for the Nerve of Wrisberg?

A

Nervus Intermedius. This nerve is formed by general viscerel exits the brainstem adjacent to the motor branch of cranial nerve 7 and provides general visceral efferent fibers which are preganglionic parasympathetic to the lacrimal, submandibular, sublingual and minor
salivary glands.

  • See Cummings 6th ed pg 2665.
257
Q

What three cancers have dramatically higher rates

in HIV patients?

A

Kaposi Sarcoma, Non Hodgkins Lymphoma, Invasive Cervical Cancer. There are three aids defining malignancies. The most commonly known is Kaposi sarcoma which is an angioproliferative disorder characterized by dark lesions which demonstrate spindle cell proliferation on histology. Kaposi sarcoma is associated with HHV8 infections and 70% will involve the head and neck. Nonhodgkins lymphoma is also significantly increased in HIV patients and is associated with EBV infections. While both cervical cancer and oropharyngeal carcinoma are associated with HPV infections and both are more common in HIV patients, only cervical cancer is considered aid defining.

  • See Cummings 6th ed pg 180.
258
Q

What is the most common aerodigestive tract

foreign body?

A

Coins Aerodigestive tract foreign bodies are most common in boys under the age of 3. Esophageal foreign bodies are twice as common as bronchial foreign bodies are most commonly involve coins. Organic matter is more common in bronchial foreign bodies but commonly this is expelled via the cough mechanism. Other common objects include toys, batteries, fish bones and jewlery.

  • See Cummings 6th ed pg 3185.
259
Q

Name the four openings into the temporal bone.

A
  1. Internal Audiotry Canal 2. Vestibular Aqueduct 3. Cochlear Aqueduct 4. Subarcute Fossa -See KJ Lee 10th ed pg 3.
260
Q

What is the triad that characterizes Wegner’s

granulomatousis?

A

Vasculitis of the upper respiratory tract, lungs and kidneys. Wegner granulomatosis is characterized by necrotizing vasculitis of the small arteries. Head and neck manifestations include septal perforations, nasal congestion, loss of structural support of the nose, voice change and subglottic stenosis. 10-20% of Wegner patients will develop airway involvement and management involves endoscopic dilation techniques, although tracheostomy may be required.

  • See Cummings 6th ed pg 208.
261
Q

What are the criteria for diagnosing relapsing

polychondritis?

A

Per the McAdam criteria to diagnose relapsing polychondritis, 3 or more of the following must by present: 1. Auricular chondritis 2. Nonerosive seronegative inflammatory polyarthritis 3. Nasal chondritis 4. Ocular inflammation 5. Respiratory tract chondritis 6. Audiovestibular damage

  • See Medscape “Relapsing Polychondritis Clinical Presentation”
262
Q

Name the four embryological segments of the temporal bone?

A

Squamous Petrous Tympanic Mastoid - See Cummings 6th ed pg

1977.